You are on page 1of 230

Frequently Asked Questions in Management

IBS August 2003. All rights reserved. No part of this publication may be reproduced, stored in a retrieval system, used in a spreadsheet, or transmitted in any form or by any means electronic, mechanical, photocopying or otherwise without prior permission in writing from ICFAI Business School.

Ref. No. IBS 0803K2 2nd Edition, August 2003 For any clarification regarding this book, the students may please write to IBS giving the above reference number for this book including chapter number and page number. While every possible care has been taken in typesetting and printing this book, IBS welcomes suggestions from students for improvement in future editions.

PREFACE
As the students near the end of their SIPs, their focus gradually starts shifting towards getting suitable final placements. To perform effectively during the recruiters selection processes, the students have to equip themselves suitably. The recruiters assess the candidates from two perspectives. First they assess whether or not, the students meet their expectations in domain knowledge (both in theory and its application). The other aspect, which the recruiters generally assess, is the personality of the candidates. In this the recruiter is keen to find out whether the candidate is likely to fit into the professional and the social ethos of the organization.

Aim of this Book:


The requirement of domain skills would have been taken care of, by concerned faculty members, during the various courses. However, there is a necessity of viewing the same from an interviewers perspective. This book to a large extent fulfills the requirement. This book comprises of questions, which are generally asked, during interviews along with the suggested answers. However there is a word of caution to the users of this book. The aim of this book is not to provide readymade answers, for the students to learn by rote. The aim of this book is to provide, the students a suggested approach towards answering various questions. Therefore this is not to be treated as a readymade guide. The students are well advised to grasp the gist of both the question and the suggested answer. In some cases the suggested answer is fairly exhaustive. The student should consider the context in which the question has been put before replying. Therefore, the students should use the book judiciously.

Preparation for the Interview


There are some pre-requisites to be met by the students before they appear in front of a selection panel. 1. Information: Students must be well informed about the organization in which they are applying for a job. The information should include: a. c. e. The companys products/ services. The companys general performance in the last one or two years and the likely trend. The names of senior officials of the company, if possible. b. The companys competitors and their products/ services. d. The state of industry to which the company belongs. If you show that you are well informed, then you would demonstrate your level of interest and motivation. In case you do not have information, you will be giving an impression of a person, who is just a job seeker. 2. 3. 4. Job Requirement and Profile. Students must be aware of what is expected of them in the assignment for which they are applying. Preparation: There is no substitute for domain subject excellence. So better be prepared in all your subjects. Attitude: There is a subtle difference between self-confidence and arrogance. Show but do not show off. Otherwise you will irritate and annoy the interviewer.
i

5. 6. 7.

Facts and Figures: While showing your factual knowledge, you may speak out some facts and numbers. But dont overdo it or you might sound like an audiocassette. Disposition: Be pleasant and cheerful and dont be snobbish. Be pleasant even if there are irritations. Bear with them. Remember that the torture is not life long. Loyalty towards your Institution: Never ever talk ill about your institution. No one (absolutely no one) likes a person who is disloyal. In case you talk ill about your institution, the interviewers usually will most likely look upon you as a potential back-stabber. Information about IBS: Be fully informed about your institution, your course curriculum and USPs. Context: The suggested answers in this booklet are to be used judiciously. Answer to any question depends upon the context in which it is asked. The answers should not be learnt by rote. For example, a rhetoric question like say How is IBS doing nowadays? has to be answered in a different way from a question How is IBS today compared to five years back?

8. 9.

Layout of the Book:


For the convenience of the students, the book has been divided into various parts, each dealing with a particular discipline, e.g. Finance, Insurance etc. Further, each part has been subdivided further into various sections. The book has been divided in the following parts: Part - 1. Economy and Business Environment. Part - 2. Human Resources Management. Part - 3. Marketing. Part - 4. Finance. Part - 5. Insurance. Part - 6. IT & Systems In spite of our best efforts some errors might have crept into this work. These, along with any other suggestions may please be brought to the notice ICFAI Business School HQ.

ii

CONTENTS
Part 1

ECONOMY AND BUSINESS ENVIRONMENT


Economy.......................................................................................................................................... 3 Business Environment ........................................................................................................................................... 7

Part 2

HUMAN RESOURCE MANAGEMENT


Recruitment / Manpower............................................................................................................... 17 Rewards / Remuneration ............................................................................................................... 22 Teams ............................................................................................................................................ 26 Training ......................................................................................................................................... 27 Industrial Relations / Employee Discipline................................................................................... 30 Employee Legislation.................................................................................................................... 32 Organisation Types ....................................................................................................................... 35 Performance & Potential Appraisal / Career Planning / Leadership............................................. 37 Manpower Rationalisation ............................................................................................................ 42 Modern HR Concepts.................................................................................................................... 44 Part 3

MARKETING
Marketing General...................................................................................................................... 63 Strategy.......................................................................................................................................... 73 Retail & FMCG Mktg ................................................................................................................... 88 Sales & Distribution ...................................................................................................................... 91 Market Research............................................................................................................................ 96 Advertising and Branding ............................................................................................................. 99

iii

Part 4

FINANCE
Financial Accounting .................................................................................................................. 109 Management Accounting ............................................................................................................ 116 Banking ....................................................................................................................................... 120 Financial Market ......................................................................................................................... 130 Capital Market............................................................................................................................. 133 Debt Market................................................................................................................................. 146 Capital Structure.......................................................................................................................... 151 Corporate Finance ....................................................................................................................... 153 Project Management.................................................................................................................... 160 Hire Purchase / Leasing............................................................................................................... 163 International Finance................................................................................................................... 167 Derivatives .................................................................................................................................. 168 Forex............................................................................................................................................ 175 Futures......................................................................................................................................... 177 Part 5

INSURANCE ......................................................................................................................... 181


Part 6

IT & SYSTEMS
IT General ................................................................................................................................ 187 Internet & Web Technologies ..................................................................................................... 193 System Analysis and Design & Software Engineering.............................................................. 195 Operating System ........................................................................................................................ 200 DBMS.......................................................................................................................................... 201 Oracle and PL/SQL ..................................................................................................................... 207 Data Warehousing and Mining ................................................................................................... 213 C .................................................................................................................................................. 215 C++.............................................................................................................................................. 218 Java.............................................................................................................................................. 222 ERP & Oracle Financial.............................................................................................................. 225

iv

Frequently Asked Questions in Management

PART - 1

ECONOMY AND BUSINESS ENVIRONMENT

Economy & Business Environment

Frequently Asked Questions in Management

ECONOMY
Question 1. Answer: What do you understand by middle class? Give some economic indicators. The middle class comprises three sub-classes: the upper middle, middle middle and lower middle. And the whole middle-class is occupied by a socio-economic position intermediate between those of the lower classes and the upper class. Middle-class can be categorized by looking at availability or non-availability of the following amenities to each household in the country: Availability of a House and other Real Assets. Availability of Assets (Radio, Television, Telephone, Vehicles, etc.). Availing Banking Services etc. Though, there is no clearly defined economic indicator for classifying the class of people, but some reports gave the following statistics. The upper middle class should have annual income of US $600,000 each in terms of Purchasing Power Parity (PPP). (Please note that the calculation of PPP is complicated, but suffice it to say that it is based on what a unit of currency can purchase in one country compared to what the same currency can purchase in another country. It is also known as the "law of one price" that governs the price level of general goods and services between the two countries). The middle middle class should have annual income of US $20,000 per year each in terms of PPP. And the lower middle class is mostly the relatively affluent people in the rural areas of India. The middle classes on the whole (i.e. upper middle + middle middle + lower middle classes) is expected to grow by 5 to 10 percent annually. Question 2. Answer: What is Fiscal Deficit? The fiscal deficit is the difference between the government's total expenditure and its total receipts (excluding borrowing). The elements of the fiscal deficit are (a) the revenue deficit, which is the difference between the governments current (or revenue) expenditure and total current receipts (that is, excluding borrowing) and (b) capital expenditure. The fiscal deficit can be financed by borrowing from the Reserve Bank of India (which is also called deficit financing or money creation) and market borrowing (from the money market, that is mainly from banks). The government's revenue receipts consists of its tax revenues (net of the share of the states) and non-tax revenues like interest on loan given to states, dividends and interest paid by public sector firms etc. What is GDP and how it is calculated? Gross Domestic Product (GDP) represents the total market value of all final goods and services produced in a country in a given year. The GDP can be calculated in the following methods: GDP = C + I + G + NX. C -All household consumption of goods and services. I -All Investments made by Households and Corporates. G -All expenditure made by local, state and federal governments.

Question 3. Answer:

Economy & Business Environment

NX -Exports minus Imports. GDP = GNP -Net inflow of Labor and Property income from abroad. The Gross National Product (GNP) is the value of all the goods and services produced in an economy, plus the value of the goods and services imported, less the goods and services exported. It is important to differentiate between GDP and GNP. GDP includes only goods and services produced within the geographical boundaries of a country, say India, regardless of the producer's nationality. GNP does not include goods and services produced by foreign players in India, but does include goods and services produced by Indian firms operating in foreign countries. Example: Hyundai's Indian operation would account for India's GDP and South Korea's GNP. Question 4. Answer: What are various models for calculation and evaluation of price elasticity? Price elasticity (E) measures the proportional change in quantity with respect to a proportional change in price. E= (percentage change in quantity) / (percentage change in price). There are two types of price elasticity: Price elasticity of demand (Ed) measures the change in quantity demanded (Qd) with respect to the change in price (P). Ed = percent change in Qd / percent change in P Where Quantity demanded (Qd) is a specific amount that will be demanded per unit of time at a specific price. Price elasticity of supply (ES) measures the change in quantity supplied (QS) with respect to the change in price. ES = percent change in Qs / percent change in P Where Quantity supplied (QS) is a specific amount that will be supplied per unit of time at a specific price. Question 5. Answer: What do you understand by nominal and capital money? Nominal money means, money at its current value (not adjusted for inflation). When nominal money is adjusted with inflationary factor, then it is called real money. For example: Price of a Car is Rs 300,000 and rate of inflation is 5%. The nominal money you pay for the car is Rs 300,000. And the real money you will spend is Rs 300,000/1.05 = Rs 285,714 (approximately). Capital money means money utilized for capital or long-term purpose. Capital money is not something tangible, but is instead anything pertaining to a promoting means of production. Question 6. Answer: Please tell us how the credit rating of companies is carried out. Credit rating is an unbiased and independent opinion regarding company's creditworthiness or capacity to meet its financial obligations. Credit rating essentially

Frequently Asked Questions in Management

reflects the probability of timely repayment of principal and interest by a borrower company. The following steps indicate the process involved in credit rating: The rating process commences when a client approaches the credit rating agency to analyze and provide rating symbol/report to security /individual/borrower, etc. The rating agency then assimilates all the necessary information required for this by meeting the management of the company/borrower. The information provided for rating process will be highly confidential and not used for any other purpose Based on the analyses of the report the rating is decided The rating will then be communicated to the client along with the reasons supporting the rating. If this rating is not agreeable, the client may appeal for reviewing the rating for which additional information will be provided If the client accepts the rating it will then be declared to the public by credit rating agency Since rating is not one time process, there will be continuous monitoring of clients performance and its operational environment. CRISIL'S RATING PROCESS

Question 7.

What is the reason for the low interest rates in Japan?

Economy & Business Environment

Answer:

Japan is facing a deflationary situation and the economy is almost stagnant. There is a huge pile up of capital (savings deposits) in the country and absolutely no demand, because of global slowdown. Japanese companies are cutting their investments leading to a near zero interest rates. Another reason is pressure corning through the foreign exchanges. Twenty years of current account surpluses have led to a huge buildup of claims - mainly dollars - on foreigners. Because of ongoing fluctuations in the yen/dollar exchange rate, Japanese financial institutions will only willingly hold these dollar claims if the nominal yield on them is substantially higher than on yen assets. In the 1990s to 2002 as US interest rates have come down, portfolio equilibrium has been sustained only when nominal interest rates on yen assets have been forced toward zero.

Frequently Asked Questions in Management

BUSINESS ENVIRONMENT
Question 1. Answer: Tell us something about Kelkar Commission Report. The Kelkar Commission Report recommended on simplification of procedures for business transactions with respect to direct and indirect taxes reformation. The report focuses on improving the velocity of doing business in India, to create a hassle-free environment with minimal transactions and minimal human interface. On the front of direct taxes, report mentions removal of exemptions under I ONI OB for exports earnings for units under EOUs/EPZs and tax holiday under 80 IA and 80 ill; disallowance of interest cost on borrowings; restricting depreciation claim to the Companies Act only; withdrawal of tax concessions under Section 35 for investment for Scientific and Research purposes etc. On the front of indirect taxes, the report mentions of a two-tier duty structure for customs, 10% for input components and 20% for finished goods. Question 2. Answer: What are the IPOs that are suitable for the Indian market? The characteristics of IPOs that are suitable for the Indian market are: The size of float should be huge Competitive edge should be sustainable and scalable Valuations should be on the lower side Should be able to generate high return on equity and also do so on the back of huge capital intake Should have an up-to-scratch management team that's able to execute the opportunity at hand, via a complex business plan Should be practicing good management and governance practices, more in spirit than in letter, which is essential to attract capital. Question 3. Answer: What are the sectors that influenced the Indian economy in the last one year? The sectors that influenced the Indian economy in the last one year were: The manufacturing sector showed signs of an upturn. The two-wheeler, four-wheeler and the auto components industries have experienced growth. So also the pharmaceutical companies, cement industries, consumer durable industries etc. Strong housing demand and government-directed road projects have buoyed the construction industry. Telecommunications, financial services and hotels had experienced a rise in demand. Agriculture, which was doing fairly well, though the effect of the devastation, caused by the floods in Orissa and other regions would take time to be assessed. Infrastructure performance continued to be sluggish as usual, particularly the power sector. Boost food processing, rural industries and export sectors were also doing well.

Economy & Business Environment

In the recently announced Ql profits, only Infosys, Wipro and a few old economy companies, such as Reliance, ITC and HLL showed improvement. The capital market performance in the past nine months had been disappointing. Question 4. Answer: Do you think BPO will succeed in India? India is expected to hog this sector for the next 5-7 years. Competitors like Canada, China and Philippines suffer from different problems. Philippines and Canada lack scalability on the part of human resources while the Chinese lack English language skills. The factors that are reinforcing outsourcing to India are: India has the largest English speaking population in the world after the United States. High availability of educated customer care professionals and computer literates. Indian companies can provide call center services to clients based in the US or the UK at less than half of what it costs in US, UK or Australia. Per employee cost in US is approximately $40,000 while in India it is only $5,000. Indian companies are increasingly adapting to international quality standards. The Government of India has recognized the potential of IT-enabled services and has taken positive steps by providing numerous incentives. Question 5. Answer: As part of your job, you are likely to make a number of business reports and also analyze them. Can you briefly tell us how you make and analyze them? The information supplied in a business report should help users who need to make decisions about a company. To make good decisions, we need to unite the business information across every system, department, and location to give a consistent view. This ensures better, coordinated decisions through the enterprise. A good report allows topics of importance to be separated from other generic data. To present a good report, we need to understand the factors impacting the business. Key performance indicators should be easily sorted, filtered, graphed, and drilled down to fully understandable form, which will help in analyzing the business factors properly and convey the important message quickly. Question 6. Answer: Why do companies outsource? Mainly companies outsource their non-core activities to achieve competitive advantage due to various reasons; traditionally companies used to outsource just to reduce and control the operating costs. Now various other advantages can also be envisioned. The company can focus more on their core competencies and utilize their free internal resources for other fruitful purposes. The company can outsource best things from best people across the globe. It can gain access to the world-class capabilities and resources that are not available internally or the functions that are difficult to manage and the company can reduce the risks through sharing the operation. What do you understand by Business Process Reengineering?

Question 7.

Frequently Asked Questions in Management

Answer:

Business Process Reengineering (BPR) is the popular term for re-optimization of organizational processes and structures, following the introduction of new advanced technologies into an organization. BPR method utilizes a proven methodology to facilitate planning and deployment of a very effective business strategy by an organization, which wants to gain significant improvement in performance, cost and quality. This generally means the organization is seeking a performance gain across major segments of its operations. The purpose of reengineering is to make all the processes the best in class i.e. the organization should discover the best processes for performing work and reengineer them to optimize productivity. How do you define "core marketing" and "core finance?" Marketing is a process that starts from identifying the consumers needs to satisfying them through product / services. Successful marketing is getting the right product in the right place at the right price, and promoting it so that everyone knows about it which means getting the right mix of four Ps i.e. Product, Price, Place & Promotion. Finance can be defined as the corporate economy or how to manage the scarce monetary resources to get the maximum return. It covers everything from raising funds to effective deployment of fund.

Question 8. Answer:

Question 9. Answer:

What do you know about GATT and WTO? The General Agreement on Tariffs and Trade (GATT) was first signed in 1947. The agreement was designed to provide an international forum that encouraged free trade between member states by regulating and reducing tariffs on traded goods and by providing a common mechanism for resolving trade disputes. Eight major trade negotiations had taken place starting from first round in 1947 (Geneva), to its eighth round in Uruguay in 1986-94. Then it was converted into WTO. The World Trade Organization (WTO) began its operations in 1995 as a successor of GATT, which represents the rules-based regime of the policy of economic globalization. The central operating principal of the WTO is that commercial interests should supersede all others. Any obstacles in the path of operations and expansion of global business enterprise must be subordinated. Its membership includes 146 nations including US.

Question 10. Please describe the operating cycle. Give examples. Answer: Operating cycle of a firm begins with the acquisition of materials and ends with the collection of the receivables by selling the finished goods. It has four components: Cash, Inventory, Bills Payable, Bills Receivables. Operating cycle = + accounts payable (in days) -inventory replacement (in days) accounts receivable (in days) Example 1: You get 14 days of credit from your suppliers. It takes 21 days to sell your merchandise. You have an all cash business. How long is your operating cycle? Ans. Operating cycle = + 14 (accounts payable in days) -21 (inventory replacement in Days) -0 (accounts receivable in days). Operating cycle= -7 days

Economy & Business Environment

A negative operating cycle means you need to pay your bills before your customers pay you. As your sales volume increases, you will need more cash to keep up with your bills. Unless you are able to invest more cash, you limit your business's ability to grow. Most businesses have negative operating cycle. Example 2: You get 30 days of credit from you suppliers. It takes only 14 days to sell your merchandise. You give 14 days of credit. How long is you operating cycle? Ans. Operating cycle =+ 30 (accounts payable in days) -14 (inventory replacement in days) -14 (accounts receivable in days) Operating cycle=+ 2 days A positive operating cycle means that you can increase sales without needing additional cash. Beware, however, that your suppliers can disrupt your business by changing their credit policy. Question 11. What is BPO and its Emerging Areas? Answer: Business Process Outsourcing (BPO) has its origins in Total Quality Management (TQM) and Business Process Re-engineering (BPR). The idea is that, corporations can improve their efficiency by reshaping themselves and concentrating more on core activities and outsourcing the non-core processes. Companies turned to outsourcing because of the need to improve operational efficiency, optimize the use of capital, increase the organization's economic value-add and address the heightened pace of change resulting from deregulation and globalization. Major corporations in the US and Europe are outsourcing their back office operations to India to save costs, e.g. employee payroll is maintained in India for their employees worldwide. The demand for BPO services in functional areas such as finance, marketing, sales, human resources and administration, is expected to remain high for the forecast period of 2003-07 (IDC). According to The Economist, on the industry front more and more intellect intensive jobs will be moved from developed economies to BPO centers. It started with Medical Transcription, Accounting and Payroll processing and will extend to Business Studies, Legal and Medical Support. Question 12. What is the difference between stakeholders and shareholders? Answer: Shareholders are the owners of the company who invested money in the form of share capital. Stakeholders include the shareholders along with customers, employees, creditors, suppliers, community neighbors, and various governmental agencies. Shareholders are those who have voting rights and rights of electing the members of the board of directors. But stakeholders don't have such rights. Shareholders share both the profits and loss of the company. But the stakeholders if they share, then it is only the profits and not the losses of the company. Shareholders value is easier to define than stake holders value, so it is easier to account for shareholders than for stakeholders.

10

Frequently Asked Questions in Management

According to the performance metrics shareholders value is sufficient to maintain investors commitment. But for stakeholders there has to be fair distribution of value created to maintain commitment of multiple stakeholders. Question 13. Do you have any knowledge about Patents? Answer: Patents are the intellectual property rights, authorized by the government, which permits its owner to exclude members of the public from making, using, or selling the claimed invention. The patent owner may be forbidden from using the invention, usually due to the existence of another patent, or sometimes due to other legal restrictions. Most countries of the world have patent systems, although the patent terms and types of patents vary. Different types of patents are like follows. Utility patents: is granted to anyone who invents or discovers any new and useful machine, article of manufacture, processes (Methods), compositions such as chemicals, drugs etc., and improvements of known devices or new uses of old devices. The patent term is 20 years. Design patents: is granted to anyone who invents a new, original, and ornamental design for an article of manufacture. Term is 14 years. Plant patents: may be granted to anyone who invents or discovers and asexually reproduces (by means other than seeds) any distinct and new variety of plants. Term is 20 years. Question 14. How is industry analysis done? Answer: Analyzing the following factors can do industry analysis: I. Environmental Forces Affecting the Industry A. Political Environment B. Social\Cultural\Demographic Environment C. Technological Environment II. Market Analysis A. Market Size B. Current rate of Growth C. Growth Projections D. Major Segments in the Market E. Buying Processes and Considerations III. Structure of the Industry A. Exit/Entry Barriers B. Value-added chain C. Major buyers and suppliers and power of each D. Range of Substitutes E. Level of Rivalry F. Major Competitors G. Possible New Entrants

11

Economy & Business Environment

H. Global vs. Multi-domestic Operations I. Level of Concentration in Operations and Locations IV. Competitor Analysis A. Size of Company B. Location of Headquarters & Operations C. Importance of Industry in Company Operations D. Targeting\Positioning Strategy E. Distinctive Operational Policies F. Global Vs Multi-domestic Operations G. Level of Concentration in Operations and Locations H. Competitive Posture I. Prospects for Future Strategy and Likelihood of Success. Question 15. What is McKinsey 7S framework? Answer: McKinsey 7S framework provides a framework for measuring the strategic attributes (of an organization. It has seven elements like: Shared values: The employees share the same guiding values (Mission, Goals) Strategy: The integrated vision and direction of the company (Corporate, business, product/market) Structure: The policies and procedures, which govern the way in which the organization acts within itself and its environment. System: The decision-making systems within the organization (Process) Style: The employees shared and common way of thinking and behaving (Leadership & Culture) Skill: Employees have the skills needed to carryout the company's strategy (Competencies) Staff: Assigning the right job to the right person and training them (Empowerment). In 7S framework - strategy, structure and systems are considered as the hardware of success, while style, staff, skills and shared values are seen as the software of success. Companies in which these soft elements are present are more successful at the implementation of strategy. Question 16. What is the current oil price and the gold price? Answer: Gold prices (Mumbai): Rs. 5265/10gms. Crude Oil (World spot price): $26.51/ barrel. Source: CMIE, June 2003. Question 17. If you want to get into a state for doing business what are the factors you would take into consideration?

12

Frequently Asked Questions in Management

Answer:

Michael Porter's Diamond model can be used to evaluate the suitability of a state for business. Its four ingredients should be taken into consideration while evaluating various dynamics of business in particular country, state, region or locality. These ingredients are: I. Factor Conditions: A state creates skilled resources and technological base. Local disadvantages in factors of production sometime lead to innovation. Adverse conditions such as labor shortages or scarce raw materials force firms to develop new methods, and this innovation often leads to comparative advantage. Demand Conditions: When the market for a particular product is larger locally than in foreign markets, the local firms devote more attention to that product than do foreign firms, leading to a competitive advantage when the local firms begin exporting the product.

II.

III. Rivalry and structure: The rivalry within the industry in the home market prepares the companies to take on challenges of the global markets at a later stage. It hones their competitive strategies and prepares them to meet the competitive challenges of the global markets. IV. Supporting industries: helps and supports the main product. For the automobile industry, the supporting industries are the auto-components industry.

13

Frequently Asked Questions in Management

PART - 2

HUMAN RESOURCE MANAGEMENT

15

Human Resource Management

16

Frequently Asked Questions in Management

RECRUITMENT / MANPOWER
Question 1. Answer: What is Competency Mapping and what are its merits to an organization? The competency mapping concept means establishing different competencies covering all levels required in an organization. The competencies are then generally categorized into four groups: - attitude-based, knowledge-driven, skills-based and value-based. The skills and knowledge competencies vary according to the requirement of various positions. The idea is to ensure a perfect job- incumbent match as well as to increase motivation and productivity levels. Each position is profiled in terms of competencies, which will enable an organization to hire the right person, plug skill-gaps in terms of right training and development inputs, and pay equitably and competitively. The mapping process helps organizations in the long run to optimize human resources cost. What is KSA? Among the most fundamental and basic aspects on which an employees employability or selection is gauged is knowledge, skill and attitude. Employee involvement contributes to organizational effectiveness only to the extent that employees have the requisite skills and knowledge to make good decisions. In the organizations of the past knowledge and skill received the maximum attention and attitude was not focused much. However, in a reengineered and delayered organization all other things remaining equal attitude is the one which organizations focus the maximum. The emphasis on attitude is almost to the tune of 65 % with knowledge and skill getting 35%. The essence is that a person having the right attitude and moderate knowledge and skill can be molded for better performance .On the other hand a person having high knowledge and skill but poor attitude is observed to be a non-team player, high strung and is found to be a misfit. What is Psychometric testing? Psychometric literally refers to mental measurement and psychometric tests attempt to make objectives standardized measures of behavior. They are often divided into tests of general intelligence, tests of specific aptitudes and tests of personality, but tests purporting to measure qualities or attributes exist. Most of the valid and reliable psychometric tests are available only to suitably qualified, trained personnel in order to ensure that tests are used appropriately. Given the proliferation of occupational tests in recent years, it is best to consult a chartered psychologist. Measures of cognitive or mental abilities are increasingly used to provide data for decisions about selection, placement, training and development. Most research conducted over several decades has shown that cognitive ability tests are among the best predictors of job performance. Explain the meaning of Assessment Center? Assessment Center is not a place but a method or process designed to assess skills or potential in as comprehensive and rigorous a way as possible. A properly designed assessment center involves the assessment of groups of participants by a team of trained observers. Candidates take part in a series of specially designed exercises or activities. These may involve psychometric tests or interviews, but generally focus on situational exercises designed to resemble critical job situations as closely as possible. In this way

Question 2. Answer:

Question 3. Answer:

Question 4. Answer:

17

Human Resource Management

candidates can be rated on how they handle situations that resemble the job or job family they are being assessed for. Their performance in these situations or simulations is assessed against a set of job related skills (generally called dimensions or criteria) derived from a prior job analysis of the target job. Question 5. Answer: Explain the meaning of 16PF (Personality factors)? In attempting to uncover the major dimensions of personality, Mr. R. B. Cattell wanted to empirically determine and measure the essence of personality. Based on that Cattell concluded that the human personality has 16 basic dimensions. The product of his marathon task was the Sixteen Personality Factor Questionnaire, better known as the 16PF, which was subsequently revised following continued factor analysis. Consistent with the factor analytic strategy, items that correlated highly with each of the 16 major factors, or source traits, were included, and those with relatively low correlations were excluded. In the standardization of the 16 PF it is in fitness of things to find separate norms provided for men and women. 16 PF facilitates generating of a personality profile, which needs to be carefully interpreted. What is Myers Briggs Type Indicator (MBTI)? The Myers-Briggs Indicator is a diagnostic instrument with origins in Carl Jungs concept of personality types. This test is used in the area of organizational manpower. It uses combinations of scores from four major scales to identify orientation toward Extraversion or Introversion, Sensing or Intuition, Thinking or Feeling, and Judging or Perceiving. When shared in a group whose members have relatively high trust in each other and relatively high communication skills, this self-disclosed information can further tolerance and understanding between members, can be used by individuals to enhance strengths in deficient areas, and in some instances can be useful in sorting out team assignments. What is Thomas Profiling? Among the gamut of techniques that are supposed to ensure that the right person is hired for the job, Thomas International Profiling System is one. It facilitates best of the behavioral fit between person and the job. This technique was developed by the New York-based Thomas International Management Systems in 1945. Among its utilities are it creates a reliable picture of a candidate and provides insights into how an employee can be shaped after he joins. With an accuracy level of 85% global corporations extensively use this technique. This technique has the advantage of being modified to suit each companys requirements as well as each employees career aspirations. Thomas Profiling consists of two distinct components: The Personal Profile Analysis (PPA), which highlights the behavioral characteristics of the applicant. The Human Job Analysis (HJA), which identifies the behavioral requirements of the job. On combining these two components the outcome is that it brings people and jobs together in a compatible match that produces optimum job satisfaction as well as maximum productivity.

Question 6. Answer:

Question 7. Answer:

18

Frequently Asked Questions in Management

This technique is available as a PC-based software and hence can provide an accurate analysis. Question 8. Answer: Explain the importance of campus recruitment? As an HR professional how will you show-case an organization to the campus? Organizations always aspire to develop good managerial talent for their future needs. It may be called the talent pool or succession planning. In furtherance to this objective, leading organizations annually visit reputed campuses in the field of management, engineering and many other fields to select best brains that pass out of these institutions. It is common to see established companies and their arch rivals fighting it out in the campus wars. Consequently, searching or alluring the potential candidate from the campus is not a simple matter. In order to decide which campuses to visit organizations concentrate on such factors as curriculum, specialization, quality of classroom education, strength of faculty, and depth of support systems such as library, computer lab, etc. These super achievers from the campus are offered anywhere between Rs. 3 lakhs p.a to Rs. 10 lakhs p.a. depending upon the standing of the campus in terms of the performance of the students. Organizations in order to make a long lasting impression need to plan its campus strategy, namely - highlighting the corporate culture as a good reason to work for the company, focus on career growth opportunities, include the alumni in the recruiting team, explain the organizational environment in terms of levels of responsibility, degree of autonomy, extent of elbow room, and potential and scope for learning. What is the need for antecedent/reference verification before a person is offered employment? Antecedent means background. From the organizational point of view it is very much necessary and important to check the background in terms of experience, knowledge, skill, inter-personal relationship, attitude of the selected candidate from the previous organization before giving him the offer. The organization should contact through correspondence or telephone the concerned official of the previous organization to verify information relevant to it. Also the organization should write to the referees provided by the candidate in his employment application and seek their impressions about the candidate. In some instances organizations verify the background through investigative agencies or send their representative to the educational institute where the candidate passed out from or even go to the police in the candidates locality to verify the background. Antecedent verification is a kind of pre-emptive step to ensure that as far as possible the candidate it is recruiting is blemishless or does not have any skeletons in his professional cupboard.

Question 9. Answer:

Question 10. What are the sources of recruitment post-IT and Internet era? Answer: Apart from regular sources like internal search, advertisements, employee referrals, employment agencies, campus recruitments the most important at present is recruiting in cyberspace. The information superhighway is making this type of recruiting less of a necessity, in the competition to be bigger, better, faster and cheaper. Many organizations find that online recruiting is a valuable weapon.

19

Human Resource Management

Question 11. What is the underlying essence of referral and job posting system? Answer: The purpose or meaning of referrals is checking references to locate suitable candidates. Under this the potential employee is asked to give the names of senior professionals or eminent people excluding his relative who have known him over a reasonable period of time. The prospective employee mentions in the application form the names of references with whom the employer can discuss and check his credentials. Referral is also applicable in respect of potential candidates referred to for future employment in the company by the present employees. In some cases even the company extends some kind of incentive payment for having suggested or referred very good candidates. By this the company is able to make cost saving as otherwise it spends substantially on other modes of recruitment. The job posting is an internal arrangement within a company. Many organizations follow this arrangement whereby the existing internal employees are first given a chance to apply for any job opening within the company. The company displays a communication in the notice board giving the various requirements of the job. Such employees who meet the eligibility criteria will have to go through a test to find their suitability for the opening. This is a very good system, which helps in obtaining commitment and motivation of thee employees. Question 12. Explain the relevance and need of induction, orientation or organizational socialization of a new employee? How is important to an organization? Answer: First impressions count. The effectiveness or otherwise of induction training will greatly influence the impression which new entrants form of an organization from day one in their new job. Good induction training will also enable new entrants to make the right impression on their new colleagues and managers, as they are quickly able to learn the information and skills, which their new job requires of them.

Question 13. What is the difference between an induction manual and organizational manual? Answer: Whereas the HR manual is a consolidation of the core HR policies and is kept up to date on an ongoing basis, the induction manual (also known as EMPLOYEE HANDBOOK) contains information /details which a new employee would look or seek for on joining an organization. It contains among other things information about the company and its various divisions, organizational history, vision and mission statement, products, employment terms and conditions, health, welfare and safety facilities, motivational awards etc. The proof of efficacy of the induction manual is seen from the speed at which it contributes to the socialization and adjustment of the new employee to the ethos and culture of the organization.

Question 14. Of late the use of the words contingent manpower has increased in the context of manpower requirement of organizations? Explain. Answer: Globally, contingency manpower has become an accepted concept and is here to stay. Instead of going in for permanent employees organizations have started going for contingent work force-a just in time workforce for blue collar jobs as well as managerial

20

Frequently Asked Questions in Management

positions demanding specialized skills for a finite period of time paying a premium for them. The power of the temporaries is on the rise alongside outsourcing movement that has gathered lots of momentum. Not only organizations but also highly qualified professionals prefer to remain freelance players, as they do not want to be employed by a particular company. These high-strung individuals derive their motivation from challenging assignments and link themselves with assignments, which contains lot of intrinsic worth. Moreover with unique talent and high earning ability these individuals prefer to move away from the rigidity of a regular job so as to spend more time with their families. In India also the concept of contingent manpower is spreading its tentacles helping organizations in downsizing costs. Question 15. What is meant by zero-based manpower planning? Answer: Zero-based manpower planning is an extension of the technique zero-based budgeting. The underlying essence of the technique including its meaning in the manpower area presupposes that every function in an organization is unnecessary, until proved otherwise. The application of this technique purely in the manpower domain stresses on the importance of ongoing organizational vigil to measure effectiveness and value addition from each job to generate bottom line benefits. In simple terms it means that no manpower should remain unutilized and should be optimally utilized. It implies regular time bound survey for optimum utilization of the available manpower resources. This will lead to annual examination of all positions and may further lead to redeployment and reassignment of people according to organizational needs. All these will be focused while ensuring operations driven structure. Consequently this will contribute to decide about the need and extent of right, sizing and to maximize costefficiency without sacrificing on operations driven structure.

Question 16. In a nutshell explain the manpower profile of an employee in the call center and IT enabled services area? Answer: A young person below the age of 35 who is fluent in spoken English with a fair amount of western accent, willing to be a 24x7 working person with no mental blocks of working in any shift and possessing a fair amount of computer working knowledge is the nearest what could be stated as the profile of a call center or ITES organization employee. This kind of organization prefers students with English medium background with a minimum graduation as education qualification. The organizations pay well to attract the right kind of profile. However, the monotony of job, working in the night shift and no clear direction on the aspect of career growth has seen a high employee turnover.

21

Human Resource Management

REWARDS / REMUNERATION
Question 1. Answer: What is position analysis? Where is its use in HR? Position Analysis includes classification of abilities, skills, knowledge, and personality traits deemed relevant to work; factors relevant to the environmental and organizational context of work; and generalized work activities (broader descriptions of work tasks common across occupations). It is a method to evaluate the job. What is job evaluation? How is position analysis relevant to job evaluation? Job Evaluation: The methods and practices of ordering jobs or positions with respect to their value or worth to the organization. Summarize the overall purpose of the position and why it is necessary within the organization. The summary should be a concise description of the total position and should include only the most significant aspect of why the position exists in terms of its goals and objectives and its purpose. What is Hay Analysis in the context of job evaluation? Edward Hay, the founder of the Philadelphia US based Hay Group, developed Hay method of job evaluation. The Hay Analysis method of job evaluation uses a consensusbased analysis by a group of evaluators to measure the relative complexity of every job in an organization. At the heart of this kind of job evaluation are the points awarded to a job on the basis of complexity of three criteria: the know-how, the problem-solving skills, and the accountability levels that relate to a job. Each job has certain complexity, or size, which can be measured by a standardized and transparent evaluation technique that, in turn, yields a number. This final score is then a linear measure of the jobs complexity. So, a job with a score of 1,200 HAY points is half as complex as one with a score of 2,400 HAY points. HAY points can be related directly to the compensation that the job should be getting within the context of a particular company. Once all jobs in the company are awarded HAY points, its easy to prescribe the correct hierarchical levels within the organization. What is the meaning of Decision Band Method in the context of Job Evaluation? This refers to a tool propounded by Prof Thomas T. Paterson in the 1970s and improved upon by Ernst & Youngs Compensation Specialists for their client companies. It is based on the premise that the value of a job depends on its decision-making requirements. Decision-making scope is a rational and balanced approach in differentiating jobs, because every employee in the organization is expected and bound to make decisions of some kind in the process of their carrying out their job in any level of the organization. This method-envisages 6 levels of decision-making, or decision bands ranging from the most far-reaching or crucial decisions on organizational objectives to the simplest decisions like the speed of data entry. It consists of a 3-step evaluation process-first; jobs are classified into 6 different decision bands based on the characteristics and type or level of the decisions that a job requires. Second, jobs within each decision band can be classified into one of two grades coordinating or non-coordinating. Third or final, the jobs can be further subdivided within each grade into sub-grades based on specific position, relevant competencies like skills, effort, responsibility and working conditions.

Question 2. Answer:

Question 3. Answer:

Question 4. Answer:

22

Frequently Asked Questions in Management

Question 5. Answer:

What is internal equity, external equity, individual equity and cost of living equity in the context of wage and salary fixation? The above equities are important part of the alignment of evaluation and compensation systems with one another and with their companys business goals. Today, rewards for an employee covering both monetary and non monetary recognition must fulfill four objectives: they must meet his basic needs and conform to the three parameters of external equity-fairness vis--vis compensation standards in the industry, internal equity a fair deal for him within the organization in comparison to his colleagues, and individual equity - reward his unique contribution to the company. Importantly, while the first three factors are determined by broad forces such as the cost of living, the rate of inflation, pay scales in the industry and pay scales within the organization, the company can link the fourth component of employee rewards directly to its strategic goals by pegging it to employees contribution towards achieving them.

Question 6. Answer:

What constitutes employee cost or cost to the company (CTC)? Employee cost consists of every conceivable incurred cost in maintaining an employee. Components of Cost. Employment costs fall into several broad categories: Recruiting Expenses: Finding technically qualified people who can function effectively in a rapidly growing start-up venture is not easy task. Devote the time to make sure that your hires are as close to perfect as possible. Anything less will be a drag on your business. Basic Salary: Basic salaries vary all over the place depending on the industry and a variety of other factors. There are data, which can help you calibrate an appropriate base salary. Be sure to establish rational salary ranges given your growth plans. This means that in most - cases there should not be great salary differentials between early hires and later employees - any "risk component" of being an early hire should be made up in the equity compensation component. Employment Taxes: In preparing your personnel budget be sure to include allowances for social security. Benefits: Basic salary and employment taxes are a minimum - in most cases you will need to provide some benefits. Typical benefits include life insurance and health coverage. Other benefits could include long-term disability insurance, dental plans, dependent care assistance, tuition reimbursement, retirement plans etc. These involve actual payment of benefits by the employer. There are also "self - funded" plans where the employer - contribution is the administrative costs - e.g. vacation is another cost but is subsumed in the basic salary. Unless you are hiring traveling salespeople, you need to provide some physical space to house the new employee. Obviously the rent per square foot varies depending on the fanciness and location of the facility. But how many square feet does an employee need? Again this varies but there are some guidelines. Work cubes are typically 8' x 8' in size and private offices range in size, in high tech figure on 225 to 250 square feet per employee when you add in common space Other Equipment: The basics these days for high tech or office workers has to include a computer and telephone. Even with decreases in PC prices, figure for a computer, for software and for telephone handset on average when you factor in installation. Dont

23

Human Resource Management

forget the periodic expensive upgrades you will need to your LAN and voice mail systems. Other Approaches: Instead of figuring out each cost component separately you can develop some simple metrics. For example a different metric for planning the manufacturing side of the business. It takes some time to really understand the cost components of business segments in order to develop the metric and the underlying assumptions need to be recalibrated periodically. Hiring is always a difficult and important activity, especially in a rapid growth business. There is always too much to do and often the new entrepreneur has little experience in hiring. It is the rare case when you can hire exactly to meet your needs. Delay in hiring and you will have trouble meeting your plan, hire too quickly and you may burn through cash before revenue catches up. Consider using independent contractors to cover peak periods if your business does not have too steep a learning curve. Although you cant control the hiring process exactly you can understand the economic metrics of what you are doing and that can help you avoid some really unpleasant results. Question 7. Answer: What are benefits and services? Benefits are important means by which organizations successfully attract, motivate and retain employees. The employers should establish and maintain an employee benefit program that rewards the efforts of employees in the organization and in comparable and competitor organizations in tandem with the prevailing market price for the knowledge, skill and attitude. Employees seek benefits and services from the employer to take advantage of low costs, taxes, and inflation protection. What is the cafeteria approach to fixing employee compensation? Many companies now follow the cafeteria approach in providing benefits to their employees. This approach lets the employees choose benefits from a variety of benefits made available to employees. Each employee chooses the benefits that he or she prefers. Factors such as an employees age, financial and family position, tax planning, savings plan, whether both husband and wife are working, attitudes and lifestyle influence choice of employee benefits. What is the meaning and purpose of Employee Stock Option Plan (ESOP)? The purpose of this Stock Option Plan is to attract and retain key employees of the organization. Share ownership offers employees a real stake in their company. Increased commitment will lead to increase the productivity of the employee.

Question 8. Answer:

Question 9. Answer:

24

Frequently Asked Questions in Management

Question 10. What do you understand by non-financial benefits? Give a few examples of it? Answer: Non-financial benefits as a method to motivate employees had its origin in the 1980s. In simple sense it is the use of awards, rewards, and other benefits for both employees and their families. All rewards do not necessarily improve an individuals financial standing. Non-financial rewards are those desirable things that are provided by an organization. They make an employees life on the job more attractive. For example, benefits like preferred office furnishing, preferred lunch hours, assigned parking spaces, preferred work assignments, business cards, secretary, impressive cities, etc.

Question 11. Explain the need and usefulness of wage survey? Answer: Pay and benefit survey consists of information on pay namely the market rates and employee benefits structure in other organizations. It enables to position the pay and benefits within the organization in tandem with its policies on reward comparabilities. The pay survey facilitates in the construction and maintenance of a competitive pay which is market compared, because of which you develop a strong base for attracting and retaining talented people, decide levels of pay for individual jobs and scales of pay structure grades, provide guidance on internal differentials vis-avis external differentials, information and data on any adjustment required to general or individual pay levels through pay reviews. Surveys may involve the national, regional or local markets or all the three. The national and regional labor markets are scanned mostly for the professional, managerial and technical staff and the local market throwing light on the clerical, highly skilled, skilled, semi skilled or unskilled blue-collar workers. Data on salary and wages can be compiled under the following headings - basic, variable pay, total earnings, benefits, other allowances, total remuneration, pay scales, pay movements, pay increases. The said data could be collated from sources such as general, national and regional published surveys, management consultant databases, industrial and occupational surveys job advertisements and market intelligence.

Question 12. Why talent and its retention have assumed profound importance in todays firms? Answer: Talent is one of the evergreen qualities of mankind distinguishing between two persons on the basis of an unique superiority. It has always been respected. In spite of their worth till around the 1990s talent was not generally assertive and was maintaining a low to moderate profile. After the march of IT subsequent to the 1980s, organizations found that it no more required unskilled or semi skilled jobs. Along with this came the dominance of knowledge or skilled workers in the organizational activity towards attaining its objective. This kind of knowledge and skilled workers were always in short supply. Business and enterprise was also gradually transforming from national into global enabling talent to extend its orbit into the global sphere. Retaining the best talent became a priority for every organization. Among the best practices available to retain talent is to create a learning environment, mapping the career path indicating a clear direction, flattening the levels, design benefits whose value increases the longer an employee stays and anticipate, identify, and attend to employees personal requirements.

25

Human Resource Management

TEAMS
Question 1. Answer: What is the meaning of a self-managed team? Self-managed teams, also known as self-directed, self-regulating, or high performance work teams, are work designs consisting of members performing interrelated tasks. Selfmanaged teams typically are responsible for a complete product or service, or a major part of larger production process. They control members task behaviors and make decisions about task assignments and work methods. In many cases the team sets its own production goals within broader organizational limits and may be responsible for support services, such as maintenance, purchasing, and quality control. Team members are expected to learn many if not all of the jobs within the teams control and frequently add on knowledge and skills rather than seniority. When pay is performance based, team, rather than individual performance, is the standard. Self-managed work teams are being implemented at a rapid pace across a range of industries and organizations. What is the underlying essence of cross-functional team in organizations? The name cross-functional itself implies functions of different fields or work areas. This task force is limited to a particular task and is temporary in nature. These teams provide a chance to the people of diverse functional areas within an organization or between organizations, to share information, develop new ideas to overcome problems, and coordinate the complex projects. Question 3. Answer: Why team and teamwork have assumed importance in todays organizations? A team is a group of interdependent people who share a common purpose, have common work methods and hold one another accountable. The most common focus of teamwork is behavior related to task performance and group process. In an effective team, task behavior and group process must be integrated with each other as well as with needs and wants of the people making up the group. Teamwork activities often begin by clarifying the teams purpose, priorities, goals, and objectives. This establishes a framework within which further work can be done. In most team-activities, groups spend some time finding ways to improve the mechanisms that structure their approach to work.

Question 2. Answer:

26

Frequently Asked Questions in Management

TRAINING
Question 1. Answer: What is the purpose of Mentoring? Mentoring is another method of transferring tacit knowledge. Many abilities and skills defy codification and they can be transferred only through observation and inculcation from people who have those abilities. Mentoring in an organization can be well established as an integral part of the corporate culture with senior managements visible, frequent, and continuing support. It helps to improve the pool of talent for management and technical jobs and helps to shape future leaders. Mentoring is an effective vehicle for moving knowledge through the organization from people who have the most experience and learning. Among the most important aspects of a good mentoring relationship is to meet on a regular basis without which good relationship cannot be established. The roles of mentor include being a counselor, coach, advisor, broker, referral agent and advocate. What is the concept of Learning organization? Who enunciated it? What do you understand my learning curve? MITs Peter Senge enunciated the concept of learning organization. Learning organization is an approach or effort with an organization to demolish outdated thoughts and create a de novo approach that encourages the top management to look at people differently and also look at the world with a new perspective. Its main thrust is on five aspects personal mastery, mental models, building shared vision, team learning and systems thinking. The learning organization possesses information and knowledge that confers a special advantage, allowing it to maneuver with intelligence and creativity. Learning curve is the capability to learn something very easily or quickly. Learning curve means how long a person learns something. The idea is that you have a line graph. The horizontal axis stands for time, and the vertical axis stands for the progress in learning, so the higher the line goes, the more the person has learned. A steep curve means that the person learned more in the least time, so imagine the graphs line going steeply upward from the zero corner. Since learning doesnt always have a steady pace, the line often appears as a curve. Question 3. Answer: What is the meaning and process of identification of training needs? Training through needs assessment diagnoses present problems and future challenges to be met through training and development. Organizations spend vast sums of money (usually as a percentage on turnover) on training and development. Before committing such huge resources, organizations will do well to assess the training needs of their employees. Organizations that implement training programme without conducting needs assessment may be making errors. Needs assessment occurs at two levels - group and individual. The areas of training needs could be knowledge, skill or attitudinal/ behavioral areas. Job-related training needs are identified at a number of stages. At recruitment stage any immediate training needs can be identified by the line managers personnel and training officers. During appraisal training needs will be identified with following-year objectives, during promotions because of additional responsibility the required training to be

Question 2. Answer:

27

Human Resource Management

identified, personal needs can be identified by the individuals or their superiors and can pass on to HR to take the necessary training. Question 4. Answer: What is T-Group or sensitivity training imply? Sensitivity training uses small numbers of trainees, usually fewer than 12 in a group. They meet with a passive trainer and gain insight into their own and others behavior. Meetings have no agenda, are held away from workplaces, and questions deal with the here and now of the group processes. The objectives of sensitivity training are to provide the participants with increased awareness of their own behavior and how others perceive them - greater sensitivity to the behavior of others, and increased understanding of group processes. Specific results sought include increased ability to empathies with others improved listening skills, greater openness, increased tolerance of individual differences and increased conflict resolution skills. The limitation about this training is that it has only temporary effect on the participants. Why is the ROI on Training has been receiving increasing importance? Top executives, who have watched their training budgets grow without the appropriate accountability measures, have became frustrated and in an attempt to respond to the situation have demanded a return on investment (ROI) for training programs. Measuring the ROI of training programs is becoming a global issue because executives from around the world are concerned about accountability. Whether the economy is mature or developing, the economic pressures of running a global enterprise make the accountability of training an issue. Through the ROI exercise an organization reviews the improvement in critical areas of organizational activity or failings in which training was provided. The ROI approach also creates an in-built compulsion in employees and their superiors sent for training to show improvement in their work areas. In the context of large-scale pressure on organizations for its survival due to cutthroat competition, customer assertiveness and globalization ever-possible area of organizational activity and cost is microscopically examined. To put it straight organizations are assessing what is the return if any kind of investment is made. This also applies to the aspect of training. After all, training in simple terms is giving new inputs on any of organizational or individual gap in the areas of knowledge, skill and attitude, which is, impeding the organization in attaining its objective. Unlike in the past, where training was considered as a paid holiday, organizations today are seeking feedback from participants about the usefulness of the training programme, the written down plan of the employee applying the gains of the training. Subsequently at periodical intervals analysis is made in terms of measuring and establishing the gains of training like percentage increase in productivity, percentage decrease in rejection rate, days taken to implement new technology, percentage increase in absenteeism, reduction in overtime spending. Hence training is assuming the contours of seriousness affair in organizations and no more a paid holiday.

Question 5. Answer:

Question 6.

What should be the constituent of training cost as a percentage of total employee cost?

28

Frequently Asked Questions in Management

Answer:

In a relative and healthy scenario training cost as a percentage of employees varies between 6% to 15%. This has been a quantum increase in training sphere as in the traditional or orthodox approach there was even no provision for training budget. In todays scenario because of constant changes taking place in the forefront of technology organizations are left with no choice but to make training an on-going activity. At the beginning of the year when the enterprise budget is prepared increasingly the allocation towards training are on the increase. This is in quite contrast to years of neglect training received with many organizations not caring to apportion any allocation for training budget. In fact in some of the organizations it was considered to be a wasteful expenditure. What is the meaning of mid-career crisis? How should organizations help employees to overcome it? The meaning of mid-career crisis is more dominant in vertical organizations where it is specialization, which is still widespread. In the aftermath of information technology making long strides decimating many manual activities and the quality revolution seeking technology to achieve more precision and perfectness in the job, organizations are trying to shed entire levels to flatter hierarchies. From the individual perspective, those whose career moved smoothly for more than a decade on a single skill were suddenly faced with the need to be multi-skilled. The pressure was to individual managers/ employees to turn into a generalist from the role of specialist. From the behavioral perspective many supervisors and managers with years of experience who were comfortable in a top down approach and power were suddenly expected by organizations to become customer oriented, portray teamsmanship, leadership and training skills. Consequently those who were unable to cope with the requirements of multiskilling and behavioral changes inevitably landed into a mid-career crisis. Companies retrenched such employees, in the first place, to make room for structural changes. Organizations after retaining the desirable employees helped them through retraining in multiple skills, teamsmanship, entrepreneurship, leadership as trainers and thorough orientation to new technology. What does the following mean Retraining, Back To School or Employability in todays context? Employability in todays scenario means individual employees with or without the assistance of their organization keeping their skill level constantly updated and acquiring new skills to ensure their performance. Updated employees in the areas of knowledge, skill and attitude are the backbone of a company. For example, middle managers if retrained are the biggest assets. Back to school or retraining means enabling potential and performing employees to acquire multiple skills. This is possible by training employees in new or needed areas through in-house training programme or external ones under which managers are sent for training at reputed business schools or established reputed trainers.

Question 7. Answer:

Question 8. Answer:

29

Human Resource Management

INDUSTRIAL RELATIONS / EMPLOYEE DISCIPLINE


Question 1. Answer: What does empowerment mean? Empowerment is a process of enhancing feelings of selfeffectiveness among employees. By identifying conditions that foster among them a feeling of powerlessness the organization ensures their removal by both formal organizational practices and informal techniques of providing effective information. Empowerment makes the employees motivated and committed and makes employees to aspire to do a better job because they get personally rewarded. Use of participative management, implementing merit-pay systems and job enrichments are examples of possible empowerment practices. It demands teamwork, information sharing, and tolerance. Poor communication, centralized decision-making, authoritarian leadership, low incentive/rewards, lack of variety in job do not help in moving the organization towards empowerment. Do you think that the Industrial Relations era with the assertive union and collective bargaining is a thing of the past? With liberalization and market economy model making rapid strides it is expected that the days of assertive unionism will give place to that of responsible unionism. With competition and an assertive customer breathing on the neck, collective bargaining will have to play second fiddle to the healthy survival of enterprise. At the same time the managements have also to play their corporate responsibility towards quality of work, life, welfare, career and reward aspects of employees in the most professional and transparent sense. The realities of industrial relations and collective bargaining are here to stay, albeit in a more mature and responsible form. Do you think Unions have lost their clout to the ravages of market economy? The market economy imposes upon the organization, adherence to certain core aspects. They could be stated as providing of products of best quality at the lowest possible price. This is possible only by focusing on productivity, a commitment to quality and shunning of conflict mentality. This has generated in both the management and union mutuality of approach to organizational issues and to deal with them in a much more open and transparent way. It will also be of interest to note that the blue-collar worker of the organized sector have made the transition from that of a blue collar to a middle class. Also the forward march of IT have gradually eliminated the jobs coming under the unskilled and semi skilled category which were once upon a time a strong bastion and fertile ground for unionism. Consequently knowledgeable and skilled workmen who now populate organizational arena have increased their individual clout and bargaining strength. All the above have contributed substantially to the unions playing a lower profile. Why is the Government in India moving slowly and cautiously in the labor sector reform? Given the situation in India is it right to go the whole way in labor sector reforms?

Question 2. Answer:

Question 3. Answer:

Question 4.

30

Frequently Asked Questions in Management

Answer:

The elected governments in India are moving slowly and cautiously in the labor sector reform due to genuine socio-economic reasons and populist & political compulsions. Unlike the developed economies, in India still there is a substantial segment of the population living below poverty levels .The process of the employer class moving into a professional approach in dealing with employee issues is still gradual. The employment market has never been consistently buoyant and the social security movement in the country is still not widespread. The realization of employability and skill acquisition as a crucial strength is yet to be realized by many including the educated. That going into labor reforms is a must has been realized by the government of the day. At the same time the pace is realistic realizing the need to avoid the specter of any social upheaval and backlash. In the march towards globalization there is hardly any option left for countries to merge and align with it. Hence every successive government of any political hue will find it difficult to avoid labor reforms. What is the essence of the Hot Stove rule to discipline? A popular and effective disciplinary rule used today by many managers is known as the Hot Stove Rule of Discipline. This rule can be defined as the concept that disciplinary action should have the same characteristics as the penalty a person receives from touching a hot stove; that is, the discipline should be immediate, consistent, and impersonal. Before any disciplinary action can be implemented, a manager must first give advanced warning. Employees must be informed clearly that certain actions will result in disciplinary actions. This is a very important step. It is not the employees fault if he is not informed of the companys rules, ethics and standards. It is managements responsibility to educate and inform all employees. It is easier to accept discipline if the rules and standards are clearly stated beforehand and understood by the employees themselves. The importance of the Hot Stove Rule is that there is immediate action taken for a negative or wrong decision made by the employee. In order for this to be productive, management must take immediate disciplinary action. The sooner the discipline is imposed, the closer it is connected to the violation. Clarify the difference between positive and negative reinforcements? The definition of reinforcement and punishment depends upon whether an event is presented or removed after a response is made, and whether the subjects responding increases or decreases. Any event that increases responding is called reinforcement and any event that decreases responding is called punishment; any event that is presented is called positive and any event that is removed is called negative. What is the usefulness of having a formal grievance process? A grievance can be understood as an employees dissatisfaction relating to his or her employment relationship a grievance procedure is another method of resolving dispute. Formally with in an organization it means that the process how to go about in case an employee has grievance is clearly laid down and communicated to all. Grievance must be addressed promptly. Procedures and forms airing grievances must be easy to utilize and well understood by employees and their supervisors. Ego clashes should not be allowed to impede the resolution of disputes. Occurrences of similar grievances must be avoided.

Question 5. Answer:

Question 6. Answer:

Question 7. Answer:

EMPLOYEE LEGISLATION
Question 1. What does corporate governance mean to you as an HR professional?

31

Human Resource Management

Answer:

Corporate governance is about promoting corporate fairness, transparency and accountability. Corporate governance is the system by which business corporations are directed and controlled. The corporate governance structure specifies the distribution of rights and responsibilities among different participants in the corporation, such as, the board, managers, shareholders and other stakeholders, and spells out the rules and procedures for making decisions on corporate affairs. By doing this, it also provides the structure through which the company objectives are set, and the means of attaining those objectives and monitoring performance. Good governance and effectiveness of the governing body are critical to an organizations success What is the purpose of Industrial Disputes Act, 1948? Under lengthening shadow of liberalization, which are the areas, which need amendment? The Industrial Disputes Act has played a very dominant role over the years in the turbulent history of employer and employee relations. Its main purpose has been to investigate and settle industrial disputes enabling to secure industrial peace resulting in more production and improve national economy. Further its aim is to ensure fair terms to workmen and to prevent disputes between the employers and the employees so that production may not be adversely affected in the larger interest of the public. It is also a legislation protecting laborers, promoting their contentment and regulate/control situations of crisis and tension where production may be affected by irrational strikes and lock outs. Over the last half century the Act has restricted the employers ability to rationalize manpower. Employers found that recruitment was easy but ensuring exit of employees by complying to the stipulations under the Industrial Disputes Act was too difficult. Under the old economy this Act was more slanted towards workmen. But, under the onward march of market economy and globalization there has been persistent clamor from employers to give them empowerment in the form of exit policy. Gradually the government is trying to bring in pragmatic reforms to labor legislation especially in manpower exit areas.

Question 2. Answer:

Question 3. Answer:

What is the purpose of Industrial Employment Standing Orders Act, 1946.Explain its coverage and applicability? The aim of this Act is to have uniform standing orders with regard to the different conditions of service namely defining the type of workmen, working hours, weekly holidays, shift working, leave facilities and procedure, nature and types of misconduct, different kinds of punishment, employer obligations in awarding punishment, disciplinary procedure, grievance procedure, methods and procedure of payment of wages, rules and procedures with regard to strikes and lockout etc. The Act applies to all organizations, which have a total of more than 100 workmen (in some states it is applicable to organizations which have 50 or more workmen). Within 6 months from the date on which the organization reaches an employee strength of 50 or 100 workmen the employer shall submit to the Certifying Officer five copies of the draft standing orders proposed by him for adoption in his organization. This Act has taken away the freedom of the employer to unilaterally fix the service conditions of his workmen. Prior to the enactment of the Act the employer was free to fix the service conditions of his workmen according to his own will.

32

Frequently Asked Questions in Management

Certified Standing Orders framed in accordance with the Industrial Employment (Standing Orders) Act have the force of law and hence are binding on the employer and the workmen. If any employer does not submit draft Standing Orders as required or makes any change in the Standing Orders without following the proper procedure he would be punished with fine. If any employer does any act in contravention of the Certified Standing Orders, he would be punished. Question 4. What is the purpose of Shops and Commercial Establishment Act? If you were the HR Manager of a call center or an ITES enterprise would you give the right importance to its compliance? The purpose is to regulate conditions of work and employment in shops and commercial establishments. Though banking companies come under the definition of `commercial establishments' for the purpose of the Act, by separate notifications banks have been exempted either entirely from the provisions of the Act or from certain provisions of the Act like opening and closing hours, hours of work, weekly holidays, etc. This Act is a social piece of legislation enacted to prevent laborers of un-organized sector and to regulate the conditious of work and employment and therefore to secure maximum benefits to the employees working in different categories of establishment viz. Shops, Commercial Establishments, residential hotels, restaurants, eating houses, theatres and other places of public amusement or entertainments. If an organization in the ITES segment qualifies under the stipulation of the Act of course he should ensure compliance. As ITES organizations work on 24X7 basis and have substantial number of female employees it needs to work by complying with many stipulations including the working hours in the night Question 5. Answer: What is the extent of applicability of Payment of Bonus Act, 1965? Is it more relevant in the context of blue-collar workers? The Payment of Bonus Act, 1965, gives to the employees a statutory right to share in the profits of his employer. Prior to the enactment of the Act some employees used to get bonus but that was so if their employers were pleased to pay the same. The Act enables the employees to get a minimum bonus equivalent to one months salary or wages (8.33% of annual earnings) whether the employer makes any profit or not. But the Act put a ceiling on the bonus and the maximum bonus payable under the Act is equivalent to about 2.50 months salary or wage (20% of annual earnings). The object of the Act is to maintain peace and harmony between labor and capital by allowing the employees to share the prosperity of the establishment reflected by the profits earned by the contributions made by capital, management and labor. After the stipulated period of exemption or infancy period the Act is applicable to a) every factory b) every other establishment employing 20 or more persons. Every employee who is drawing a salary or wage up to Rs 3,500 per month and who has worked for minimum period of 30 days in a year is entitled to be paid bonus.

Answer:

33

Human Resource Management

ORGANISATION TYPES
Question 1. Answer: What is a PCMM organization? The People Capability and Maturity Model is an organizational change model. It is designed on the premise that improved workforce practices will not survive unless an organizations behavior changes to support them. The People CMM provides a roadmap for transforming an organization by steadily improving its workforce practices. As do all Capability Maturity Models, the People CMM consists of five maturity levels, or evolutionary stages, through which an organizations workforce practices and processes evolve. At each maturity level, a new system of practices is added to those implemented at earlier levels. Each overlay of practices raises the level of sophistication through which the organization develops its workforce. Within this environment individuals experience greater opportunity to develop their career potential and are more motivated to align their performance with the objectives of the organization. What is Flatter Organization and how does it differ from vertical organization? Flat organizations consist of wide spans of control and are conducive to better morale and managerial effectiveness. Flat structure is a development to the times in which we live as it is an answer to the growing reality that the superior cannot keep close control over several subordinates at the same time. Consequently the need for delegation arose. Delegation climate encourages individual initiative and self-control, which is more effective than control imposed. The behavioral advantage of flat organization is based on the assumption that people are governed by self-direction and self-control, and no external controls are required to mobilize their efforts towards the organizational goal. Flat organization relates to the kind of organization as envisioned under McGregors theory y. Flat structures provide a simplified upward and down ward communication because of the involvement of few levels producing less distortion and providing more accuracy. Flat structures incorporate the advantages of decentralization, individual responsibility, and initiative and make available valuable experience in decision-making although coordination may be difficult. Explain the meaning of Matrix and Net Worked organization? The Matrix structure results when a project structure is superimposed on a functional structure. This organizational design originally developed in the aerospace industry where changing customer demands and technological conditions caused managers to focus on lateral relationships between functions to develop a flexible and adaptable system of resources and procedures, and to achieve a series of project objectives. Matrix organizations now are used widely in manufacturing, service, nonprofit, governmental, and professional organizations. Every matrix organization contains three unique and critical roles: the top manager, who heads and balances the dual chains of command; the matrix bosses (functional, product, or area), who share subordinates; and the two-boss managers, who report to two different matrix bosses. In a network structure, business exists as independent units within the corporate framework but have links with a web of other companies. There is focus on the part of value chain, which it does best. Thus a computer company, which has its strength in

Question 2. Answer:

Question 3. Answer:

34

Frequently Asked Questions in Management

marketing, leaves manufacturing to some other firm in the industry and even tends to outsource its support functions from specialists. Companies in fast-changing markets use this structure. Question 4. Answer: What is the meaning of virtual organization? Virtual organizations are a temporary network of companies, which come together swiftly to exploit fast changing opportunities. The collaborators in this structure share costs, skills and access to international markets. They contribute to the virtual organization what they are best in. This structure is based on high-level of trust among the collaborators.

35

Human Resource Management

PERFORMANCE & POTENTIAL APPRAISAL / CAREER PLANNING / LEADERSHIP


Question 1. Answer: Why now more than any time the issue of leadership has assumed importance in organizations? Todays organizations in order to be successful and survive should be flatter, fleeter and empowered organizations. To operate this kind of organizations in the forefront are the managers who possess qualities representing the new paradigm of leadership: team builders instead of the archetype invidualistic player, facilitators instead of giving merely instruction and coaches and not the dictator. Persons with qualities of entrepreneurial leadership are spotted at the entry level and consciously developed. Winning companies are learning not to just hire leaders but to grow them. Toadys leaders are expected to anticipate conflict, resolve it, and bring people together. In todays organizations leaders are not born but groomed. Unlike the past where leadership was meaning to be at the top, today appropriate leadership quality is short listed for every tier of the company and trained a formal in-house programme to retrain people in leadership is developed by organizations. Organizations are modifying their reward system appropriately to constantly reinforce leadership behavior. Why is performance appraisal so important to organizations? Performance means contribution made by an individual employee in the accomplishment of organizational objectives. Performance can be observed and measured by a mix of quantity, quality, time and cost. The success of an organization will therefore depend on its ability to measure accurately the performance of its members. Performance appraisal means all those procedures that are used by the organization to evaluate the personality, the performance, and the potential of its group members. People do not learn unless they are given feedback on the results of their actions. For learning to take place, feedback must be provided regularly and it should register both successes and failures and should follow soon after the relevant actions. Performance appraisal comprises the following steps: - select performance factors to be evaluated mutually and set the standards to be achieved, set the performance review period, measure actual performance, compare performance with set standards and rate it with a suitable scale, communicate the rating to the appraisee. Performance appraisal identifies performers from non-performers, provides opportunity to prepare persons for higher responsibilities, identify training and development needs and identification of potential and career and succession planning. Please explain the meaning and finer aspects of Management by Objectives (MBO)? The MBO technique is one of the ways of seeking to control, coordinate, and motivate managers. It consists of six stages Corporate objectives defined at board level. Management tasks analyzed, with formal job specifications allocating responsibilities and decisions to individual managers.

Question 2. Answer:

Question 3. Answer:

36

Frequently Asked Questions in Management

Performance standards set. Specific objectives agreed and set. Individual targets harnessed with corporate objectives. Management information system established to monitor achievement against objectives. The review mechanism enables the bosses to make sure that the above is happening especially the key result areas, that are a strong feature of MBO. Above all objectives give greater freedom to both management and employees in deciding how performance is to be measured. They also have greater motivational effect since the standards are discussed and agreed upon both by the management and the employees. Question 4. Answer: What is the Balanced Score Card approach to organizational and employee performance? Balanced Score Card (BSC) is a concept helping you translate strategy into action. BSC starts from the company vision and strategies out of which critical success factors are defined. Measures are constructed that aid target-setting and performance measurement in areas critical to the strategies. Hence, Balanced Scorecard is a performance measurement system, derived from vision and strategy, and reflecting the most important aspects of the business. The Balanced Scorecard concept supports strategic planning and implementation by federating the actions of all parts of an organization around a common understanding of its goals, and by facilitating the assessment and upgrade of strategy. What is the utility of performance review and feedback? As an organized approach what should be its frequency? Performance review is the meeting in which the superior and subordinate get together to review the latters performance. Well-conducted reviews at reasonable intervals not only give feedback to the employee regarding his performance but also have an impact on the employees future level of effort, activities, results and task direction. The review not only evaluates an employees performance but also his potential for development. Effectively review takes place every day. However, as an organized approach it should be a detailed review once in 3 months. The systems approach to performance review suggests fixing of date and time in advance, venue to be secluded and free from interruptions, availability of wholesome information, ensure active involvement of the appraisee and always close a review with a firm date for the next interim review. What is the need for 360-degree appraisal? Should it be used only in employee development or used for determining the rewards? The 360-degree evaluations allow an employee to be assessed not just by his superiors, but also by his peers, subordinates and even customers. It is a systematic collection and feedback of performance data on an individual. It is done in a systematic way through questionnaire or interviews. Data are gathered and fed back to the individual participants in a transparent way designed to promote understanding, acceptance and ultimately changed behavior. This feedback system assesses managers in terms of the competence

Question 5. Answer:

Question 6. Answer:

37

Human Resource Management

they possess or more specifically through the detailed behavior, which constitutes them. So far, 360-degree concentrated on feedback at the senior level in organizations. However, as organizations are beginning to appreciate the power and value of such tools, other jobholders are now finding that their behaviors are coming under the microscope. Question 7. Answer: What is the significance of self-appraisal? Normally it is completed by the staff member prior to the manager/supervisor completing the Performance and Staff Development Plan. It provides the staff member with the opportunity to provide his/her perspective regarding performance during the appraisal period. This worksheet also prepares the staff member for a participatory discussion with the manager/supervisor about his/her performance. (This worksheet is similar to the Performance and Staff Development Plan that the manager/supervisor uses to summarize the staff member's performance). The Self-appraisal Worksheet will be included in the staff member's official personnel file in the Division of Human Resources/Records. Self-appraisal is a pro-active measure, which in a way ensures that the employee takes utmost interest in his performance and that from his side there have been appropriate professional efforts in achieving the objectives fixed for him. What is the importance of role-playing in improving employee performance? This method of training involves action and practice and a much-favored form. Participants learn by doing. It is not a drama or mere acting but is useful method to project life between learning and working on the job and creating live business situation in the classroom. Two or more participants are given parts to play before the rest of the class. In role reversal, the two players change parts. In role rotation, all the people involved in the training session take all the roles in turn. Putting one self in somebody elses shoes is always valuable, and something useful always does rub off into the real life activity. What is the difference between key performance area and key result area in respect an employee? of

Question 8. Answer:

Question 9. Answer:

Key result areas (KRAs) describe performance goals - business, functional and behavioral with defined time frames and are decided jointly by the employee and his/her manager at the beginning of the year. These KRAs are used to map the employees progress and based on the results, the company decides to plug performance gaps with the help of relevant training inputs. Key performance areas (KPAs) are the important thrust areas of job or position for which a specific manager is responsible. In fitness of things KPAs should be clearly established and communicated to the incumbent of a job positioning. In the past where there was lack of clarity prevailing on a job position, roles or KPAs. Since the last decade more organizations are progressively giving due importance to map out KPAs.

Question 10. Why is it that career planning has become a focus area in organizations?

38

Frequently Asked Questions in Management

Answer:

People perform better when they feel trusted and see meaning in what they are doing. People want to know the possibilities for their own growth and career opportunities. As managers have information about the growth plans of the organization, it is their responsibility to transmit information to their subordinates and to assist them in planning their careers within the organization. In todays organizational environment with retention becoming a focus area long-term corporate growth plans are not kept secret but are made known to the employees. Employees are helped to prepare for change whenever such change is planned. Important changes are discussed at all levels to increase employees understanding and commitment.

Question 11. What is potential appraisal and succession planning? Answer: Potential typically represents latent qualities in an individual, which manifests in concrete terms while performing various tasks/ jobs. Among the qualities, which clearly make a potential employee stand out are the following: a. Analytical power b. Creative imagination c. Detached and wholesome view d. Effective leadership e. Conceptual skills f. Technical skills g. Commercial skills h. Communication skills i. Willingness to take additional responsibilities j. Results-orientation k. Subordinate-development l. Teamwork and team-building m. Problem solving and decision-making etc. Succession planning is the business of identifying particular individuals as possible successor for specific positions and suitable posts for particular individuals in the organization. A succession plan should be much more than an organization chart showing who will succeed whom. It should in the first place set the business scheme against which future succession will take place. It should indicate the likely shape and size of the organization and the content of future management jobs. It should not be rigid and fixed at a point of time. It should be a flexible framework, which can cope with the change in business plans, unexpected resignations, and the emergence of new talents by recruitment or, for example, for an acquisition. Question 12. What is High-low matrix in finding potential? Answer: Otherwise known as the two-by-two matrix it combines the elements of performance and potential appraisal. In the matrix model, the vertical axis measures potential while the horizontal, performance. Both are further sub divided into two parameters-high and low resulting in four quadrants of classification-low performance high-potential low performance, high potential - high performance and low-potential high performance. In order of importance-I) low performance persons improve or quit, II) high potentiallow performance people are the problem children who can be converted to star performers if a location, boss or job profile is changed for them, III) High potential-high performance are the star performers and that have to be kept engaged with complex assignments all the time and groomed to take up the top positions in the company and IV) Low potential-high performance are solid citizens-high skills but limited capacity to grow beyond their current job profile-they constitute 70 to 75 % of the companys employees.

39

Human Resource Management

Attached to this is the potential appraisal system using a five point grading scale excellent, very good, good, weak and insufficient - to evaluate employees on the four broad attributes of conceptual effectiveness, operational effectiveness, interpersonal effectiveness and achievement motivation. The employee is appraised by the management development review team consisting of functional heads with whom the appraisee has close interaction in the normal course of his job. Question 13. What are the criteria applied in todays performing organizations to categorize nonperformers? Answer: The non-performers are those employees who are question marks. It means that in an increasingly organizational setting where performance measures are clearly demarcated these persons have not been able to even achieve average performance ratings. Their failure is purely due to their own knowledge, skill and attitude inadequacy. The company asks such employees to improve performance levels. If that effort fails, organizations work towards a planned separation.

40

Frequently Asked Questions in Management

MANPOWER RATIONALISATION
Question 1. Answer: Explain the meaning of the word pink-slip used commonly in todays organizations? Pink-slip euphemistically is the other word asking an employee mostly of managerial class to call it quits. The reasons for asking a managerial person to quit is more out of business reasons and state of organization health reasons rather than possibly the failure of the employee to perform. The companys financial health and market realities technology upgradation, joint ventures or business process reengineering may compel an organization to cut and reduce cost among other areas in manpower area. So companies to maneuver the sensitive aspect of exit are developing a win-win arrangement put into implementation by the top management, HR professionals and the employees at the end of the exercise. Consistency, compassion and cooperation must be the hallmarks of the successful organizational exit practice. What is the underlying meaning of downsizing? Downsizing as a positive and purposive strategy: "a set of organizational activities undertaken on the part of management of an organization and designed to improve organizational efficiency, productivity, and/or competitiveness. In simple terms downsizing is defined as a reduction in the size of the work force. This definition provides some analytical clarity, because it does not imply a value, either positive or negative, and encompasses a wide range of possible approaches. Thus defined, downsizing does not necessarily imply a reduction in the assets of the organization; for example, an organization may contract out a function that was previously done by permanent employees. The elimination of the jobs of the employees constitutes downsizing. In any event, it should be acknowledged that downsizing has altered the rule of the employment "game". The way these changes have tended to be theoretically euphemized is by indicating that the "psychological contract" between employers and employees has been violated .No longer can the employer offer job security. The "new" psychological contract being marketed is conditional employment, with the availability of training and development opportunities to help keep employees "employable", even if not at the particular company. Downsizing for the sake of cost reduction alone has been castigated intellectually as shortsighted and neglectful of what resources will be needed to increase the revenue stream of the future. Do you agree that the right to hire and fire should be given to the employer? In a protected economy and that too in a developing country like India hire and fire was a strict no-no. But with winds of liberalization blowing much stronger, globalization taking place much faster, competition, quality, customer and survival becoming crucial factors the apprehension and resistance to hire and fire are becoming muted. The suggestion that an exit policy and freedom to hire and fire be introduced in Indian companies is realistic, but needs to be done selectively. Unlike in India, in the west those who are fired are protected under various government welfare schemes that provide them with food, shelter, healthcare and elders welfare. Also these countries have a low birth rate and high education level. India does not have that kind of social security net. The government in India is trying to align with the global requirement and at the same

Question 2. Answer:

Question 3. Answer:

41

Human Resource Management

time is also likely to come out with mandatory separation packages through the support of legislation to ensure that separation if it takes place is with a reasonable financial package. Question 4. Answer: What do you understand by exit policy in the Indian context? What are the legislative limitations in India in rationalizing work force? Exit policy means an employer can exit from the business or enterprise at his wish. The reasons could be business is not doing well or reasons beyond his control. In a developing country like India with the Constitution aiming for a welfare state till now the employers have found that starting business is fine but closing it is absolutely difficult if not impossible. The labor legislation especially the Industrial Disputes Act, 1947 stipulate that the employer needs to take all kinds of prior permission before he can close business if the number of employees in his enterprise are more than 100.To top it, the judiciary in the country has given good number of verdicts which have been adverse to employers. Subsequently, India going the liberalization way from 1991 employers have been clamoring for progressive exit legislation and also the right to the employer to close business at his choice more in tandem with the kind of right enjoyed by employer in the west. Although the laws still remain stringent gradually there is a hint of reforms mood seeping in the outlook towards employers right to exit.

42

Frequently Asked Questions in Management

MODERN HR CONCEPTS
Question 1. Answer: As an HR professional what do the words Gender bias, and Sexual harassment, mean? How would you preempt the possibility of such happening? It is unlawful for an employer, employment agency, or labor organization to discriminate against any person because of race, color, sex, religion, or national origin with respect to employment practices, terms or conditions of employment, union membership or representation, and/or exclusion, applicant referral, etc. Sexual Harassment is unwanted/unwelcome sexual advances and/or requests for sexual favors, and verbal or physical conduct of a sexual nature that requires submission as a condition of employment (thats easy to understand). Or requires submission as a basis for employment decisions(tougher to digest) Or creates an intimidating, hostile or offensive environment (who interprets that one)? Gender-based conduct is unwelcome if the recipient did not initiate it and regards it as offensive. Some sexual advances, ("come here babe, and give me some of that") are so crude and blatant that the advance itself shows its unwelcome conduct. In a more typical case, however, the welcomed conduct will depend on the recipients reaction to it. One thing is certain, if someone says no, or stop, or leave me alone, what follows next could very well constitute conduct that will be considered sexual harassment. Sexual harassment is serious business! How employers react to it and take corrective action is sometimes even more serious. The courts take a very dim view of the employer who fails to recognize the problem and take appropriate corrective action when an employee complains of sexual harassment, and quite often the penalty for the employer can be equally if not more injurious to the employer as it is to the harasser. Employers are well-advised to stay informed regarding sexual harassment, and their obligations under the law as well as with public policy Question 2. Answer: What does the word Glass Ceiling implies? Glass ceiling is a prejudice or phenomenon in the world of employment where women lose out in the race for career growth into the top echelons of an organization in comparison to their male counterparts. This does not seem to be apparent on the look of it but yet a peep through statistical analysis of the women manager growing up the organizational ladder could be a clear indicator. Also, it is observed in the salary aspect that women getting less than men. In terms of symbolic parlance this kind of subtle disparity is not visible like the glass but yet that clear disparity is there. What is global HRM? Due to IT, globalization, etc., there is a need for study of global trends in HR, Workforce Diversity, macro economic and social issues that impact HRM in different countries. Further, HR practices have evolved in different countries drawing from their socioeconomic-ethnic backgrounds. A study of these aspects relating to business economics forms the backdrop for global HRM.

Question 3. Answer:

43

Human Resource Management

Question 4. Answer:

What do you understand by the coinage of the word GLOBAL WORKTEAMS? The concept of teamwork has been making deep inroads into organizational working in the last two decades. At the same time the business world has extensively moved into the global orbit. These two combined have created the concept and reality of global work force and inspired a search for the key competencies of global team members. Generally the competencies of a global team must accommodate the teams cultural makeup, mission, geographic distribution, and technological capabilities. Team members must have non-technical as well as technical skills so they can work together smoothly in conditions of rapid change. Teams at worldwide levels are made up of representatives from developmental sites and manufacturing sites. These cross- functional teams are from such areas as marketing, manufacturing, design engineering, product development, customer support, sales and distribution. The teams are trained in team building, foreign languages, valuing differences, skills in listening, conflict resolution, project planning, computer literacy etc. Global work teams should approach every issue with flexibility. Global team leaders are a must for success of global work team. Global team leaders are those who have a track record in the ability to learn and coach others. They should possess interpersonal skills and understand group processes. They must be culturally astute (transcultural competence), flexible and be able to deal with ambiguity. Global team leaders must be able to coordinate across time, distance, and culture. In some cases, they manage teams in which the members speak different languages, use different technologies, and have different beliefs about authority, time, and decision-making. In fact the team members may never have met face-to-face. Increasingly, international organizations are developing global human resources strategies and training. What is the significance of a HR Manual to an organization? The HR manual is an attempt to standardize the core HR processes in the organization. It is evolved and is an outcome of participation at various levels. This important documentation is normally coordinated by the Human Resource Department. The manual provides transparency in HR aspects of organization. Its contents are comprehensive and cover the entire gamut of HR management issues, from recruitment to retirement. It empowers managers to take decisions, ensuring uniformity in application, and toning up administration of HR. The manual should possess an in-built flexibility and dynamism which is possible through regular review, audit and reframing of the policies. Have you heard about the concept of core competency propounded by Mr. C K. Prahlad? C.K. Prahalad is a renowned corporate strategist as well as chairman and cofounder of San Diego-based Praja, which specializes in finding new ways to experience content on any Web-enabled device. He's written extensively on core competency and global strategy. A Core Competency is a special skill or technology that creates unique customer value. A companys specialized capabilities are largely embodied in the collective knowledge of its people and the organizational procedures that shape the way employees interact. Over time, investments in facilities, people, and knowledge that strengthen Core Competencies can create sustainable sources of competitive advantage. Your core competence is what youre good at. A Core Competency should

Question 5. Answer:

Question 6. Answer:

44

Frequently Asked Questions in Management

Provide significant and appreciable value to customers relative to competitor offerings; Be difficult for competitors to imitate or procure in the market, thereby creating competitive barriers to entry; Enable a company to access a wide variety of unrelated markets by combining skills and technologies across traditional business units. One of the toughest things in the life of any company is when its core competence is no longer a core process. For example, in the 1980s, big companies like HP and Motorola had Six Sigma capability in manufacturing as a core competence, and it was core, meaning it differentiated them effectively. By the 1990s, smaller companies had the same capability. Now their core competence was no longer core. Question 7. Answer: What is the fundamental meaning of outsourcing? Every enterprise in fitness of things would like to concentrate on core activities related to its line of product. Logically it means that given a choice on all other allied activities related to the product it would be prudent and worthwhile for the enterprise to get it done through outside sources. This would result in cost and time, which makes business sense. Outsourcing means down loading certain activities of the organization to out-side parties, which has been a conscious decision. This concept has gradually caught the imagination of organizations and in the last decade and more has come to hold the center stage in strategy formulation and economies of scale. This is in stark contrast to the practice of in sourcing followed extensively in the past. The success of outsourcing depends to a great extent on the ability of the organization to build a strong and reliable supply chain through creating a formidable vendor base. What is broad banding in organizational levels and categories? It was common to see organizations abound with a variety of grades and scales prior to the 1980s with Information Technology yet to make its impact. This worked well in the past or possibly was the right answer to times when organization was focused on functional expertise and vertical organizational structure. In contrast with fast changes taking place organizations are faced with the predicament of what to do with a traditional pay system. Broad banding is the compression of a hierarchy of pay grades or salary ranges into a small number (typically four or five) of wide bands. Each of the bands will therefore span the pay opportunities previously covered by several separate pay ranges. It reflects the new emphasis on horizontal processes, which is frequently a product of a business process re-engineering exercise. It enables processes originally created to sustain hierarchy and vertical movement to be replaced, support operational and role flexibility and team working. Thus it helps in providing a pay structure, which will fit delayered organizational structure. Broad banding facilitates lateral career moves and internal mobility by developing increasing levels of competence, skill acquisition in employees resulting in career development, continuous learning, adaptability and flexibility including lateral career moves. It has led to simplification of the administrative processes

Question 8. Answer:

45

Human Resource Management

relating to wages and salary as it has reduced dependence on elaborate job evaluation systems. Question 9. Answer: What is the meaning of Flexitime, Telecommuting, Job sharing, Compressed workweeks, and Sabbaticals? What are their utilities for organizations? Flexitime is allowing the employees to have flexibility in working hours within the given organization limits. It allows employees to work a full complement of the same hours each week but modify the schedule to vary from the usual 9 to 5. For example, few employees might work from 7 a. m to 3p.m., while others arrive at noon and leave at 8 p.m. Compressed workweeks Workers on the job, work for four longer days instead of five shorter ones, leaving a day a week when they can attend to personal business and avoid the commute. The nine to five offices, complete with its archaic attendance rules, crowded cubicle spaces and pathetic parking lots are being shunned in favor of flexiwork schedules. Telecommuting is a work arrangement whereby employees are allowed to work from their own homes, instead of traveling to office everyday. This avoids the aspect of office interruptions, enables the people to work during off-hours per their convenience, offers lowered infrastructure cost, increases productivity and provides the scope for work-life balance. Job sharing is an arrangement whereby two employees divide the time and responsibilities of one full-time job. Older employees, parents or students do typical sharing. This enables a broader range of skills available for the work, more energy from both workers than either might have done, reduced absenteeism. Sabbaticals are an arrangement whereby long-term employees take off for six months or a year without pay to obtain further education or pursue other interests. Question 10. What is the meaning and significance of Knowledge Management? Answer: Till date there is no established definition of knowledge management. The nearest acceptable definition of KM is that it is the development of tools, processes, systems, structures and cultures explicitly to improve the creation, sharing, and user of knowledge critical for decision-making. Knowledge management (KM) is concerned with both tangible knowledge assets and intangible knowledge or intelligence possessed by employees and stakeholders. Not only is acquiring and storing data important, but the real key to effective KM is sharing and leveraging information and knowledge. The same can be said of human/intellectual capital. No longer just dependent on financial capital or capital equipment, todays organizations need human capital (employee experience, skills, and ideas/ creativity) to be used and leveraged for competitive advantage.

Question 11. Today most of the organizations are putting maximum emphasis on Service aspect? Why? What do you understand by a service organization? What does the statement the customer is the king, imply? Answer: In a protected economy, in a compeitionless world any thing produced is sold. The customers had few options in buying a required product. The concept of quality in

46

Frequently Asked Questions in Management

product or any other allied service was pass. In a nutshell the customer had hardly any say. This was the profile of market and customer two decades back. The aspect of service was too imperfect in the lexicon of enterprises. Service in its original sense is the act of providing the best details on a product before a customer made a decision to buy, best product on buying best good feel after buying so that in case of need for the product next time he will invariably come to you. It is the feeling of delight and satisfaction. The importance of quality, the assertiveness of customer and the sight of the customer having options to choose among the same range of product made the organizations realize the importance of best service to lure and retain the customer. Service in its total sense could mean delighting the customer in whatever activity one had to undertake for him. In an organization it means that in giving excellent service to a customer the internal customers within the organization are also made a part of the quality and service loop. Without this task of producing the best to the final customer will be badly affected. In that sense the customer has become the king because he is the factor around which all the important activities of the organization in one way or other revolve. Question 12. What is the meaning and significance of employees satisfaction survey in Organizations? Answer: Employee Satisfaction Survey (ESS) is a tool by which a reasonable estimation of the employees attitude towards their jobs is elicited. Organizations drive towards its final destination of creating highest customer satisfaction is dependent on the extent of employee satisfaction. Satisfied employees perform and the only way to find out that is through a survey. Organizations in fitness of things should conduct the employee satisfaction survey ones a year. This regularity and frequency is important since such an exercise enjoys credibility only if the areas of weakness thrown upon by the previous surveys have been acted upon. If you are not willing to act, it is better not to conduct an ESS. It is imperative that the top management involvement is a must. ESSs are normally custom-designed, using about 10 dimensions along which employee satisfaction is measured. Every employee gets a standard questionnaire of around 97 questions relating to the management, quality of supervision, communication, relationships, work environment, performance, job, recognition, quality, pay and benefits. The ESS is based on the philosophy of confidentiality and anonymity. The employee is advised not to mention the name or employment number but mention the department to which he belongs. Usually, it takes a week to 10 days to complete a company-wide survey, which has a five-point rating scale: strongly agree, agree, disagree, strongly disagree and neutral. Ultimately the top three areas of weaknesses thrown up by the ESS become the basis of a yearlong HR Action plan. ESS is like taking preventive action and is also a proactive tool, which gives to the top management a view of the organization state of health.

Question 13. Do organizations need to have a system of exit interview? What is its utility? Answer: Yes organizations need to have an exit interview system. If done professionally it gives to the organizations the reasons for an employees decision to exit. In an era where many of the organizations are having their back to the walls in employee retention the exit interview provides solid insight into the reasons enabling the organizations to do the necessary course correction. The exit interview consists of an intense tte--tte with the quit employee by his departmental superior or if not by the HR Head. Also the employee leaving needs to fill up the exit form. The exit feedback is consolidated and researched

47

Human Resource Management

once in a quarter to establish the root causes of employee turnover and ensuring appropriate action plan to correct them. It has been observed that between success and failure of the exit interview system is the commitment of management to make a genuine implementation of the interview feedback and also timely implementation of the action plan on the findings. Question 14. What is Q-12 approach to individual attitudinal survey? Answer: A concept propounded by Marcus Buckingham and Curt Coffman in their famous book FIRST BREAK ALL THE RULES WHAT THE WORLDS GREATEST MANAGERS DO DIFFERENTLY. They wanted to find out what is the core within an organization that contributes to a workplace being considered as strong. They wanted to tell it to the world that if any organization brings all the core elements together in one place then they could create the kind of workplace that can attract, focus, and keep the most talented employees. According to them measuring the strength of a workplace can be simplified to 12 questions. They are: Do I know what is expected of me at work? Do I have the materials and equipment I need to do my work right? At work, do I have the opportunity to do what I do best every day? In the last seven days, have I received the recognition or praise for good work? Does my supervisor, or someone at work, seem to care about me as a person? Is there someone at work who encourages my development? At work, do my opinions seem to count? Does the mission/purpose of my company make me feel like my work is important? Are my co-workers committed to doing quality work/ Do I have a best friend at work? In the last six months, have I talked with someone about my progress? At work, have I had opportunities to learn and grow? These 12 questions are the simplest and most accurate way to measure the strength of a workplace. If an organization can create the kind of environment where employees answer positively to all twelve questions, then one will have built a great place to work. Question 15. Why has communication assumed lot of importance in to days organizations? Answer: Effective communication is essential for achieving organizational goals but remains one of the major problems facing modern management. Communication is significant in achieving managerial and organizational effectiveness. Today in the IT industry this communication process has gained a prominent place. The use of information technology to the communication process in organizations is known as management information systems or MIS. It keeps members of the organization informed about the internal and external happenings relevant to a task and of interest to the organization.

48

Frequently Asked Questions in Management

Question 16. Explain some of the modern approaches to employee communication? Answer: In a world where Information Technology has made long strides, organizations where practices are required to be transparent and in era where the clamor from employees is for more empowerment and involvement organizations had no choice but to give more attention on communication. The innovative forms of communication tool used in organizations are-Company intranet, town hall meetings, news letters, management by walking around by the CEOs, question answer sessions, e mail, employee satisfaction survey, audience with the CEO on a scheduled basis etc.

Question 17. Why lot of focus is given to organizational culture and culture-building in todays organizations? Answer: Organizational culture has been accepted as an important dimension for the study and understanding of organization behavior. Since the 1990s corporate culture is recognized as pervasive, significant to corporate success, and difficult to change. Sometimes, an organization feels the need to change its culture for different reasons. It may need to adapt itself to the changes in the external environment. Organization culture is the pattern of basic assumptions, values, norms, and artifacts shared by organizational members. These shared meanings help members make sense out of everyday life in the organization. The meanings signal how work is to be done and evaluated, and how employees are to relate to one another and are significant to others such as customers, suppliers, and government agencies. In the same token corporate culture represents those basic assumptions, values, norms, and artifacts that have worked well enough to be passed on to succeeding generations of employees. For example, the cultures of many companies (e.g. IBM, Sony, Hewlett Packard, Tatas) are deeply rooted in the firms history. They were laid down by a strong founder and have been reinforced by top executives and corporate success into customary ways of perceiving and acting. The growing appreciation that culture can play a significant role in implementing new strategy has fueled interest in the topic, especially in those firms needing to adapt to turbulent environments. Question 18. Explain Maslows need hierarchy? Answer: Abraham Maslow drawing conclusions mainly from his clinical experience, believed that a persons motivational needs could be arranged in a hierarchical order. He suggested that once a given level of need was satisfied, it ceased to motivate. The next higher order level of need had to be activated so as to motivate an individual. Self-actualization Esteem needs Love needs Safety needs Physiological needs Question 19. Explain theory X, Y and Z?

49

Human Resource Management

Answer:

McGregor's Theory X and Theory Y : Douglas McGregor (1960) summarized two possible views of management in worker motivation. Theory X is the traditional view of direction and control. It states that the worker dislikes work and tries to avoid it. The function of management, therefore, is to force the employee to work, through coercion and threats of punishment. The worker prefers in most cases to be directed and wants to avoid responsibility. The main motivator for the worker, therefore, is money. Theory Y is the humanistic/self-actualization approach to human motivation. Sometimes called the human resources model, it states that work is natural and can be a source of satisfaction, and that when it is, the worker can be highly committed and motivated. Workers often seek responsibility and need to be more fully involved with management to become motivated. Theory Y is most likely to be used when management utilizes worker participation in organizational decisions. In their book In Search of Excellence, Peters and Waterman (1982) stated that one of the chief differences between American and Japanese management is that American managers tend to use Theory X and Japanese managers tend to use Theory Y. This difference may be lessening, as evidenced by the practices of the management of the General Motors Saturn plants. In his book Theory Z, William Ouchi (1981) described the characteristics of the Japanese companies that produce high employee commitment, motivation, and productivity. Many Japanese employees are guaranteed a position for life, increasing their loyalty to the company. Careful evaluation occurs over a period of time, and the responsibility for success or failure is shared among employees and management. Most employees do not specialize in one skill area, but work at several different tasks, learning more about the company as they develop. And Japanese companies are often concerned about all aspects of their employees' lives, on and off the job. According to Ouchi, Type Z organizations tend to have stable employment, high productivity, and high employee morale and satisfaction.

Question 20. What according to you is the scope and coverage of HR in todays context? Answer: The scope of HR has improved and increased in leaps and bounds in tandem with the change in the underlying reasons for success of an enterprise. Todays organizations are giving focus on customer, quality, competition and price for which one needs to have talented, motivated, performing and value adding employees in a networked computerized environment. The nomenclature unskilled, semi skilled and nonperforming employees are a thing of the past in the organizational mosaic of today. With competency-based recruitment, planned induction, developmental learning, transparent career growth, exit research, employee satisfaction, pay for performance holding the center stage the dimensions HR have undergone a sea change.

Question 21. What is the meaning of the now frequently used word PARADIGM SHIFT? Answer: Paradigm means a broad model, a framework, a way of thinking, or a scheme for understanding reality. The impact of internationalization, information technology, diversity, ethics and composition of manpower consisting of traditional baby boomers, emancipated genXers and techies raised on computers has led to a paradigm shift. This means todays and tomorrows organizations and management are to adapt to new rules with different boundaries requiring new and different behavior within the

50

Frequently Asked Questions in Management

organization limits for them to achieve success. Paradigm shift implies a quantum shift in not just learning, but also how we learn; not just doing things differently, but questioning whether we should be doing many of the things which are otherwise termed as time tested Question 22. What is the difference between Personnel Management and Human Resource Management? Answer: Personnel Management represented a phase in the growth of the profession when its role was as a mere caterer of manpower, taking care of the routine and maintenance aspect of manpower, involve in fire fighting, comply with welfare objectives more of a legislative compulsion rather with any concern orientation and manage the industrial relations. This was a phase in which both the internal and external customers were not assertive and the gains of computer revolution had not made much of an impact. In contrast the Human Resources Management phase that started around the 1990s signifies a substantial change in the profile of an employee. He was found to be the most crucial element for organizations prosperity and growth. Consequently quality recruitment, management of his performance, training and development, appropriate reward for recognition and retention were the instruments used under the movement termed as human resources management, which has now come to stay.

Question 23. What is the difference between the maintenance role of HR and the present day expectation of HR manager being a business strategist? Answer: The maintenance and business strategist flavor to HR role are typical responses to prevailing times and the value given to human resources by organizations. In a suppliers market where it was the entrepreneur who had all the advantages the organizations priority on human resources always received secondary importance. It was also a time when socialism was having its influence across the world .The governments of the day expected a welfare orientation from employers towards employees and framed many welfare, safety and health legislation. The Trade Union movement was also active and the aspect of conflict between management and labor was always in the air. Consequently it can be said that till around mid 1980s the role of HR was more of maintenance and firefighting in nature. Beyond mid 1980s with IT making rapid strides and customer, competition, quality, talent and retention holding center stage in organizations growth it was inevitable that managements gave their attention to HR. The human resources angle in framing and building organizational strategies to attain business goals gained ground. Question 24. What according to you are the new areas of HR? Answer: Some of the emerging areas of HR are: Employee Learning, Leadership Development, Compensation and Benefits Management, Managing Mergers and Acquisitions, Global Resource Mobility, Global Workforce Diversity, HRIS/ Employee Research etc.,

Question 25. What type of job-assignments do you wish to do on joining our company? Answer: Some of the assignments could be as follows

51

Human Resource Management

Study of reasons for attrition; organizing employee satisfaction survey, compensation survey, helping in Budget, study on HRIS, competency mapping etc., (Here the candidate should know some depth of what he/she opts to say) Question 26. What do you think are the current trends and issues in HR? Answer: Trend is towards outsourcing of services in HR, HR management for Call centers, Employee Retention, Right Sizing, Utilizing early retired employees, etc.,

Question 27. What do you think is the future of HRM? Answer: Human Resource Management is now considered as a distinct strategic function for achieving excellence in business. The future of HRM is bright It also gets more involved with business to become strategic partner for business; hence changes in nature of business will mean constant realignment of HR practices and functions. This makes the field of HR important and interesting.

Question 28. What is the meaning of Human Resources Information System? Why is that many organizations are trying to switch over to HRIS? Answer: Human Resources Information System (HRIS) is a systematic way of storing data and information for each individual employee to aid planning, decision-making, and submitting of returns and reports to the external agencies. Information is needed for internal control, feedback and corrective actions and for statutory compliances. In the pre-IT era HR departments used to share hardware and files with other departments. Later, companies began to develop information systems devoted exclusively to human resources applications. These systems came to be known as Human Resources Information Systems. A HRIS system should be designed around a database consisting of employees and their position records. The access to the database could be in batch mode or through on line transactions with ad hoc enquiries .A number of reports such as age analysis, absence reports, salary review reports and employee profiles could be used to facilitate the functions such as personnel and salary administration, leave and absence recording, skill inventory, medical history, accident monitoring, performance appraisal, training and development, manpower planning, recruitment, career planning, collective bargaining.

Question 29. What is the meaning of ERP? Why should HR professional be aware of ERP? Answer: Organizations today face the twin challenges of globalization and shortened product life cycle. Globalization has led to unprecedented levels of competition. To face such competition, successful corporations should follow the best practices in the industry. Shortened life cycles call for continuous design improvement, manufacturing flexibility, super-efficient logistics control and better management of the entire supply chain. All these need faster access to accurate information both inside the organization and from the entire supply chain outside. The organizational units such as finance, marketing, production and HRD need to operate with a very high level of integration without losing flexibility. Enterprise Resource Planning (ERP) is an extremely powerful tool, which provides a seamless information system to support the various functional business modules of an enterprise. ERP packages if chosen correctly, implemented judiciously

52

Frequently Asked Questions in Management

and used efficiently will raise the productivity and profits of the companies dramatically. At the same time many companies fail in this regard because of their inability to choose the right product, incompetent and haphazard implementation, and inefficient usage. Question 30. What is the meaning of ISO 9000 and ISO 14000? Why should HR professional be aware of ISO? Answer: The ISO 9000 a set of five universal quality systems standards - provides a uniform framework for quality assurance that can be used worldwide. In effect, ISO 9000 has codified into a single system the essential principles common to diverse quality management and assurance standards. The International Organization for Standardization, Geneva, Switzerland, developed ISO 9000. It strives to attain drive towards unification by guaranteeing cross-border quality among the 12 members of the European Community through a third party audit system. With a third party audit, an organization implements a quality management system to satisfy one of the three ISO 9000 contractual standards. ISO 9001 is the most extensive standards pertaining to marketing, design, production and testing, ISO 9002 pertains to testing and inspection, ISO 9003 and the narrowest and least used standard. The company calls in an accredited third-party registrar or assessor to audit the quality system and certify that it meets ISO 9000 requirements. The companys quality system is then registered. The term ISO 9000:2000 standards have been reviewed and revised as part of an on-going process to keep up with changes, responses and the needs of individual companies, trying to be flexible enough to introduce it to more industries, which create a uniform Quality Management System. ISO 14000 is the environment management system. It has similar sections. It covers documentation control, training, management, record keeping etc only. It covers only environment arena. It challenges a company to decide for itself what its major environmental impacts are going to be and decide on how it plans to address and document those impacts.

Question 31. What is the meaning of TQM and how is useful to an Organization? Answer: Total Quality management is a business philosophy with worldwide appeal. TQM interested companies want to increase their employees quality awareness and change their attitude. The basic tenets of TQM are to gain competitive advantage by reducing costs by eliminating non-conformance cost and to maximize the quality of products and services. To put it precisely TQM is an integrated organizational approach in delighting customers (both internal and external) by meeting their expectations on a continuous basis through everyone involved with the organization working on continuous improvement in all activities or actions along with proper problem solving methodology. In a companys vision and journey towards world class, TQM is a must which means four important things - Total employee involvement, Just in time, Continuous improvement and Total quality control. In aiming for TQM you have to sell the philosophy to gain the commitment to satisfying customers. The objective is to introduce all employees to the

53

Human Resource Management

benefits of TQM philosophy, which emphasizes customer needs toward over output. It is to encourage employees to work as teams, to improve the quality of products and services, and to try to get it right the first time. Question 32. Have you heard about Malcolm Balridge Award for Excellence? Answer: The Malcolm Baldrige National Quality Award is one of three global quality awards coveted by companies all over the world. While the award is largely US-centric, the basic parameters for selection of companies have been adapted by several organizations across the globe to evaluate quality. Malcolm Baldrige was Secretary of Commerce from 1981 until his death in a road accident in July 1987. Baldrige was a proponent of quality management as a key to this countrys prosperity and long-term strength. He took a personal interest in the quality improvement Act that was eventually named after him and helped draft one of the early versions. In recognition of his contributions, Congress named the award in his honor. The Baldrige performance excellence criteria are a framework that any organization can use to improve overall performance. Seven categories make up the award criteria: Leadership Examines how senior executives guide the organization and how the organization addresses its responsibilities to the public and practices good citizenship. Strategic planning Examines how the organization sets strategic directions and how it determines key action plans. Customer and market focus Examines how the organization determines requirements and expectations of customers and markets.

Information and analysis Examines the management, effective use, and analysis of data and information to support key organization processes and the organizations performance management system. Human resource focus Examines how the organization enables its workforce to develop its full potential and how the workforce is aligned with the organizations objectives. Process management Examines aspects of how key production/delivery and support processes are designed, managed, and improved. Business results Examines the organizations performance and improvement in its key business areas: customer satisfaction, financial and marketplace performance, human resources,

54

Frequently Asked Questions in Management

supplier and partner performance, and operational performance. The category also examines how the organization performs relative to competitors. For many organizations, using the criteria results in better employee relations, higher productivity, greater customer satisfaction, increased market share, and improved profitability. According to a report by the Conference Board, a business membership organization, A majority of large US firms have used the criteria of the Malcolm Baldrige National Quality Award for self-improvement, and the evidence suggests a longterm link between use of the Baldrige criteria and improved business performance. Question 33. What do you understand about the commonly used word Change Management? Who is a change agent? Answer: Organizations must adapt to increasingly complex and uncertain technological, economic, political and cultural changes. The awareness on the part of the organization on the need to change and then putting in place the machinery and process to achieve the change is change management. The creation of a change management approach to facilitate an organization to create effective responses to these changes and, in many cases proactively influence the strategic direction of the firm is a must. After all organizations are in the midst of unprecedented uncertainty and chaos and nothing short of a management revolution will save them. Three major changes are shaping change in organizations: globalization, information technology and managerial innovation. Change agent is the term for anybody whose role in some management development program, or possibly in some wider corporate context, is to facilitate change. In many of the cases it is the outside consultant who is a change agent till the change is put in place subsequent to which it is an internal person who takes over. In recent times it is argued and expected that every good manager must be a change agent. Question 34. What are Paretos 80:20 principles? Answer: The famous 80:20 rule, set out by an Italian economist long ago, is basic to better management. Pareto observed that in any series, a small proportion accounted for a large number of outcome; hence the laws description the significant few and the insignificant many. Translating that into business terms, 80% of a firms profits will come from 20% of its products and 80% of its sales from 20% of its customers. Analyzing a business in this light shows the management where to concentrate its efforts. Concentrate on tight control of the significant 20% and you will achieve vastly greater savings than from effort wasted on insignificant 80%.

Question 35. What is the meaning of Brainstorming? Answer: Under this method, a problem is posed and alternative solutions are invited from the participants. Their suggestions are then critically examined and the assessment of their abilities is made. The purpose of this method of training is to enable the executive to take maximum number of decisions within a limited period of time.

Question 36. What is Quality Circle and how is it useful to an organization?

55

Human Resource Management

Answer:

Quality Circle (QC) is a group of workers from a shop or department who meet regularly under the leadership of a foreman or section head to examine work problems that affect the quality of output, and to recommend solutions to these problems. The number of workers is usually six to eight, and membership is absolutely voluntary. The success of actual quality circle depends on the contribution that can be made by solving the particular problem. QCs have demonstrated that, in the right hands and used in the right way, they can work anywhere.

Question 37. Why is that in the last more than a decade organizations are giving lot of emphasis on total cost management including employee cost? Answer: The basic tenet of total cost management practiced by organizations of every hue is intelligent optimization. Control your cost otherwise it has the power to sound your death knell. Unlike the cocooned and protected environment of the past where cost was not detected to have a profound impact on the future of the enterprise, in a globalized, competitive and customer - driven economy best product at the lowest price is the survival mantra. Cost is no longer what you incur as a matter of course. It is what the market is willing to pay; it is what the competition allows. No more one is in a position to shift the price of his inefficiency into the price of the product. In the arena of human resources cost management means a total approach to increasing the companys output to the maximum extent with the least additional monetary inputs in its people management activities at every stage of the value chain. It also means an analytic total manpower requirement, position and competency mapping right selection and right productivity by every employee.

Question 38. What is the level of academics-industry interface in the field of HRM during your course at IBS? Answer: There is a good academics-industry relationship. Our Summer Internship Program (SIP) and Management Research Projects (MRP) are primarily intended to make the students get a first-hand feel of the workplace intricacies and issues. The summer internship program enables the students to experience the rigor of business environment and combine the concepts learnt in classroom with the real-life situations in business organizations. MRP provide students an opportunity to work while they are still studying. There are a large number of projects in the field of HRM. To get these projects, we need to have a good relationship with industry. There is however a scope for further improvement as in any field of activity.

Question 39. What do you understand by HR audit? How is it beneficial? Answer: A healthy HR function in an organization is as important as the physical and mental well being of a human body. Typically the basic reason why organizations prefer to conduct an HR audit is to get a clear judgment about the overall status of the organization and also to find out whether certain systems put in place are yielding any results. HR audit also helps companies to figure out any gaps or lapses and the reason for the same. Since

56

Frequently Asked Questions in Management

every company plans certain systems and targets, an HR audit compares the plans to actual implementation. The concept of HR audit has emerged from the practice of yearly finance and accounting audit, which is mandatory for every company, to be done by external statutory auditors. This audit serves as an examination on a sample basis of practices and systems for identifying problems and ensuring that sound accounting principles are followed. Similarly, an HR audit serves as a means through which an organization can measure the health of its human resource function. Organizations undertake HR audits for many reasons: 1 2 3 4 To ensure effective utilization of human resources. To review compliance with tons of laws and regulations. To instill a sense of confidence in the human resource department that it is wellmanaged and prepared to meet potential challenges and opportunities. To maintain or enhance the organizations reputation in a community.

An audit is a systematic process, which examines the important aspects of the function and its management, and is a means to identify strengths, weaknesses and areas where rectification may be warranted. An audit is done on sampling basis. And in sampling, not every instance or situation can be examined. An HR audit can be used by an organization for multiple purposes. Some of the more common reasons are: To identify and address HR-related problems. To seek out HR-related opportunities. To conduct due diligence for mergers and acquisitions. To support initial public offerings. How an audit is conducted is very often determined by its intended use. For instance, the type of audit used to ascertain HR practices may be significantly different from the type of audit used to support an initial public offering. Although the areas examined may be similar, the process used and the depth of inquiry will vary from the intended outcome. The audit process: The HR audit process is conducted in different phases. Each phase is designed to build upon the preceding phase so that the organization will have a very strong overview of the health of the HR function, at the conclusion of the audit. These phases include: Pre-Audit Information: This phase involves the acquiring and review of relevant HR manuals, handbooks, forms, reports and other information. Auditors forward a pre-audit information request to the client who compiles the necessary information for review. Pre-Audit Self-Assessment: In order to maximize the time spent during subsequent portions of the audit, a pre-audit self-assessment form, if sent to the client can be of use.

57

Human Resource Management

The self-administered yes/no questionnaire asks a number of questions about current HR policies and practices. The completion of this self-administered questionnaire allows auditors to identify key areas for focus during the HR audit. On-site Review: This phase involves an on-site visit at the clients facility interviewing staff regarding HR policies and practices. A very in-depth HR audit checklist is completed. Records Review: During the on-site visit, a separate review is conducted of HR records and postings. Employee personnel files are randomly examined as well as compensation, employee claims, disciplinary actions, grievances and other relevant HR related information are checked. Audit Report: The information gathered is used to develop an HR audit report. The audit report categorizes action needs into three separate areas. The areas that are urgent and important (UI), not urgent needs but important (NUI), not urgent but not important needs (NNI)), and important opportunities needs (IO). As a result of this scheme of classification, managements can prioritize their steps. The critical areas: The comprehensive HR audit covers all areas of HR management like recruitment practices, training and development, compensation and benefits, employee and union relations, health, safety and security, miscellaneous HR policies and practiceswelfare, strategic HR issues, manpower planning/budgeting. Besides classifying needs in each of the above areas, the HR audit also cites relevant laws, cases and research to support the recommendations. Preparation for an audit Auditor engagement: If external firm carrying out the audit, it is preferable to set terms in writing defining and agreeing on scope .If using internal resource it is better to appoint them formally with clarity on scope and select persons who are non political or those who are not high on hierarchy. Also, if internal persons are auditing there must be training in auditing. Documents, manuals, handbooks, forms and reports auditor must have access to relevant information contained in employee files and other confidential documents of the organization. Auditors must be given unrestricted access to records, once they sign agreement for confidentiality. Data gathering: Completion of a self-assessment questionnaire significantly expedites the audit process and allows for better audit planning. On-site access: The on-site portion of the audit is the most critical. Using audit findings: How does an organization use HR audit results? Since the HR audit results are classified, an important aspect is already taken care of. Critical needs should be the first ones to Organizations generally have three options for dealing with audit results. Use the HR audit as a blueprint or action plan for addressing HR needs.

58

Frequently Asked Questions in Management

Address as many needs as possible using the organizations internal expertise and resources. Contract out those need areas where internal expertise and resources are not available or do not fit in the core competencies of the organization.

An HR audit is much like an annual health check. It can perform the same function for the organization. An audit is a means by which an organization can measure where it currently stands and determine what it has to accomplish to improve its HR functions. It involves systematically reviewing all aspects of human resources, usually in a checklist fashion, ensuring that the government regulations and company policies are being adhered to. The key to an audit is to remember that it is a tool to discover and not to test. There will always be room for improvement in every organization.

59

Frequently Asked Questions in Management

PART - 3

MARKETING

61

Marketing

62

Frequently Asked Questions in Management

MARKETING - GENERAL
Question 1. Answer: How did the marketing concept evolve? Before industrial revolution took place, producers were concentrating on products and productivity improvement. The whole business process was inward looking. However production process focused for improving efficiency and optimum utilization of resources increasing competition, changes in consumer tastes and life styles coupled with the presence of a few similar looking products with increased volumes of production necessitated manufacturers to concentrate more on identifying and satisfying specific consumer needs. This changed the total concept of business from product orientation to market orientation and consequently from mere selling activity, the organization has started looking at the business in terms of satisfying customer needs. What are the key success factors in direct marketing? Customer Database, Targeting, Appropriate media, Appropriate distribution. Since direct marketing facilitates first hand information of customers, marketing function can be aligned more suitably to achieve marketing objectives. Question 3. Answer: How will you access customer needs? Customer needs are of different types based on physiological, psychological, social and many other factors. For need identification the marketer has to study the complete profile of the customer. Marketing research is one of the basic tools for understanding customer needs. What is the difference between customer need and want? Need is driven by the behavioral stimuli in customer. To satisfy the need customer develops wants eg., biological need is thirst. Individual may want to quench the thirst with water or soft drink. How does one segment the market demographically? Demographic segmentation on population can be done based on urban, rural population, gender, age group, income levels, per capita consumption of different items and other classifications. What is the importance of 4 Ps in marketing? Study and understanding of product, price, place and promotion are essential in making marketing mix decisions. In planning marketing strategy external market forces like industry, competition, customer profile, government policies, state of economy factors are to be aligned with internal policies on product, price, place and promotion to achieve optimum marketing results. What is the difference between Marketing and Selling? Can there be selling without marketing and vice versa? Are Modicare products, marketed or sold?

Question 2. Answer:

Question 4. Answer:

Question 5. Answer:

Question 6. Answer:

Question 7.

63

Marketing

Answer:

Peter Druckers concept of marketing is Total Business viewed from the point of customer. Concept explained by Kotler, Chartered Institute of Marketing, American Association Of Marketing emphasizes identifying customer wants and needs and satisfying them by creating value exchanges for transferring products and services for mutual benefits to customers. Selling involves exchanging goods / services and recovering value. In other words, it can be termed as a process of changing ownership. The main purpose of marketing is to make selling redundant. Modicare products are marketed as the company involves more of relation building than direct selling.

Question 8. Answer:

What do you understand by marketing mix? Are they different from products and services? Marketing Mix consists of internal factors influencing the target customer. The internal factors are product, price, place (distribution) and promotion. When matched with the external factors i.e., competition, consumers, govt.. policies and nature of the economy they result in effective marketing mix. This will enable organizations to attract customers and meet the Marketing objective. Since the nature of services marketing is different, people, process and physical evidence are added to the marketing mix.

Question 9. Answer:

What is the importance of 4 Ps in a Car Rental company? While the 4 Ps are important in devising optimum marketing mix, the other three elements of people, process and physical evidence are also required to be considered, as car rental is basically a transport service for customers.

Question 10. How do you differentiate the following pricing concepts? Image-pricing Time-pricing Location Pricing Answer: Image Pricing: 1. 2. 3. Companies price the same product at two different levels based on the image. An IBM PC is more costly than an assembled PC such as Zenith even though both of them perform at the same level. Customer pays more for the IBMs brand image. Taj Group of hotels price differently for the same kind of service at its different business units viz. luxury hotels, business hotels, economy hotels etc. In this case, for each price category, there is a little variation in the service provided. Even though the quality of the shirts manufactured by Raymonds and Van Heusen is same, but the customer pays more to Van Heusen since it has more brand image than Raymonds.

4.

64

Frequently Asked Questions in Management

5.

In general, a perfume manufacturer can put the perfume in one bottle, give it a name and image, and price it at Rs.X an ounce. The same perfume can be put in another bottle with a different name and image and price it at Rs.X+1 an ounce.

Location Pricing: 1. Companies price the same product differently at different locations based on the customer preferences, govt. regulations etc., even though the cost of offering at each location is the same. Most of the consumer goods are priced less in eastern states of India where as the price in southern states is more. Companies price the same product differently during different seasons, days or hours. Some of the white goods like air-coolers, refrigerators etc. are priced differently during different seasons of the year. During summer they tend to be more priced because of the high demand. This also the same case with woolen-wears.

2.

Time Pricing: 1. 2.

Question 11. Briefly tell us about various pricing strategies? Answer: There are two main basic pricing policies. In penetration policy, prices are at lower levels to competition to increase penetration and market coverage and to attract large customer segments to become one of the major players in the market. This can be adopted only when the competition is intense in terms of quality, price and product acceptance in market place. When the competition is weak, price is fixed at the maximum level the market can absorb to skim the profits before the competition catches up with their improvement in marketing mix offerings.

Question 12. Differentiate between a discount and an allowance? Answer: Discount 1. 2. 3. 4. 5. A discount is a price reduction offered to buyers during a specific period of time or a gift to the buyers. Some companies give cash discounts to their wholesale distributors if they pay their bills promptly. During festival seasons, most of the companies offer discount prices to their buyers in order to increase the sales by attracting more customers. Some companies also give discounts in the price when the buyers buy large volumes. Ski manufacturers offer seasonal discounts to retailers in the spring and summer to encourage early ordering hotels, motels, and airlines offer seasonal discounts in slow selling periods.

Allowances 1. Allowances are extra payments designed to gain reseller participation in special programs.

65

Marketing

2. 3.

Promotional allowances are the payments or price reductions to reward dealers for participating in advertising and sales support programs. Allowances are most common in consumer-durables.

Question 13. Differentiate between mark-up pricing and target return pricing? For a novel product which would you follow? Answer: Mark-up Pricing 1. 2. 3. Markup pricing is to add a standard markup to the products cost. Markups are generally higher on seasonal items, specialty items, slower moving items, items with high storage costs and demand-inelastic items. Drug stores are marking up the price of some drugs high. Just as a patient who visits a doctor and gets handed a Band-Aid is not billed for just Band-Aid, the customer has to pay for the pharmacists time and other expenses of maintaining a pharmacy department. Markup pricing generally ignores the current demand and perceived value. Mark up pricing works only if the marked-up price actually brings in the expected level of sales. When Philips introduced the videodisc player with a mark-up price, Philips wanted to make a profit on each player, which did not work when competitors entered with a lower price. Target-Return Pricing is the price set to yield the target rate of Return on Investment. Canon prices its copiers in order to yield the target return on investment General Motors prices its automobiles to achieve a 15 to 20 percent Return on Investment

4. 5. 6.

Target-Return Pricing 1. 2. 3.

For a novel product, mark-up pricing is followed. Question 14. Tell us what do you know about Telemarketing? Answer: Telemarketing is the marketing of products /services through telephonic interaction instead of distributing and selling through trade channels. Marketers provide complete details of the product, price, availability and sales terms to prospective customers. Since the customer is interacting directly with the organization, he will get first hand information and in the process is able to make purchase decisions for ordering on telephone and getting the product delivered at doorstep. It provides convenience in shopping in terms of time and cost in visiting the sales outlets for buying.

Question 15. What is societal marketing concept? Answer: The organizations main task is to determine the needs, wants and interests of target markets and deliver the desired satisfactions more effectively and efficiently than competitors in a way to preserve or enhance the consumers and the societys well-being.

66

Frequently Asked Questions in Management

It does not mean to defy business sense. It aims at increasing profits. But care of societal needs is also one of its targets. In other words, societal marketing also takes care of social responsibility of the organization for the benefit of the society. e.g: Johnson and Johnson setting up plants for destroying bio-degradable packing material. Question 16. How does one go about marketing products and services? Answer: Since most of the products are tangible in nature they can be communicated, distributed, stored and advertised on features and benefits and priced to match customer acceptance levels. Services on the hand cannot be easily marketed due to reasons of intangibility, perishability, heterogeneity and inseparability of customer from actual service. It is marketed taking help of the other marketing mix elements of people, process and physical evidence of service. Physical evidence provides some element of tangibility to the service in the form of good offices, equipments, ambience etc. Process of service and people facilitates fulfillment of service promises. Since people are involved in the process of service delivery, selecting and training people to interact with customers to ensure their satisfaction and the entire service process designed to match the service needs of the customer, are additional marketing functions which are essential for marketing services.

Question 17. What are the components of promotional mix? Who should be responsible for this, incharge agency or the company? Answer: Advertisements, Sales promotion, Personal selling, Public relations and Direct Marketing are the components of promotional mix. Since the companies stakes are at the highest in all decisions regarding marketing mix, the company should be responsible rather than the agency in-charge. Question 18. What is the difference between Domestic and International marketing? Answer: The way products/services are marketed in domestic and international markets are different due to customer profiles and market structures, international competition and global economy. For domestic marketing, marketers study external domestic factors to formulate marketing mix policies. In international marketing, the marketer needs to study all the factors in the global environment to match marketing mix policies. For example prices offered in the importing country by exporters from other exporting countries and foreign exchange conversions are prime factors to fix prices.

Question 19. What are the opportunities and threats that typical Indian business houses have been encountering due to the globalization of business? Answer: Opportunities i. ii. iii. iv. To enter new markets To serve more demanding customers To be exposed to global competition To generate economies of scale

67

Marketing

v. vi.

To tap into new sources of knowledge To be exposed to global best practices

Threats i. ii. iii. iv. v. Unfamiliar business and cultural environments Foreign exchange rate fluctuations Political risks Legal & Contractual risks Well entrenched competitors

Question 20. What is the documentation involved in international marketing? Answer: Shipping bill, Bill of Lading, invoice in the currency specified by customer, product quality inspection certificate from authorized agency, certificate of country of origin, customs clearance certificate, import export certified code of RBI and any other statutory documents specified by the exporting and importing country, letter of credit issued by importers bankers with openers confirmation for negotiating export bills for payment.

Question 21. How will you convince the customer that your product is better than your competitors? Answer: Showing your competitors in inferior light is bad selling and marketing. Instead it is always beneficial to stress your product features, which are different compared to competitors offering and how they benefit user better compared to competitors. In case there is very little difference with competitors products it will be necessary to highlight your superior customer service and what extra you will be giving in deliverables in service.

Question 22. What kind of marketing does Eureka Forbes follow? Answer: Eureka Forbes follows principles of Direct Marketing. They use their own trained frontend and back-end sales personnel. There are no marketing intermediaries and third party sales efforts. Front-end sales personnel directly contact institutional and domestic customers with house calls to explain product features/ benefits, satisfy customer queries. Even the advertising campaign is designed to communicate the benefits of direct purchase from the manufacturer.

Question 23. What are the key differences between a product-centric approach and a market-centric approach? Answer: Product concept stresses on 1) 2) 3) Quality Performance Innovative Feature

Product-centric focuses on creating superior products Marketing Concept Stresses on 1) Target Market

68

Frequently Asked Questions in Management

2) 3)

Fulfilling consumers, demand Communicating to consumers

Market centric focuses on customer needs satisfaction. Question 24. Differentiate among convenience, shopping, specialty and unsought goods with appropriate examples? Answer: Convenience goods are goods that the customer usually purchase frequently, immediately and with a minimum of effort. Eg. Toothpaste. Shopping goods are goods that the customer, in the process of selection and purchase, characteristically compares on such bases as suitability, quality, price, and style. Eg. Televisions. Specialty goods are goods with unique characteristics or brand identification for which a sufficient number of buyers are willing to make a special purchasing effort. Eg. Fine crystal. Unsought goods are goods that the consumers do not know about or do not normally think of buying. Eg. Red Cross blood donations.

Question 25. What are the different stages in the PLC? How do you extend the PLC of a product like Khadi Kurta? Answer: The different stages in the PLC are Introduction, Growth, Maturity and Decline. Khadi Kurta is in maturity stage of PLC. The PLC of Khadi Kurta can be extended by increasing variants, improving features, changing prices, and generating excitement around the product through promotion and schemes. Also recycle where growth starts again after maturity by applying newer uses and application of the same product. Question 26. What do you understand by STP? Assume you run a laundry business in a Mumbai suburb. Explain your STP strategy. Answer: STP means Segmentation Targeting and Positioning. These are strategic steps, the top management takes while launching a product/service. 1. A Mumbai suburb perhaps will have a certain demographic characteristic. 2. Choose the demographic segment that best suits this service, keeping in view the life-style. 3. Position the service as convenience to busy office-goers, home-makers, bachelors etc. vis--vis a washing machine.

Question 27. Define the following concepts. a) b) c) d) e) f) Answer: a) Image Position Product features Competitive advantage Positioning strategy Customers perception Image: Image is the set of beliefs, ideas, and impressions a person holds regarding product / service.

69

Marketing

b) c) d) e) f)

Position: The place in the minds of consumers, where the product is situated vis-vis the competitors. Product feature: Characteristics of the product that supplement the products basic function. Competitive advantage: Strong points in terms of customer benefits, which a company may possess with respect to its competitors. Positioning strategy: Brand building strategy in the minds of consumers vis--vis the competitors. Customers perception: How the customer is viewing the companys brands.

Question 28. What are the factors that influence the consumer-adoption process for a newly developed product? Explain the consumer-adoption process for Solar Cooker, & Home Delivered Pizza. Answer: Factors influence the consumer-adoption process i. ii. iii. iv. Personal factors: Psychology, readiness to buy new things, and confidence Economic factors: Economic risk Social factors: What others will say Product related factors: Kind of product, risk involved.

Solar cooker - Personal factors, and Economic factors Pizza - Personal factors, Social factors Question 29. Explain the characteristics of services. Explain with an example of a hair cutting service at your neighborhood. Answer: The characteristics of services are Intangibility, Inseparability, Variability and Perishability. Example- hair cutting varies from customer to customer, barber to barber Question 30. How can goods producing companies improve their product support services? Answer: Pre-sale Services: Facilitating and value augmenting services Post sale service: Customer service department, Repair, Maintenance services A support service is also a service. So services quality can be the edge over any other services like improving reliability, and assuring responsibilities. Another approach could be to identify the gaps in services and play there. E.g., services specification and delivery may be wrong. Question 31. Do you think the globalization of business made businesses in India more market oriented? Give few examples? Answer: Globalization of Indian economy has started bringing in new generation of products, latest technology and competitive prices. Indian organizations are required to reorient

70

Frequently Asked Questions in Management

their policies and processes to compete with the global products. Market orientation on providing world-class products has become an important corporate strategy. Eg: Insurance business, Automobile sector, and Consumer durables. Question 32. What do you understand by marketing of non-profit organizations? Is it necessary to market them? Answer: Non-profit organizations have goals that are normally beyond profits. Hence relationship building with client base is important. Clients have to appreciate their higher goal of a non-profit organization. Eg- CRY, Red Cross, Blue Cross and number of NGOs/Govt. services. Yes, there is a necessity to market them in order to get desired response from the targeted customers.

Question 33. What public and ethical issues do direct and online marketing raise? Answer: Irritation, Unfairness, Deception and fraud, Invasion of privacy

Question 34. Why do you think that an organizations business should be centered around the market and not around the product? How do you relate marketing myopia with this concept? Answer: Needs, wants, demands, taste & preference exist in the market. Organizations attempt to fulfill those with their products & services. The term Marketing Myopia was coined by Prof. Theodore Levitt. He explained the disadvantages of excessive preoccupation with the product or production or selling, ignoring the customer in the process. It means a colored or distorted perception of marketing and a short sightedness about business. Excessive attention to production or the product or selling aspects at the cost of the customer and his actual needs creates this myopia. Such a myopia leads to a wrong or inadequate understanding of the market and hence failure in the marketplace. It even leads to a wrong or inadequate understanding of the very nature of the business in which the organization is engaged. Question 35. In pricing the real issue is not price but value. Do you support the comment? What is the difference between price and value? Answer: Yes, I support the comment because, 1. 2. 3. 4. 5. Value is the bundle of benefits offered to the customer for which he is willing to pay a price. If the price is less than the value, customer will buy the product. Customers want to assess the total costs of acquiring, using, storing, and disposing of the product. In pricing a product, companies should be careful enough on the value that the company offers to customer with the products they buy. Price of the product should always be less than the value bundle that the customer buys. Then only he will think it feasible to buy the product

Question 36. How should a price be set on a product or service for the first time? Can you guess how GE set its bulbs price, Ford set its Model T. Price, Sony set its walkmans price?

71

Marketing

Answer:

In setting a pricing for a product for the first time, companies have to decide on the marketing objectives such as survival, maximum current profit, maximum market share, maximum market skimming, or product-quality leadership. GEs Bulb 1. Comparing with the cost, convenience of available substitutes for generating light.

Fords Model T 1. 2. Comparing with the cost of Horse & Buggy and the cost that the market will bear. For the required volumes what is the price that can be charged

Sony Walkman: 1. 2. 3. 4. New to the world No comparable product Market skimming Estimate customers delight value through research and eventually settle for market penetration pricing.

Question 37. How does the service marketing of services differ from that of marketing of goods? Which is more difficult? Why? Answer: Services differ from products in the following ways; Intangibility, inseparability, heterogeneity, and perishability. The difference in marketing of services compared to marketing of goods is the result of above. Services cannot be produced, stocked and patented. Customer is involved in service process delivery. Unlike products, which are produced for stocking and delivering, services cant be standardized all the time. Consumer involvement warrants customization. Consumer process for selecting specific service provider is complex due to very nature of service marketing. Amount of Experience and credence assume more importance in the selection process.

72

Frequently Asked Questions in Management

STRATEGY
Question 1. Answer: What is BCG Matrix and what does it show? This matrix was coined by Boston Consulting Group, a reputed management consultancy firm. This is a portfolio evaluation model, which evaluates various Strategic Business Units (SBUs) of the firm on relative market share and industry growth rate. This matrix helps managers address the issue of resource allocation between various SBUs and the business that the firm should exit/reinforce its presence. Explain Competitive Advantage, Competitive Strategies, and Generic Strategies. Which one are we following? (Hotel Industry) There are multiple sources for a firm to buildup competitive advantage. For example Sonys competitive advantage is in miniaturization, 3M in innovation etc. However the central issue is sustaining the competitive advantage. Michael E Porter has proposed three generic strategies viz., cost leadership, differentiation and focus. Motels and wayside inns generally aim for cost leadership. Other high-end players like Hilton, Marriott etc., have found innovative basis for differentiation. Question 3. Answer: What are competitive advantage and core competence of a company? How are they different? Illustrate your answer with the help of a suitable example. Competitive Advantage: Competitive edge with respect to its competitor Core competence: Competence inherent within the company The concept of competitive advantage means a superior position relative to competition. There are two questions. 1. 2. Do I perform some function in a superior/distinctive way, compared to competition? Does the superiority/ distinction mean something in terms of customer value?

Question 2. Answer:

It is essentially a position of superiority on the part of the firm in relation to its competition in any of the functions/activities performed by the firm. The functions/activities may include R&D, production, finance, marketing etc. The superiority may also cover the resource and capability dimension and factors like technology. The big winners in any industry usually possess superiority/distinction in several functions/areas. For strategies to work, a firm must possess relevant competitive advantage. For long-term success, the competitive advantages have to be durable. And a durable competitive advantage can emanate only from a strength that is unique to the firm. It thus follows that no firm can keep succeeding over the long term, without such a unique strength. This unique strength is the subject of core competence. What we are talking about is a durable and unique competence. The attributes of core competence as propounded by Prahalad and Hamel are

73

Marketing

1. 2. 3.

Core competence is a fundamental, unique and inimitable strength of the firm It is largely a technological competence It is a knowledge base Name Sony Canon 3M Honda Core competence Capability for miniaturization Unique strength in Optics, Imaging and microprocessor controls Capability for making coatings, adhesive and substrates and combining them in multiple ways Capability of making different size engines which gives it an advantage in diverse products like cars, motorcycles, lawnmowers and generators Unique strength in chemical technology

Sl.No. 1 2 3 4

5 Question 4. Answer:

Dupont

Explain Porters five forces Model with the help of the example of your business school. Five forces that determine the intrinsic long-run profit attractiveness of a market or market segments are: Potential Entrants, Suppliers, Industry Competitors, Buyers and Substitutes. Industry Competitors: IIMs, XLRI, SOM of IIT Bombay, ISB, Jamnalal Bajaj, SP Jain Narsee Monjee, Symbiosis and several other private sector players, State universities Buyers: Graduate students seeking a career in Management in India Suppliers: Faculty, Books, Software companies, Hardware companies, classroom equipment etc Potential Entrants: Educationists setting up their enterprises, private sector companies wanting to diversify into management education (example of ISB) etc Substitutes: Management education abroad in USA, UK, Australia etc

Question 5. Answer:

How are the SBUs categorized in the BCG matrix? Where will you put the manual typewriters of Godrej? Question Mark, Cash Cow, Stars and Dogs. Typewriters from Godrej can be categorized as Dogs

Question 6. Answer:

How does the Dell model of SCM work? To start with, Dell broke up its downstream operations into three separate channels for sales, service and delivery. Each channel has built-up competencies that are in many ways superior to other companies in the industry. As opposed to push strategy, Dell works on pull strategy since every link in its supply chain is linked. What are push and pull strategies?

Question 7.

74

Frequently Asked Questions in Management

Answer:

In Toyota Production System (TPS), the pull system is accomplished by KANBAN. In pull system, the following processes go to pick up what they need to replace what they have used up from preceding processes. An approach opposite to this / push system. Can you explain the following competitive strategies with suitable examples? a) b) c) d) Frontal attack Flank attack Bypass attack Guerrilla attack

Question 8.

Answer:

Frontal Attack: Frontal Attack is a game for the No. 2 or No.3 company in a field. The key principle is to find a weakness inherent in the leaders strength and attack at that point. 1. 2. 3. The main consideration is the strength of the leaders position Find a weakness in the leaders strength and attack at that point Launch the attack on as narrow a front as possible

Pepsi-Cola is winning the cola war with archrival Cola Cola. One major reason is that Coke has not been effectively utilizing its strategic advantages. Flank attack: The most innovative form of marketing warfare is flanking. Over the years, most of the biggest marketing successes have been the flanking moves. 1. 2. 3. A good flanking move must be made into an uncontested area Tactical surprise ought to be made into an important element of the plan The pursuit is as critical as the attack itself

McDonalds continues to dominate the burger business, but Burger King and Wendys have made progress using some of the classic principles of marketing warfare. Bypass Attack: This is a game for market leader only. There are three key principles to follow, the most surprising of which is the strategy of attacking yourself and not the enemy. 1. 2. 3. Only the market leader should consider playing defense or bypass The best defensive strategy is the courage to attack oneself Strong competitive moves should always be locked

Nobody plays the marketing warfare game as well as Big Blue in Computers. But even IBM can fall flat on its face when it tries to compete on a battleground it doesnt own Guerrilla attack: Most of the players in a marketing war should be guerillas. Smaller companies can be highly successful as long as they try to emulate the giants in their field. 1. 2. 3. Find a segment of the market small enough to defend No matter how successful you become, never act like the leader Be prepared to bugout at a moments notice

75

Marketing

The beer business was in the process of consolidation in 1986, from hundreds of local breweries down to a handful of national ones in US. At a time when the smaller competitors should concentrate their forces, they did just the opposite. Question 9. Answer: Please tell us the type of industries which you would like to target to market our hotel and why. One has to thoroughly understand the existing segments to answer this question since segmentation precedes target marketing.

Question 10. What are the drivers of demand of cement products in the near future? Answer: This is a case of derived demand. Demand for cement depends on user industries viz., housing activity, infrastructure projects, tax breaks for housing activities, interest rate on housing loans etc.

Question 11. Why are foreign players coming into cement industry? Answer: Size of the market, future growth potential, saturation in their domestic markets, strength of organized players compared to their global counterparts etc.

Question 12. Explain market development, product development and market penetration strategies. Answer: Market penetration strategies consider gaining more market share with its current products in their current market. Product development strategy considers whether it can develop new products of potential interest for its current markets. Market development strategies consider whether it can find or develop new markets for its current products. Market penetration strategy is fraught with less risk as it capitalizes on existing competencies. The challenges involved are (i) increasing usage rate (ii) converting nonusers into users (iii) taking away customers from the competitors. Market development and product development are the strategies of a higher degree of risk than market penetration. Question 13. If I give you a totally new product and ask you to penetrate a totally new market, how would you go about it? Answer: Understand the need the given product satisfies, identify the options customers have got to satisfy the need, identify the segment that has the greatest urge to satisfy the need, communicate to the target segment about the product, make it available at appropriate price.

Question 14. How do markets evolve, and what marketing strategies are appropriate at each stage of market evolution? Answer: Markets go through the following four phases: Introduction, Growth, Maturity, and Decline. Introduction: Major R&D emphasis, minimal growth in sales, rapid technological change in the product, operating losses and a need for resources to support a temporarily unprofitable operation.

76

Frequently Asked Questions in Management

Growth: More competitors enter in the market at this stage. Factors such as Brand recognition, product/market differentiation and financial resources to support both high marketing expenses and the effect of price competition on cash flow become important. IBM entered the personal computer market in the growth stage and was able to rapidly become the market leader with a strategy based on key strengths in brand awareness and the financial resources to support advertising Maturity: As the product/market moves through a shakeout phase and into the maturity stage, market growth continues but at a decreasing rate. The number of market segments begins to expand, while technological change in product design slows considerably. The result is more intense competition, and promotional or pricing advantages or differentiation become key internal strengths. Technological change in the process design becomes intense as the many competitors seek to provide the product in the most efficient manner. Where R&D was critical in the development stage, efficient production has now become critical to the businesss continued success in the broader market segments. Decline: Here the important factors tend to be cost advantages, superior or customer relationships, and financial control. Competitive advantage can exist at this stage, at least temporarily, if a firm serves gradually shrinking markets that competitors are choosing to leave. It is important to note that the relative importance of various functional strategies differs across stages of Product/market evolution. Question 15. Please give us an overview of the evolution of telecom market in India. Answer: For a long time state owned companies dominated this industry. Mostly they were driven by social objectives. Cross subsidization was rampant and network penetration is abysmal. Technologically the network was in a bad shape. This industry has seen sweeping changes, in the post liberalization era. Customers have more options today. This industry is also experiencing great amount of convergence, especially in the last two decades.

Question 16. What is benchmarking? Describe the steps in carrying out benchmarking in the realm of marketing. Answer: Benchmarking: - Benchmarking is the process of determining who is the very best, who sets the standard, and what that standard is. Eg: in retail banking provision of getting bank balance over telephone. Steps in bench marking: 1) 2) 4) 5) 6) 7) Determine the function to benchmark Identify the key performance variables to measure Identify the best in-class companies Measure the companies performance Specify the programs and actions to close the gap Implement and monitor results

77

Marketing

Question 17. As a marketing manager one should always be on the lookout for potential customers. How do you know who is your target customer? Answer: Out-of-box thinking is very essential for this. Marketer has to answer how his offer can satisfy the existing/modified/new need of the customer. The onus of educating the customer about the usage of his offer in different situations is on the marketer.

Question 18. What is customer delivered value? What is a value chain? Answer: Customer delivered value is the difference between total customer value and total customer cost. Total customer value is the bundle of benefits customers expect from a given product or service. Total customer cost is the bundle of costs customers expect to incur in evaluating, obtaining, using and disposing of the product or service. The customer seeks a mix of benefits. The customer seeks value. The customer has to pay a cost for acquiring this value. The cost includes the price plus other elements of cost to him, economic and non-economic. He is happy when the value exceeds the cost he incurs. The larger the value gap, the greater is his satisfaction. He compares the valuechain gaps of competing offers and selects the one that gives the best trade-off. Marketing is a value creating and value delivering process Standard Chartered Bank: The company offers global credit to all its cardholders, while most others have country specific cards. Thus, the Standard Chartered customer gets a substantial facility at no extra cost. Federal express: The company allows customers to track packages through the companys web site. This facility has enhanced the value of FedExs offer for the customer. Porter has suggested that every activity performed by a firm creates some value, which reflects finally in the firms product offer, and that these activities are linked into a chain. He calls it the firms value chain. The significance lies in the fact that he views the whole business task as a unified chain meant to deliver value to the customer. Question 19. What are the approaches to segmentation? Answer: The approaches to segmentation are: Geographic Segmentation, Demographic Segmentation, Psychographic Segmentation, Behavioral Segmentation, Multi - attribute Segmentation and Hybrid Segmentation

Question 20. What is the basic difference between differentiating and positioning? Can you illustrate your answer with the help of a suitable example? Answer: Product Differentiation refers to a strategy a company adopts to differentiate one product (or brand) different from other product (brand) in terms of features and benefits they offer to customer. Eg: HMT wrist watch offers accuracy of time. Rolex differs from HMT. It provides ego satisfaction to buyer (pride of possession) For successful marketing organization, it is imperative that their product, is different distinctive and unique.

78

Frequently Asked Questions in Management

Positioning is how the product is perceived by the Minds Eye of customers. Positioning for the product is planned based on consumer psychology. Eg: Colgate tooth paste is positioned as a tooth paste for total protection of tooth while Close up is positioned as the toothpaste for fresh breath Question 21. Gap wants to introduce Kurta Paijamas for the Indian Market? What challenges does Gap face in developing these new products? Answer: Major challenges could be: 1. 2. 3. 4. 5. Lack of Consumer / Market knowledge Low level Technology Govt. Policies / Local constraints Understanding the psychology of channel members Trained / Skilled manpower

Question 22. IBM, Compaq, Zenith, HP, HCL are some of the leading PC manufacturers for the Indian market. A Taiwan based company considers entry in the market. What are the major attributes on which it can decide to compete by differentiation? Answer: Performance Price Brand Image Features After sales services Facility upgradation/Compatibility with other systems Question 23. What will constitute the macro environment of an oil producing and distribution company like IOC? Answer: Research in the following key areas: 1) 2) 3) 4) 5) Socio-cultural Demographic Economic Technological Politico-legal

Question 24. What is repositioning? How can you reposition Bata? Answer: Repositioning involves altering or changing a products or brands position in the minds of consumers through a change in marketing communication. Bata is positioned as a durable but costly shoe brand. To reposition itself, Bata has to change its pricing strategy, advertising focus and perhaps the distribution strategy too. Question 25. What are the different approaches towards positioning? How is Hero Honda positioned vis--vis Enfield in India?

79

Marketing

Answer:

The different approach towards positioning areAttribute positioning, Benefit positioning, Use or Application positioning, User positioning, Competitor positioning, Product Category positioning, Price positioning and Quality. Hero Honda is positioned in the consumers mind as a fuel-efficient two-wheeler for the college-going or young male, where as Enfield symbolizes macho image. It typifies a rough-and-tough strength.

Question 26. Define

1. 2.

Under positioning Over positioning

Give examples for the above positioning errors and explain Answer: Under positioning: Buyers have only vague idea of the brand. Eg. Coke and Pepsi, in spite of their ads, are still under positioned leading to such trust war Over positioning: Buyers may have too narrow image of the brand. Eg. Cherry Blossom as a brand is so over-positioned that consumers have not accepted C-B shoe horns, C-B instant polish, which are mere brand-line extensions. Question 27. What is value delivery network? How is it different from supply chain management? Answer: Companies partner with specific suppliers and distributors to create a superior valuedelivery network. The value delivery network is a concept broader than Supply Chain Management. It rests on the premise that consumers need value addition through consumption rather than just access to the products/services. Value connects customer driven approach, rather than an efficiency-driven approach as in Supply Chain Management.

Question 28. What are the marketing strategies appropriate for each stage of the product life cycle? Answer: Introduction: Skimming, Penetration Growth Stage: Product Improvement, entry into the new market, enhancement of distribution channel Maturity stage: Market modification, product modification, and marketing mix modification Decline stage: Harvesting, Divesting Question 29. In a competitive industry, what can a market leader do to expand, defend, and prolong its market leadership? How have Cinthol, Cibaca tooth paste, Godrej Storewels, Onida T.V etc., been able to do it? Answer: Expand the market. Redefine the boundaries. Stimulate new customers to enter the market. Have an abundant mentality.

80

Frequently Asked Questions in Management

Have courage to attack oneself. Kill your own products through introduction of new products like the way 3M does. The Leader defines the race. He sets the bar and becomes the trendsetter. Like Sony does in consumer electronics. Strong competitive moves should always be blocked. This is the game played by the market leaders. And you find strands of this strategic theme amongst Cinthol, Cibaca Toothpaste, Godrej Storewels, Onida TV etc. Question 30. Give an example of any reputed company to elucidate the following terms. Mission, Vision, Strategy, Goal, Marketing Plan and Product Plan. Answer: Coca Cola India Mission: To take coke at the arms reach of desire Vision: Vast potential in increasing 10 to 15 times in terms of per capita consumption of beverage Strategy: Building Coke brand and retail infrastructure to achieve twin objective of increasing market size and taking leadership position in cola market Goal: Rs10, 000 crore by 2004 Marketing Plan: Sales target and resource requirement in terms of financial budget, manpower, logistics for a market like Delhi, or any other specific market. Question 31. How is strategic planning carried out at the corporate and business level? Do you know any model, used for carrying out strategic planning exercise? Can you briefly explain the model with the help of a suitable example? Answer: Corporate level: SBU wise Business unit level: Product-market wise BCG matrix Strategic Management Model consists of the following: 1. Strategic analysis & Choice a. b. c. d. Mission External Environment Analysis (PEST Analysis, Porters 5 forces analysis) Company Analysis (Value Chain Analysis, comparison with competitors) Analysis & Choice i. ii. iii. 2. At the corporate level BCG Matrix, GE Nine Cell Matrix At the business level SWOT analysis Selection of Strategy Selection Matrix, Model of Grand Strategy Clusters, Ansoffs product-market matrix

Strategy formulation a. LT Objectives

81

Marketing

b.

Grand Strategies Concentration strategy, Product development, Market development, Innovation, Vertical integration, Horizontal integration, Concentric diversification, Conglomerate diversification, Joint venture, Turnaround, Divestiture, Liquidation. Annual Objectives Functional Strategies

c. d. 3.

Strategy Implementation a. b. c. d. Policies Structure Control and Evaluation Feedback

Explain by giving the example of Infosys, HLL etc Question 32. Marketing strategy is a series of integrated actions leading to a sustainable competitive advantage, said John Scully. Can you explain competitive advantage to a non-MBA, semiliterate dealer of yours? How? Answer: The concept of competitive advantage means a superior position relative to competition. There are two questions. 1. 2. Do I perform some function in a superior /distinctive way, compared to competition? Does the superiority/distinction mean something in terms of customer value?

It is essentially a position of superiority on the part of the firm in relation to its competition in any of the functions/activities performed by the firm. The functions/activities may include R&D, production, finance, marketing etc. The superiority may also cover the resource and capability dimension and factors like technology. The big winners in any industry usually possess superiority/distinction in several functions/areas. Competitive advantage stems from Positioning and the Value Chain tailored for that positioning. Based on Segmentation and Targeting, typically a company adopts one of the three positioning and in turn helps the company in deciding to adopt one of the three generic strategies cost leadership, differentiation or focus. The primary and support activities in the value chain are tailored to meet that positioning. Competitive advantage stems from the unique activities of the company in relation to the competition. The activities have a fit and are mutually reinforcing. It is not the individual activities but the activity system that gives the competitive advantage. Question 33. How can an IT company practice total quality management? Explain with suitable examples. Answer: TQM is an organization wide approach to continuously improving the quality of all the organizations process, products, and services.

82

Frequently Asked Questions in Management

IT companies practice TQM through the Capability Maturity Model (CMM) formulated by Software Engineering Institute (SEI) of Carnegie Mellon University. CMM Level ranges from 1 to 5. At level 5, the error level is at its lowest. Higher the level, more is the capability in the prevention of errors through matured processes. In India, Infosys, Wipro, Satyam have achieved CMM Level 5 Question 34. Describe core competence for the following companies. Colgate & Palmolive, Microsoft, Haldiram Bhujiwala, SBI, Bharat Sevashram. Answer: Colgate & Palmolive: Branding Microsoft: Leadership position Haldiram Bhujiwala: Franchisee network SBI: wide reach of its branches Bharat Sevashram: Network and image Question 35. What do you understand by the intensive growth strategy? Answer: Intensive growth strategy is used to identify opportunities to achieve further growth within the companys current businesses. The three intensive growth strategies are Market-penetration strategy, Market-development strategy and Product-development strategy.

Question 36. What are the major dimensions along which a companys marketing offering can be differentiated? How can you differentiate a service offering say, a banking, insurance, a consultancy etc? Answer: Forms Features Performance quality Conformance quality Durability Reliability Reparability Style Design Service differentiation Personnel differentiation Channel differentiation Image differentiation The differentiation strategy revolves around aspects other than price. A firm adopting differentiation strategy can price its product on the perceived value of the attributes of the offer. Differentiation helps a firm move away from price competition. In the marketplace, firms differentiate not only on the unique features of their products, but even on simple facts like the collaboration, location of plant etc. Firms use any particular

83

Marketing

one that gives them a relative advantage. To make differentiation of work, a firm should possess relevant competitive advantages. Citibank differentiates on its personalized service. It claims that it employs only professionally qualified personnel and the person who answers a customers phone call will be competent to solve all the problems faced by the customer. IBM uses technology and service as its differentiation planks. Caterpillar Tractor uses its service strength / global dealer network. Rolls Royce, its quality We must also appreciate that Price and Differentiation cannot remain mutually exclusive. Jack Welch, the CEO of GE, aptly captures this idea when he says that in a highly competitive market, a firm has to offer the best product, coming out of the best technology, at the lowest price. In short, meaningful differentiation is competitively more effective and enduring than low-cost production alone. When the two strategies are combined in a single company, the results are spectacular. So the winner is one whose offer is distinct and also price competitive. Question 37. Differentiate between a Joint venture and a licensing as strategies to enter a foreign market. Which is a riskier proposition? Why? Answer: Licensing is an entry and expansion strategy with considerable appeal. A company with technology, know-how, or a strong image can use licensing agreements to supplement its bottom-line profitability with no investment. The only cost is the cost of signing the agreements and of monitoring their implementation. The principal disadvantage of licensing is that it can be a very limited form of participation. Potential returns from marketing and manufacturing may be lost. The agreement may have a short life if the licensee develops its own know-how and capability to stay abreast of technology in the licensed product area. In some cases, licensees may turn themselves into competitors or industry leaders. This is especially true because licensing enables one company to leverage and exploit another companys resources. The advantages of a joint venture include the sharing of risk and the ability to combine different value chain strengths. One company might have in-depth knowledge of a local market, an extensive distribution system, or access to low-cost labor or raw materials. Such a company might link up with a foreign partner possessing considerable expertise in the areas of technology, manufacturing, and process applications. Companies that lack sufficient capital resources might become partners to jointly finance a project. Finally, a joint venture may be the only way to enter a country or region if governments favor local companies. Alternatively, there may be local laws that prohibit foreign control but permit joint venture. Joint ventures have their own disadvantages. The main disadvantage is the very significant costs of control and coordination associated with working with a partner. Also, as noted previously with licensing, a dynamic joint-venture partner can evolve into a stronger competitor. Cross-cultural differences in managerial attitudes and behavior can present formidable challenges as well.

84

Frequently Asked Questions in Management

Question 38. A South Korean Company is going to enter Indian market very soon. They hire you as a marketing consultant. What kind of analysis you should be carrying out for them? Answer: Industry analysis Suppliers, buyers, substitutes, threat of entry, rivalry Porters diamond Related industries, Competitive rivalry, Factor conditions, Demand conditions Political risk analysis Stability of government, government policies, etc. Question 39. How has Mohans Meakins benefited with the entry of Kelloggs in the breakfast cereal market? Answer: Kelloggs invested huge amounts, educated customers and created the market. Mohan Meakins has taken full advantage and has grabbed a good market share. Essentially, Mohan Meakins has reaped free rider advantages.

Question 40. What should be the right recourse for a typical Indian company in the confectionery business like Parrys in the context of globalization of business? Please give examples in support of your argument. Answer: It is inconceivable that Parrys will be not able to compete with MNCs who have deep pockets. It may be better to sell off its brands for a good price now and become a contract manufacturer for one of the global MNCs. Eg.- Parle brands such as Gold Spot, Limca, Citra sold to Coke Question 41. BATATA was a strategic alliance of three very reputed companies. Can you name them? What are the factors responsible for the formation of such a strategic alliance? In general, why would a company go in for a strategic alliance? Answer: Birla, AT&T, TATA MNCs form strategic alliances for various reasons. A few of them are listed below. 1. 2. 3. 4. 5. 6. 7. To access new markets eg: Mobils alliance with BP to penetrate European markets. To gain access to local distribution network eg: P&Gs joint venture with Godrej in India. To improve manufacturing processes and gain access to new technology eg: HCLs tie up with HP in India. To gain access to management know-how eg: Elbees tie up with UPS in India. To gain access to additional financial resources eg: Nissan's tie up with Renault in Japan. To achieve risk reduction eg: collaborative research efforts between Siemens and Philips in the semiconductor business. To pre-empt competition eg: the recently announced alliance between General Motors and Fiat.

85

Marketing

Question 42. What is the impact of Chinas entry into WTO? Do you think India needs to panic about Chinas competitiveness? Answer: China has demonstrated in the past two decades that it is very competitive in the exports of low cost manufactured items. In items like toys for example, China is a truly global player. The Chinese are also attempting to replicate their success in consumer durables. But in truly value added items, the Chinese are still way behind. The Chinese are also handicapped by lack of knowledge of English. But the Chinese can never be underestimated. They are making a lot of effort to move into high tech industries. The overseas Chinese are playing a very important role in this regard. So, Indian companies must take the Chinese seriously.

Question 43. Is it possible to improve both, the customer satisfaction and profitability, simultaneously? Can you give an example for that? Answer: Yes, it is possible to improve both the customer satisfaction and profitability, simultaneously. Sony Corporation: Quality Maruti Udyog limited: Value for money Hindustan Lever: Brands, Distribution network Sail: Largest steel maker of India Taj Group hotels: Premium but unique Five star hotels Calcutta university: Traditional and orthodox system Question 44. Do you think satisfying 100% customers all the time is a worthwhile mission for a company? Why? Answer: It is difficult to be all things to all people. The best position to take would be that of differentiation. And this would mean segmenting the market, targeting the marketing and positioning the product with differentiation. The factors that allow a company or product to stand out in an increasing competitive market-place area. Tyranny of choice: Consumers have more choices. Companies must give customers the tools they need in purchasing decisions, to draw them to their products. These days the average supermarket stocks 40,000 brand items. However an average family gets 80% of its needs from 150 brand items, which means that there is a good chance the other 39,850 items in the store will be ignored. Those that dont stand out will get lost in the pack. Companies must address differentiation in 3 key ways. 1. 2. 3. If you ignore your uniqueness and try to be everything for everybody, you quickly undermine what makes you different. If you ignore changes in the market, your difference can become less important. If you stay in the shadow of your larger competitors and never establish your differentiation you will always be weak.

86

Frequently Asked Questions in Management

b.

Reinventing the USP: Companies must move away from differentiation based solely on product, and engage consumers in ways that truly reach them. Gillette reinvents shaving every few years: with two bladed razors (Trac II), adjustable two-bladed razors (Atra), shock-absorbent razors (Sensor) and now with three bladed razors (Mach 3). The last product is the result of $750 million in research, patents, testing and all-round excruciating hard work.

c.

Successful differentiation strategies: It has little to do with creativity or imagination and more to do with a logical approach to engaging customers. 1. 2. 3. 4. Be first. Gillette pioneered razor blades and remains the leader. Maintain Attribute Ownership. Specialize in your market. Examples of Gap, Victorias Secret and Foot Locker Make your product in a special way. When Crest introduced its fluoride cavity prevention toothpaste, they made sure everyone knew that it contained Flouristan, though no one knew what that was. However, it sounded impressive. Make sense in Context. Nordstorms idea of better service played perfectly into the context of a department store market that was reducing its people and service as a way of cutting costs. Find the differentiating idea. Have credentials. As the worlds favorite airline, British Airways should fly more people than any other airline. Communicate your difference

Four Steps to Differentiation: Step 1:

Step2: Step3: Step4: d.

Growth and Sacrifice in Differentiation: Growth can kill differentiation by tempting companies to thin out their product lines in search of mass acceptance. The negative effects are very well explained in two key ways. 1. 2. 3. The company becomes distracted The company overextends its product lines. McDonalds, for instance, built a successful business on inexpensive, high-speed cheeseburgers. When the company decided to branch out into pizzas, chicken and kids menu items, its growth slowed and its hold on the fast food market weakened. When you study categories over along period of time, you can see that adding more can weaken growth, not help it. The more you add, the more you risk undermining your basic differentiating idea.

Pursue profitable growth. And not growth for growths sake

87

Marketing

RETAIL & FMCG MKTG


Question 1. Answer: What is retailing? Is it more than just distribution? Is Amazon into retailing or distribution of gift books and toys? Retailing involves directly catering a product / service needs to the individual customers. Retailing operations include stocking / displaying products in quantity and size needed by individual customers and providing ease and convenience of shopping making it a pleasurable experience with prompt and reliable service. Distributor handles bulk volumes and supplies products to other channel members like whole sales, dealers and stockists who in turn transfer products down the line to individual customers through retailers. Amazon is into retailing and not distribution. It initially tried but failed due to vast coverage of operations involved in distribution. Question 2. Answer: Can you name a major retailing organization? If yes, briefly describe the functioning of that organization? Food world 1. 2. 3. 4. 5. 6. 7. 8. Question 3. Answer: Sourcing products Logistics Management Stocking / Display Publicity / Promotion Personal selling Servicing Handling customer grievances Training personnel

Explain the significance of promoting retail revolution in India. At the outset more than one component of the society can get involved in promoting retail revolution. The government, the consumer, the manufacturers of goods & services and the retail trade itself. Each of these member groups has differing reasons to promote retailing. The government would like to do so for generating employment for the youth of the nation. The consumer would like to see such a promotion with a hope of getting quality goods and services, greater convenience and a feeling of social betterment. The trade would consider this as an emerging opportunity and the manufacturer looks at retail as a more professional and a low cost alternative to the loosely structured traditional retail function. Can you describe the current trends in retailing in India? In comparison to the retailing in developed countries, how do you view retailing in India? In contrast to developed countries, retailing in India is still in the initial stages. Rapid transition is taking place in the market with super markets coming up to cater to customers the entire range of products needed regularly under one roof. Shop display

Question 4. Answer:

88

Frequently Asked Questions in Management

and promotions are becoming popular to attract consumers. In Indian market, there is untapped market potential in small towns and villages. a) b) c) d) e) f) Supermarkets are coming up in all metros and mini metros in India Supermarkets will also spring up in B and C class of cities Departmental stores will differentiate based on image/products IT oriented processing Home delivery Better and more pleasurable shopping experience to the consumer

Current Trends: 1. Retailers are getting more important in the chain. 2. E-tailing is catching up. 3. Store brands getting popular. 4. Emergence of super / giant stores. Question 5. Answer: Will the kirana stores suffer because of the retail revolution? The traditional kirana stores that was typically located in a busy locality, operating from a poorly-lit and loosely arranged stocks, will continue to exist though with a changed setting. While the total retail industry is expected to touch a Rs.800,000 crore mark by the year 2005 supported by a 20% annual growth rate, the organized sector will have a share of Rs.160,000 crore only, indicating that the bulk of the retail trade would continue to be with the traditional kirana store. However, it can be expected that the more enterprising among the kirana store owners might spruce up their shops to conform to the raised expectations of the customer in terms of display, lighting etc. Another possibility is a clear polarization of the customers around the type of stores resulting in clearer market segments. Differentiate between store and non-store retailing? Give examples of each. When the goods / services are stocked and sold in retail from a premises, it is called Store Retailing. Eg: Food World. Goods / services sold directly without intermediaries, marketing directly at door step, sale through automatic vending machine or buying services directly from the service provider fall into the category of non-store retailing. Eg: Modicare products, Ordering books / cassettes on the net from manufacturers, Electricity supply Question 7. Answer: What is the retail format suitable for the Indian customers? Indian customer comes under a very heterogeneous group. During the periods of limited or no choice the entire population was sourcing its requirements from the same or at best from similar-looking outlets. The population, would even flock to the public distribution shops (Ration shops) during times of poor supply and no options. Howeverduring periods of choice and variety being the trend, retailing will be no exception. We

Question 6. Answer:

89

Marketing

will have a large variety of outlets serving the demands of the people. We may have exclusive outlets (ex: textiles, electronics etc.,) or outlets having a wide range of goods and services from groceries to financial services or outlets focused on specific income groups. Looking at the developments of the society we may have stores classified on incomes and therefore the price of the merchandise. Question 8. Answer: Are you comfortable with retail marketing? My interests in retailing were declared in the affirmative when I chose marketing. If Advertising is the apparent facet of marketing, retailing is the revenue- generating center for the organization. I would certainly be comfortable with retail marketing. Tell us what you know about the FMCG industry? FMCG stands for Fast Moving Consumer Goods. The FMCG consists of those goods that are purchased frequently by the consumers. Therefore, customer retention is an important aspect of the marketing function. Also FMCG goods also need be sold through the outlets that provide convenience of purchase.

Question 9. Answer:

Question 10. How is industrial marketing different from consumer marketing? With the help of suitable examples, can you illustrate how the buyers (in two different cases) make their purchasing decisions? Answer: Consumer Products * Large consumer universe * Emotional buying decision * Generally low unit value * Low purchase volumes * Few layers for purchase decision * Limited after sales service * Limited customer relationship Industrial Products * Small consumer universe * Rational procurement decision * Generally high unit value * Bulk purchases * Multiple layers in decision process of procurement. * Pronounced and regular after sales service. * Apart from customer relationship, concept of reciprocity. -Each exchanging their products eg : Steel mills buy compressors while Compressor manufacturers procure steel

90

Frequently Asked Questions in Management

SALES & DISTRIBUTION


Question 1. Answer: What are the five most crucial skills required by a sales manager? Enhancing effectiveness on all of the following: i. ii. iii. iv. v. vi. Question 2. Answer: Choosing the right time for negotiation Formulating a negotiation strategy Relationship marketing Sales Force Training Negotiation Leadership/Team building

Can you by giving examples illustrate selling concepts? Selling concept presumes innately the non-consuming disposition of the human being. Therefore, the individual will have to be communicated about the various aspects of the product that would result in a purchase. While talking of the evolutionary order of the concepts marketing concept is more advanced than the selling concept, the latter one is still relevant in developing economies like India. For example, in case we need to tell the customer the superiority of a toothbrush over a neem stick, then we need to demonstrate the ease of use and superior quality of the output, which is part of the selling process. We need to ensure that the stock of product is sufficient and therefore monitoring stock at the retailer or the wholesaler level is an important function of the sales person. Identify the activities in the distribution value chain. In general, wholesalers are used when they are more efficient in performing one of the following functions 1. 2. 3. 4. 5. 6. 7. 8. 9. Selling and promoting Bulk breaking Warehousing Transportation Financing Buying and assortment building Risk bearing Market information Management services and information

Question 3. Answer:

Appointing wholesalers can be done when the market size and the customer universe are large. Management may not find it feasible to establish branches, depots and employ large sales force. Reducing organization overheads and ability of the channels to market the products effectively are the other benefits. Question 4. Sell this pen in 10 different ways?

91

Marketing

Answer:

Before communicating the 10 ways to sell a pen we need to appreciate the consumer in terms of the reasons that are at the top of the mind before he makes a buy. He might buy a pen as a replacement, to gift to his child, to match with his dress, to communicate a status etc. Therefore while selling a pen, I will ensure that I am aware of the objective of the purchase and thereafter each objective is a way to sell. How would you sell software packages? Software is a service and therefore not tangible. The customer would look at the service with a greater doubt than he does goods. I would therefore use aspects like my earlier products, the qualifications of my technical people etc., as evidences of my abilities that would help the customer reduce his ambiguity. Thereafter I would provide details of my offer on a comparative platform to highlight the advantage of my offer. This would bring the customer closer to my offer and at this stage will clinch the deal. How can a ready made shirt manufacturer make its sales promotion more effective? It requires an analysis of the relation between individual incentives to the consumers and their impact on sales. The sales promotion can be in terms of a free bag promoted through Point of Purchase. How do you sell space while working in a newspaper/ media company? Selling media space to advertising agencies and to the brand promoter is different. When we consider the media group, the members of the group are presumed to be knowledgeable about the prices, comparative circulation figures etc. Their major consideration would be in terms of favorable positions, payment norms, comparative benefits etc. However, for the brand promoter the dimensions of reach and the corresponding cost to reach will be of great relevance. Added to these differences, media space also being a service needs to be tangibalised using facts and figures to start with. The selling function will also include feedback to the editorial team as to enable them plan specific industry-based coverage etc. The sales person will also take up to effectively communicate, the plans of the newspaper to increase, general as well as specific group, circulation. What do you mean by single level system and multilevel marketing? Give one example of single level system and multilevel marketing? In single level system, only one level of the channel directly markets the product. In multilevel marketing, different channels directly market products to customer. The example of single level system is Tupperware and the example of multilevel marketing is Amway.

Question 5. Answer:

Question 6. Answer:

Question 7. Answer:

Question 8. Answer:

Question 9. Answer:

What decisions do companies make in designing sales force? How do you go about allocating territories to sales force? Development of sales force objective Strategy - How much market, what growth rate, sales target Structure - Multi-layered, direct marketing

92

Frequently Asked Questions in Management

Size - Geographic expanse of the market Compensation - Rewards or returns One way of allocating territory is to divide the market into zones and put people familiar with the zones. There are advantages/disadvantages with each way. One has to weigh the gains & losses and then decide. Question 10. What are the various sales forecasting methods? Answer: A range of methods may be used to forecast sales. However, the choice of the method to be used will depend on a number of factors including, the demands on accuracy, the costs involved and the frequency with which the forecasts needs to be made. The methods that are usually used to forecast sales are time series techniques, causal techniques, and the judgmental methods.

Question 11. Differentiate between a C&F agent and a distributor? Answer: C & F agent does not take title of the goods whereas a distributor takes title of the goods while in transit. Major differences are: C & F Agent Keeps custody of product Distributes as per company directives Earns fixed commission Distributor Buys and Stocks products Develops his own channels Earning depends on markup he can get based on Max. price specified by the company Has own sales force / promotes product on his own

Sales personnel / support given by Co.

Question 12. What are the differences between product management and sales management? Answer: Product management as an organizational alternative to sales management is a feature of the multi-product organization. Moreover, when markets are large such functioning is very essential. The product management takes the total responsibility of the product from the stage of idea generation to the stage of commercialization. Sales management as a functional choice takes care of sales and the allied issues.

Question 13. What are the roles of marketing channel in the overall function of marketing? Explain the role of the channel in the following areas of marketing? a) b) c) d) e) Answer: Marketing of a real estate project Marketing of soaps and detergent Marketing of a major event Marketing of software products and services Marketing of insurance products

The roles of marketing channel:

93

Marketing

1. 2. 3. 4. 5. 6. 7. 8. a) b) c) d) e)

Transfer of goods Transfer of ownership Placing of the product Collection of information Handling risk connected with carrying out the channel work Placing order to manufacturing Bulk breaking and repacking to suit other channel members / customer Storage and movement of physical product Marketing of a real estate project-Transfer of ownership Marketing of soaps and detergent- Transfer of goods Marketing of a major event- Transfer of information Marketing of software products and services- Transfer of ownership Marketing of insurance products-Collection and dissemination of information

The roles of the channel in the following areas of marketing are

Question 14. What are the various types of distribution channels? Answer: We may classify the channel types based on either ownership or number of levels or nature of activities. The manufacturing company may have its own outlets like Raymonds or Reliance. A single organization other than the manufacturing one might own a chain of retail outlets like the India coffee house. Alternatively each location might be taken up by a distributor who has nothing to do with another person in another area. The channel may be classified as a single tier or multi tier depending on the number of levels of the intermediaries. We also have the types of wholesalers, retailers etc.,

Question 15. What is the function of a stockist? Answer: The stockist is a key element in the value delivery system. The stockist stores the products of the company and routes them through the retail network to the customer. Usually a stockist operates within a defined geographical domain. He also functions as an information conduit for the company.

Question 16. What is merchandising? What are the differences between distribution and merchandising? Answer: Merchandising is a term commonly used in the retailing industry. When we say merchandise we refer to a category of items like textiles, toiletries detergents etc., Therefore a merchandise manager in retailing would be a n equivalent to the product manger of goods marketing organization. The distribution function predominantly concerns logistics or making the goods available for consumption while merchandising includes creating availability as well as enabling sale of the merchandise. The decisions that are made by a merchandiser are the categories of items that need to be made available, what will be the display pattern, what will be the discount structure and what

94

Frequently Asked Questions in Management

types of reordering norms are requires. Therefore the merchandise manager will consider demand, availability requirements, margins, discounts etc. Question 17. What are the benefits of direct marketing vis--vis channel marketing? Answer: Advantages of Direct Marketing are: 1. 2. 3. 4. Get to know the customer pulse Can control Quality Can get customer feedback efficiently Brand & Company ethos can be transferred

Question 18. What is channel conflict? How does the role of a channel manager in terms of resolving channel conflicts? Why does it arise in the first place? Answer: Individual channel members are often more concerned with their short run goals, leading to channel conflict. Horizontal conflict occurs among firms at the same level of the channel. Vertical conflict occurs between different levels of the same channel. Channel conflicts are common when the market size is limited and all channel members try to concentrate on the same market and customers. Sometimes differential pricing policy of organization also results in conflict among channel members. Question 19. What is value delivery network? How is it different from supply chain management? Answer: Companies partner with specific suppliers and distributors to create a superior valuedelivery network. The value delivery network is a concept broader than Supply Chain Management. It rests on the premise that consumers need value addition through consumption rather than just access to the products/services. Value connects customer driven approach, rather than an efficiency-driven approach as in Supply Chain Management.

Question 20. How can channel conflict be controlled? The big retail players like Gaint Super Market, Food World can give discounts on the strength of their volume of business. Small retailers are crying fowl. How will you resolve this conflict? Answer: The channel conflicts can be controlled by i. ii. iii. Adoption of super ordinate goals Exchange persons between two or more channel levels Co-option

The resolving of the above said conflict can be done by a. b. Joint membership in and between trade associations Diplomacy, mediation, arbitration

95

Marketing

MARKET RESEARCH
Question 1. Answer: What is market research? How can a company benefit from it? The systematic design, collection, analysis and reporting of data relevant to a specific marketing situation facing an organization is marketing research. Marketing research is an input for making better and informed decisions. What are the various steps involved in market research? The steps involved in the market research process include: defining the problem and research objectives; developing the research plan; implementing the research plan and interpreting and reporting the findings. What tools do you use for market research? Tools of market research include commercial data sources, online databases and Internet data sources, observational research, survey research, experimental research, focus group interviewing, and in-depth interviewing. What factors distinguish a good marketing research from a poor marketing research? What is research fatigue? 1) 2) 3) 4) 5) 6) 7) Scientific method Research creativity Multiple methods Independence of models and data Value and cost of information Healthy skepticism Ethical marketing

Question 2. Answer:

Question 3. Answer:

Question 4. Answer:

Good marketing research is having characteristics of reliability and validity, which is generally void in poor marketing research. The research when used over a period of time should yield some accuracy (reliability) and should measure what it is expected to major (validity). Research Fatigue is one of the errors, which can occur on behalf of respondents when timing of collecting information is wrong, or it can be an error occurring on behalf of interviewer who is over burdened with research and time frame to do that. Question 5. Answer: What do you know about market intelligence? Marketing intelligence is the systematic collection and analysis of publicly available information about competitors and developments in the marketing environment. A marketing intelligence system gathers, analyzes and distributes information about the companys competitive, technological, customer, economic, social, political and regulatory environments. Its goal is to improve strategic decision-making, assess and track competitors actions and provide early warning of opportunities and threats.

96

Frequently Asked Questions in Management

Question 6. Answer:

What is need for having a marketing information system? Explain this with the help of a cola company? To carryout analysis, planning, implementation and control responsibilities; and take appropriate decisions marketing managers need information about the developments in the marketing environments. MIS supports them in the decision making, providing needed information in a timely, accurate and specific manner. To understand and evaluate how the company is performing with respect to its competitors. For a cola company the issues of information are: Consumer related areas Present need-gap in the minds of the consumers Trend in terms of changes of taste and preferences Level of maturity of the consumers with the product category Trade & Distribution related areas Level of reach Distribution efficiency and effectiveness Sales & Promotion Related areas Effectiveness and efficiency in the areas of sales promos

Question 7. Answer:

Differentiate between primary and secondary data? Which would you rely more? Why? Primary data is the information collected for the specific purpose at hand whereas, Secondary data is the information that already exists somewhere, having been collected for another purpose. The primary data is comparatively more reliable as the information collected for the problem in hand is relevant, accurate and sufficient.

Question 8. Answer:

What are the ingredients of secondary research? Secondary data consist of information that already exists somewhere, having been collected for another purpose. Secondary research is a good starting point and often helps to define problems and research objectives. In most cases the company must also collect primary data. In what a way a focus group is different from an interview? When would you employ focus group? A focus group is a gathering of six to ten people who are invited to spend a few hours with a skilled moderator to discuss about a product, service organization, or other marketing entity. Whereas, interview is for gathering information quickly from individuals by posing some questions. Its one of the primary research tools used and a qualitative study. Its used in the early stages of attribute research to pin-point relevant product related beliefs on attributes and

Question 9. Answer:

97

Marketing

to develop an internal picture of consumer attributes (especially the beliefs and attributes they associate with particular products and services). Question 10. How would the marketing research be different for industrial products as compared to FMCG? Answer: Industrial products buying is more of rational buying whereas consumer products are more of emotional buying. Practically speaking, there is much difference in marketing research between industrial products as compared to FMCG. It all depends on the research objective and management problem rather than on types of goods. The difference may be in using combination of qualitative techniques and quantitative techniques of research.

98

Frequently Asked Questions in Management

ADVERTISING & BRANDING


Question 1. Answer: What do you understand by advertising? Advertising is any paid form of non-personal presentation and promotion of ideas, goods, or services by an identified sponsor. State the difference between Advertisement and Publicity? Advertisement is paid whereas publicity is not paid. Ads create short-term push whereas publicity is more long-lived effecting the brandimage. Normally publicity is a one-way (irreversible) process. Question 3. Answer: What do you understand by surrogate advertising? Surrogate advertising is an indirect form of advertising, which is used in situations where advertising will be legally banned. In marketing, how does communication work to the customer? Can you describe briefly the various modes of communication of a leading consumer finance company? The various modes of communication of a leading consumer finance company can be Advertising Sales promotion Public relation or publicity Personal selling Direct marketing Question 5. What kind of advertisement (rational / emotional), do you think, will work well for marketing of the following products / services? a) b) c) d) e) f) g) Answer: a) b) c) d) Soaps & Detergents Adhesive Refrigerator Timeshare Marketing Marketing of Mutual Fund Anti Smoking Campaign Milk Based Ice-cream Soaps and detergent: More on rational for detergent Adhesive: More on Rational Refrigerator: More on Rational Timeshare Marketing: Depends on the Package

Question 2. Answer:

Question 4. Answer:

99

Marketing

e) f) g) Question 6. Answer:

Marketing on Mutual Fund: Rational Anti Smoking Campaign: Balanced Milk Based Ice-cream: More on Emotional

In advertising when is pricing used to influence the customer? In advertising, pricing influences consumer behavior leading to purchase and nonpurchase. The factors, which affect these relationships are: unique value effects, substitute effect, shared-cost effect. What proportion will you adopt while making allotments to advertisements and promotions from the total budget for marketing? The five specific factors which affect advertising budget allotment are: stage in plc; market share and customer base; competition and clutter; advertising frequency and product substitutability. Media categories can be chosen using the following variables: target audience media habits, product features messages and costs. What are the major steps in developing an effective marketing program? Can you develop a communication program for a Television company for the Indian market? How effective are road shows? An effective marketing program should be developed with an appropriate mix of advertising, sales promotions, public relation, personal selling and direct marketing based on the resources of the company and the marketing objective. Road shows are effective for products that are new models and consumers need to be taught as to how to use them. Television in India is a matured product. Features of TV are not new to the consumers. Hence road-shows for TVs may not be a good idea. If the advertisements are rational highlighting only one or two features to position the TV or to strike a cord with the consumer it is an effective marketing program.

Question 7. Answer:

Question 8.

Answer:

Question 9. Answer:

Tell us something about brand positioning. Positioning is the act of designing the companys offering and image in target consumers mind.

Question 10. What do you mean by reach and frequency? Do you think these factors are inversely proportional in a given budget? Why? Answer: The number of different persons or households exposed to a particular media, schedule at least once during a specified time period - is called reach. Whereas, the number of times within the specified time period that an average person or household is exposed to the message - is called frequency. They are not inversely proportional. One can choose different channels to use different segments using appropriate frequency. However, within a segment, the rule seems to be applied. Question 11. What is the difference between a brand and a product?

100

Frequently Asked Questions in Management

Answer:

While brand is a name, term, symbol, design or logo, product is of an identifiable generic nature object.

Question 12. What are the major areas, concerning managing brands? What is the negative impact of Brand Management in the company? Answer: The major areas of concern for a company 1. 2. 3. 4. To brand or not Cost and benefit of developing brand equity Brand strategy Brand repositioning

Negative impact of brand management portrays a Turf mentality among brand managers leading to cannibalization of a few brands and Sub optimal utilization of resources. Question 13. Give examples of print ads or TV ads for the following themes 1. 2. 3. 4. 5. 6. 7. 8. 9. Quality Economy Performance Fear Guilt Humor Pride Sympathy Trust

10. Community 11. Social consciousness 12. Nationalism 13. Warmth of emotion Answer: 1. 2. 3. 4. 5. 6. 7. 8. 9. Quality: Its a Sony Economy: Babool Paisa Vasool Performance: Sensor Excel, Tata Sumo Fear: Microsofts campaign against software piracy, AIDS, Tobacco Guilt: Income Tax Campaign Humor: Ruffle Potato Chips Pride: Most of the premier quality clothes, Onida Sympathy: Awareness campaign against child labour Trust: Moov, Dettol

101

Marketing

10. Community: Sahara India 11. Social consciousness: Benetton Ads 12. Nationalisation: Bajaj Auto 13. Emotion / warmth: Johnson & Johnson baby products Question 14. Is Advertising the only way to entice a target market towards product? Explain. Answer: Advertising is not the only way to entice customers to buy the product. Advertising itself cant sell products or services. It is one of the elements in the marketing mix of product, price, place and promotion. And the four elements of the marketing mix when focused on consumer satisfaction results in attracting the target consumer to the product or service. Advertising is an effective tool to communicate, convince and induce purchaser behavior of consumers when all the other elements are aligned to consumers tastes and interests.

Question 15. Which are the worlds largest ad agencies? Why do the parent groups have multiple agencies under them? Answer: There are three major worldwide players in the advertising industry- the Omnicom Group, the Interpublic Group, and the WPP Group. Among them they command the bulk of ad revenue globally. Each of these three groups, in turn, has three major global agencies under its portfolio. The Omnicom Group has three of the worlds ten largest ad agencies- BBDO Worldwide, DDB Worldwide, and TBWA Worldwide. INTERPUBLIC Group currently has two major agencies- McCann Erickson and the Lowe Group. The Lowe Group was formed in October 1999 by merging Ammirati Puris Lintas and Lowe & Partners Worldwide. Thus, Interpublic also had three major agencies in its portfolio till recently. WPP Group: J. Walter Thompson and Ogilvy and Mather are the two largest agencies. This group has recently acquired Young and Rubicam. They have multiple agencies under them. These groups do not merge their agencies to avoid client conflicts. Clients do not like choosing agencies that work for their competitors in the same industry. Thus, a group cannot grow beyond a certain point without running into client conflict problems. In order to avoid such problems, groups like Omnicom and WPP keep their agency subsidiaries separate. This multi-agency structure allows the parent group to grow larger than a single agency could. Question 16. What is an integrated multiple channel brand strategy? Explain with an example? Answer: A brand has to be communicated to specific target consumers. Integrated multiple channel brand strategy uses a number of channels utilized for this purpose. Care should be taken to have uniformity or consistency in messages along with subtle fine-tuning depending on the specific segment targeted.

102

Frequently Asked Questions in Management

Question 17. Differentiate between a private brand and a national brand? Is Coca Cola in US a private or a national brand? Answer: Private brand is a brand created by a reseller of a product or a service. The brand owned by a manufacturer for example - Lux of HLL. National brand is a brand having evolutionary lives. Coke started as a private brand, predominant in the US. But today it is a global brand, so much so that the other name for globalization is Coca Colonization, according to some critics of the phenomenon. Question 18. What do you understand by Brand Equity? How is Brand Equity different from Brand name? Answer: The value of Brand Equity is based on the extent to which it has high brand loyalty, name awareness, perceived quality, strong brand associations, and other assets such as patent, trademark and channel relationships. As the name suggests it is more than just a brand name. It is the total of all endeavors that go in creating the image of the brand among consumers. There are various methods of evaluating brand equity. Brand name is only the vocalized part of the brand.

Question 19. Pepsi wants to enter into a new market of breakfast orange juice in Bangladesh. Pepsis acquired Tropicana brand name has been selected for this purpose. Evaluate this brand in terms of both product-market fit and product-company fit. Answer: Product-Market Fit: Essentially targeting the consumer for a different type of usage Product Company Fit: Synergy for achieving total synergy in entire beverage market Synergy in processing related areas also Bangladesh is a developing country with the per capita consumption of orange juice for breakfast quite high. The product necessarily has to be launched after a thorough study of the breakfast pattern of the target market. Tropicana is a well-known brand worldwide but its ability to change the behavior of consumption in a third world country needs to be assured and evaluated. Question 20. Why has Dell become so powerful as a brand in such a short period in spite of Goliaths like IBM, Compaq, HP in the Market place? Answer: Dell may be credited with making a high-tech product like the PC into an appliance. The main reasons can be attributed to 1. 2. On the consumer front, its interface empowering users and directing to end user strategy. Its excellent supply chain management strength cutting its work-in-process to negligible compared to competition.

Question 21. Which are the top ten brands of the world as ranked by Interbrand in 2002?

103

Marketing

Answer:

1. Coca Cola, 2. Microsoft, 3. IBM, 4. GE, 5. Intel, 6. NOKIA, 7. Disney, 8. McDonalds, 9. Marlboro, 10. Mercedes.

Question 22. What is the basic idea behind Co branding? Please give suitable example from India and abroad. Answer: When two brands come together to create greater value to the customer then it is called Co-branding. Coke and McDonalds have done that globally, while Dettol and ICICI Prudential are doing in India.

Question 23. Differentiate between brand extension and line extension? Can these both happen at once? Give an example. Answer: Brand Extension: Where a company uses its existing brand name to launch new products in other categories. Line Extension: It is a type of brand extension. Line extension trap should be avoided to extend brand in products hurting the consumers regard for a company. Example of both the above in a single category - Will Sports is an example of a brand extension to launch a new line of products. Question 24. It is not always profitable to go in for brand building. Do you support this view? How would you brand Indian software development skills? Answer: Cost of brand building Vs Benefit of branding. The point to note is that branding cost is an immediate process whereas the benefits are always long-term goals. If viewed from a long-term image building prospective, it is always profitable to go in for brand building. We may lose out to cheaper labor nations unless we do something to brand our software development skills. Branding a nation is not a new and impossible proposition.

Question 25. As a marketing manager of an Insurance company, how do you go about designing an effective communication program for your company? Answer: Insurance is a pure service. The intangibility of the service makes it difficult for people to appreciate its worth. The advertisement campaign should therefore be straightforward and simple. The message can be benefit-based visual picturization. A comfortable, though bereaved family, old couple (happy), childrens education / marriage etc.

Question 26. Explain the difference between consumer promotion and trade promotion? Which is the most preferable from a companys perspective? Answer: Consumer promotion is directed towards the end consumers whereas the trade promotion is directed towards marketing intermediaries like dealers and distributors. It depends on the stage of PLC (Product Life Cycle) of the product. Initially when the demand is not yet built, the product has to be pushed and here trade discounts or trade promotion in general is advisable. Later to retain loyalty and to convert into a pull marketing, consumer promotions are desirable.

Question 27. What is sales promotion? Give an example of sales promotion? Is giving discount a better option?

104

Frequently Asked Questions in Management

Answer:

Sales promotion consists of a diverse collection of incentive tools, mostly short-term, designated to stimulate quicker or greater purchase of particular products or services by consumers or the trade. It is mostly observed in the Consumer Non-Durables (CND) or Fast Moving Consumer Goods (FMCG), giving a smaller pack free with a bigger. We see in the market, one soap free with three (Cinthol), a smaller bottle of tomato ketchup free with a bigger bottle. Infact, giving discount is not a better option since it tends to dilute the brands image in the consumers minds.

105

Frequently Asked Questions in Management

PART - 4

FINANCE

107

Finance

108

Frequently Asked Questions in Management

FINANCIAL ACCOUNTING
Question 1. Answer: What is the primary objective of Financial Accounting? The primary objective of the Financial Accounting is to communicate and provide information to the investors and creditors on the economic activities of the enterprise that will help them in their investment decisions. What are financial statements? Name the major financial statements. The Financial statements are the reports that result from the process of accounting which allow the interested parties to evaluate the profitability and the solvency of the business. The major financial statements are Question 3. Answer: Profit and Loss Account Balance sheet Cash Flow statement

Question 2. Answer:

What is the difference between balance sheet and profit & loss account? The balance sheet is one of the most important financial statements of a company. It is reported to investors at least once per year. It may also be presented quarterly, semiannually or monthly. The balance sheet provides information on what the company owns (its assets), what it owes (its liabilities), and the value of the business to its stockholders (the shareholders' equity). The name, balance sheet, is derived from the fact that these accounts must always be in balance. Assets must always equal the sum of liabilities and shareholders' equity. A company's income statement/profit and loss account statement is a record of its earnings or losses for a given period. It shows all of the money a company earned (revenues) and all of the money a company spent (expenses) during this period. It also accounts for the effects of some basic accounting principles such as depreciation. The income statement is important for investors because it's the basic measuring stick of profitability. A company with little or no income has little or no money to pass on to its investors in the form of dividends. If a company continues to record losses for a sustained period, it could go bankrupt. In such a case, both bond and stock investors could lose some or all of their investment. On the other hand, a company that realizes large profits will have more money to pass on to its investors.

Question 4. Answer:

What are the principal qualitative characteristics of financial statements? The principle characteristics of financial statements are the attributes that make the information provided in the financial statements useful to the users. The principle qualitative characteristics are Understandability: They should be readily understandable to the users. For this purpose users are deemed to have reasonable knowledge of business and economic activities. Relevance: To be useful information must be relevant to the decision-making needs of the users.

109

Finance

Reliability: Information is said to be reliable when it is free from errors, bias and can be depended upon by the users to represent faithfully, which it purports to represent. Comparability: Users must be able to compare the financial statements of an enterprise through time in order to identify trends in its financial position and performance. Question 5. Answer: What is meant by the quality of financial reporting? What is conservatism, and how does it affect the quality of earnings? The quality of financial reporting refers to how close the financial statements are to economic reality. The closer the financial statements are to economic reality, the higher is the quality of financial reporting. The less that management uses discretionary means to manipulate earnings, the higher the quality of financial reporting. Conservatism means that management should take great care not to overstate assets and revenues and not to understate liabilities and expenses. The more conservative management is in making accounting judgments, the higher will be the quality of financial reporting. What are the major constraints on relevant and reliable financial statements? The major constraints are Timeliness: If there is undue delay information becomes irrelevant. Balance between cost and benefit: The benefits derived from information should exceed the cost of providing it. Balance between the various qualitative characteristics: In practice it has become necessary to achieve an appropriate balance between the qualitative characteristics. True and fair view presentation: There is no clarity in the term true and fair view as required by the Companies Act. The conceptual framework does not discuss this. Question 7. Answer: What are the golden rules of Accounting? The golden Rules of Accounting are: Question 8. Answer: Debits always equal Credits Increases do not necessarily equal Decreases Assets - Liabilities = Owner's Equity (The accounting equation)

Question 6. Answer:

What is Fundamental Accounting equation? Accounting equation is a mathematical expression used to describe the relationship between the assets, liabilities and owner's equity of the business model. The basic accounting equation states that assets equal liabilities and owner's equity, but can be modified by operations applied to both sides of the equation, e.g., assets minus liabilities equal owner's equity. What are Accounting Standards? List few advantages. Accounting Standards are rules and criteria of accounting measurement evolved by several accounting standard setting bodies established in developing and developed countries.

Question 9. Answer:

110

Frequently Asked Questions in Management

i. ii. iii.

International Accounting Standard Board (IASB) - International Accounting Standards. Financial Accounting Standards Board (FASB) US Generally Acceptable Accounting Practices (specifically Statements on Financial Accounting Standards). In India Institute of Chartered accountants of India Accounting Standards. Reduces to a reasonable extent eliminates confusing variations in the accounting treatment. Lays down disclosure requirements beyond that required by law. To a limited extent facilitates comparison of financial statements globally.

The advantages

Question 10. Discuss the GAAP measures used in India. Answer: The financial statements are prepared under the historical cost convention, in accordance with Indian Generally Accepted Accounting Principles (GAAP) comprising of the accounting standards issued by the Institute of Chartered Accountants of India and the provisions of the Companies Act, 1956, as adopted consistently by the company. All income and expenditure having a material bearing on the financial statements are recognized on the accrual basis. The preparation of the financial statements in conformity with GAAP requires, that the management of the company (Management) make estimates and assumptions, that affect the reported amounts of revenue and expenses of the period, reported balances of assets and liabilities and disclosures relating to contingent assets and liabilities as of the date of the financial statements. Examples of such estimates include, expected contract costs to be incurred to complete software development, provision for doubtful debts, future obligations under employee retirement benefit plans and the useful lives of fixed assets. Actual results could differ from those estimates.

Question 11. Tell us what you know about Accounts Receivables and Payables? Answer: Accounts Receivable, normally abbreviated as A/R, is the money that is currently owed to a company by its customers. The reason why the customers owe money is that the product has been delivered but has not been paid for yet. Companies routinely buy goods and services from other companies using credit. Although typically A/R is almost always turned into cash within a short amount of time, there are instances where a company will be forced to take a write-off for bad accounts receivable if it has given credit to someone who cannot or will not pay. This is why you will see something called allowance for bad debt in parentheses beside the accounts receivable number. Accounts Payable is the money that the company currently owes to its suppliers, its partners and its employees. Basically, these are the basic costs of doing business that a company, for whatever reason, has not paid off yet. One company's accounts payable is another company's accounts receivable, which is why both terms are similarly structured. A company has the power to push out some of its accounts payable, which often produces a short-term increase in earnings and current assets. Question 12. Tell me something about Accounting for goodwill.

111

Finance

Answer:

Goodwill is considered to be one of the largest intangible assets, the value of which companies want to reflect correctly in their financial statements. Accounting for this asset, poses many challenges for accountants, as it is an unidentifiable intangible asset.

Question 13. Can you provide us a suitable definition of goodwill? Answer: Goodwill as an intangible asset can be defined from two approaches: Residuum approach Under this method, goodwill is taken to be the difference between the purchase price and the fair market value of an acquired companys assets. Excess profits approach Under this method, the present value of the projected future excess earnings over normal earnings for similar businesses is recorded as goodwill. Due to uncertainty of future earnings, valuing goodwill using this method is difficult. Question 14. What is debenture redemption reserve? Answer: The companies (Amendment) Act 2000 require every company to create debenture redemption reserve for redemption of debentures out of appropriation of profits every year until redemption. This reserve cannot be utilized by the company except for the purpose of redemption.

Question 15. What is deferred revenue expenditure? Answer: Deferred revenue expenditures represent types of assets whose usefulness do not expire in the year of their occurrence but generally expires in the near future. These types of expenditures are carried forward and are written off in future accounting periods. Sometimes, we make some revenues expenditure but it eventually becomes a capital asset (generally of an intangible nature). Example, if we undertake substantial repairs to the existing building, the deterioration of the premises may be avoided. If we charge the whole expenditure during the current, the current year expenses are affect. However, since the benefit of this expenditure is enjoyed over a number of years. So, to overcome this only a part of the expenditure is charged current year and the balance carried forward and written off gradually during the future periods.

Question 16. What are contingent liabilities? Answer: These are liabilities, which materialize on the happening or non-happening of an event. Contingent liabilities are not real liabilities and as such do not appear in the liability side of balance sheet. But are disclosed by way of a note in the balance sheet.

Question 17. What is depreciation? List few methods of providing depreciation. Answer: It is common knowledge that when an asset is used over a period of time, it looses its value. This loss in value is called depreciation. Pickles defines it as "the permanent and continuing diminution in the quality, quantity or value of an asset" Depreciation is the continuous shrinkage of book value of an asset. Few method of depreciation are

112

Frequently Asked Questions in Management

Straight line Method: An equal amount is written off every year during the working life of an asset so as to reduce the cost of the asset to nil or its residual value at the end of its useful life. Reducing Balance Method: A fixed percentage of the diminishing value of the asset is written off each year so as to reduce to its break up value at the end of its life. Machine hour method: If it is practicable to keep a record of the actual running hours of each machine, depreciation may be calculated on the basis of the hours for which the concerned machine worked. Question 18. What do you understand by contract account? Answer: Account in which posting data for contracts or contract items are processed for which the same collection/payment agreements apply. Contract accounts are managed on an open item basis within contract accounts receivable/payable.

Question 19. Please tell how you can analyze a balance sheet vis--vis the performance of the company in the capital market? Give examples with reference to some specific parameters. Answer: The analysis of a balance sheet can identify potential liquidity problems. These may signify the company's inability to meet financial obligations. An investor could also spot the degree to which a company is leveraged, or indebted. An overly leveraged company may have difficulties raising future capital. Even more severe, they may be headed towards bankruptcy. These are just a few of the danger signs that can be detected with careful analysis of a balance sheet. Beyond liquidity and leverage, there are certain very important benchmarks and aspects, which are helpful in the analysis of balance sheet. Revenues/Sales growth Bottom line growth ROI - Return on Investment Volume Market Capitalization Company management PSR (Price-to-Sales Ratio) Return on Equity Debt-to-Equity Ratio Beta Earnings Per Share (EPS) Question 20. Define FIFO and LIFO. Explain what effects that FIFO and LIFO have on the balance sheet during a period of rising prices and during a period of falling prices?

113

Finance

Answer:

FIFO is the inventory cost flow assumption that treats the first goods in as the first goods sold. LIFO is the inventory cost flow assumption that treats the last goods in as the first goods sold. In a period of rising prices, FIFO values inventory at current costs. However, LIFO would value inventory at costs that the company could have incurred years ago. The analyst should take the LIFO cost flow assumption into account and consider adjusting the inventory of a company using LIFO upward to account for inflation.

Question 21. What are marketable securities? Answer: Marketable securities are cash substitutes. Marketable securities are investments with short-term maturities with little risk due to interest rate fluctuations. Examples of marketable securities include Treasury Bills, Negotiable Certificates of Deposit, and Commercial Paper.

Question 22. A financial accounting system provides information for external decision makers and a management accounting system provides information for a firm's internal decision makers. Consider the general information needs of these two categories of decision makers. How are their information needs different? How are their information needs similar? What does your consideration of these differences and similarities suggest about the relationship between the financial accounting system and the management accounting system as components of the overall accounting system? Answer: The primary external users of accounting information are investors and creditors whose decisions often require them to make comparisons between companies. To support these inter company comparisons, they need information that is itself comparable. This requires financial accounting information to be fairly standardized in terms of not only its basic manner of presentation, but also in terms of how economic events are identified, measured, and recorded. The information needs of internal users (i.e., managers) are generally more focused on their single firm as they seek to plan and control its operations. Accordingly, inter company comparability is less of a concern in managerial accounting, meaning that standardization of accounting practice is not as important. However, like external users, internal users will have some decisions that require information that is comparable between companies. For example, year-end bonuses might be based on the company's performance relative to other companies in the same industry. Therefore, the information needs of internal and external users generally overlap somewhat. Accordingly, the financial and management accounting systems are not two completely separate systems, but rather are partially overlapping subsystems within the overall accounting system.

Question 23. What is the entry for Deferred Tax liability according to AS22? Answer: Deferred tax assets and liabilities should be distinguished from assets and liabilities representing current tax for the period. Deferred tax assets and liabilities should be disclosed under a separate heading in the balance sheet of the enterprise, separately from current assets and current liabilities. The break-up of deferred tax assets and deferred tax liabilities into major components of the respective balances should be disclosed in the notes to accounts.

114

Frequently Asked Questions in Management

The nature of the evidence supporting the recognition of deferred tax assets should be disclosed, if an enterprise has unabsorbed depreciation or carry forward of losses under tax laws. Question 24. What is Sec 72A of Income Tax Act? Answer: Incentive for amalgamation extended to hotels and certain banks - Sec. 72A. The benefit of carry forward and set off of accumulated losses and unabsorbed depreciation would be extended in the case of amalgamation of a company owning a hotel with another company or an amalgamation of a banking company with the State Bank of India or its subsidiary or other specified banks. Two additional conditions for amalgamating a company will have to be fulfilled, viz. that it should have been engaged in the business for at least 3 years during which the accumulated loss has occurred or the unabsorbed depreciation has accumulated and it has held continuously as on the date of amalgamation at least 3/4ths of the book value of fixed assets held by it two years prior to the date of amalgamation.

Question 25. What is Section 80CCC(1)? Answer: Deduction under section 80CCC(1) This section was introduced with effect from Annual Year 1997-98. Under this section Jeevan Suraksha Pension Plan of LIC of India is most popular option. Jeevan Suraksha Pension Plan is a contributory Pension Plan and contributions paid to LIC of India can be reduced from taxable income subject to maximum of Rs.10, 000/- p.a. Accordingly a person who is in 31.5% tax bracket can save income tax of Rs.3, 150/- (including surcharge) by contributing Rs.10,000/- towards Jeevan Suraksha Pension Plan in a year.

115

Finance

MANAGEMENT ACCOUNTING
Question 1. Answer: What is a Cost? What do you mean by cost unit? A Cost is a resource consumed to accomplish a specified objective. A Cost Unit is a unit of output in the production of which the costs are incurred. What is Budget? Budget is a quantitative representation of the policy to be pursued during a specified period of time for purpose of attaining predefined objectives. What is Standard cost? It is a predetermined or forecast estimate of cost to manufacture a single unit, or a number of units of a product, during a specific immediate future unit of a product. What is a Standard costing? It is the preparation and use of standard costs, their comparison with actual costs, and analysis of variances to their causes & points of incidence. What is a direct cost? A cost which can be economically identified with a specific saleable cost unit. What do you understand by cost center? It is a smaller segment of activity or area of responsibility for which costs can be accumulated. Responsibility in a cost center is restricted to costs only. How is investment center different from cost center? Investment center is a profit center whose performance is measured by its return on capital employed. Cost center is a smaller segment of activity or area of responsibility for which costs can be accumulated. Responsibility in a cost center is restricted to costs only. How does standard costing affect performance? A control technique which compares standard costs and revenues with actual results to obtain variances which are used to stimulate improved performance. How do you calculate opportunity cost? The value of a benefit sacrificed in favor of an alternative course of action.

Question 2. Answer:

Question 3. Answer:

Question 4. Answer:

Question 5. Answer: Question 6. Answer:

Question 7. Answer:

Question 8. Answer:

Question 9. Answer:

Question 10. What is EOQ? Answer: Economic Order Quantity-It represents the quantity of goods ordered which minimizes the sum of inventory ordering costs and carrying costs

Question 11. How do the financial experts use cost sheet

116

Frequently Asked Questions in Management

Answer:

Through the use of a cost sheet, financial experts estimate the detailed cost in respect of a cost center or a cost unit and also makes inter-firm comparison by including cost data of different firms.

Question 12. How do you identify direct labor? Answer: Labor that is directly identifiable with a specific product or activity.

Question 13. What is the importance of Cost-Volume-Profit Analysis? Answer: Managing cost is one of the most important aspects of a successful business. A firm should have a clear understanding of the financial impact of every decision it takes. For example, when a firm acquires loan, its fixed cost increases due to increase in payment by way of interest. In such a situation, a firm should be able to analyze sales volume required to cover the additional cost incurred. Cost-Volume-Profit (CVP) Analysis evaluates various business decisions and helps the finance manager to account for any deviation caused in the profits, by manipulating cost and sales of the firm. CVP analysis can also be stated as the relationship between cost (fixed and variable), volume (in units or in rupees) and profit. Question 14. Can you identify the two basic tools used for CVP analysis? Answer: Contribution margin analysis Break-even analysis Question 15. What is BEP analysis? Answer: Break-Even analysis is an analytical technique to study the relationship between fixedassets, variable costs, profits and sales. The Break-Even Point (BEP) represents the level of sales at which the operating income and operating costs are equal so that profit is zero.

Question 16. How do you calculate contribution margin? Answer: The difference between the selling price and the variable cost of a product or service. Both the per-unit manufacturing and non-manufacturing variable costs are deducted from the selling price to determine the contribution margin. In aggregate, contribution margin is the difference between total sales and total variable costs. Contribution Margin Ratio = Sales Variable Costs / Sales Question 17. What is break-even point for a company? Answer: The activity level that yields zero profit. It is the level at which there is neither profit nor loss. Here the total revenue equals the total costs.

Question 18. How do you calculate absorption costing? Answer: A product costing method in which all costs of production, direct and indirect, fixed and variable, are included in the cost of products. Also called full costing.

Question 19. What is Budgeting process and its various forms?

117

Finance

Answer:

Budgeting is a complex process, which maybe divided into the following phases: Identification of potential investment opportunities: The planning body develops estimates of future sales, which serve as the basis for setting production targets. This information is in turn helpful in identifying required investments in plant and equipments. Assembling of Investment Proposals: Investment proposals are defined by production department and other departments are usually submitted in a standardized capital investment proposal form. Investment proposals are usually classified into various categories for facilitating decision-making, budgeting and control. Decision-Making: A system of rupee gateways usually characterizes capital investment decision-making. Under this system executives are vested with the power to okay investment proposals up to certain limits. Preparation of Capital Budget and Appropriations: - Projects involving smaller outlays and which can be decided by executives at lower levels are often covered by a blanket appropriation for expeditious action. Projects involving larger outlays are included in the capital budget after necessary approvals. Implementation: Translating an investment proposal into concrete project is a complex, time-consuming and risk fraught task. Delays in implementation, which are common, can lead to substantial cost overruns. Performance Review: This is a feedback device. It is a means of comparing actual performance with projected performance. It may be conducted most appropriately when operations of the project are stabilized.

Question 20. What do you understand by Zero-based Budgeting? Answer: A method of budget review and evaluation that requires all projects and programs, new and old, to justify all resources. Each project starts the budget evaluation process without a resource commitment even if it is an ongoing project. The main idea behind ZBB is to challenge. The existence of every budgeting unit and every budget period.

Question 21. How a company can be benefited by Zero-Based Budgeting? Answer: There are plenty of reasons. Such as: i. ii. iii. iv. v. vi. Results in efficient allocation of resources as it is based on needs and benefits Drives managers to find out cost effective ways to improve operations Detects inflated budgets Useful for service department where the output is difficult to identify Increases staff motivation by providing greater initiative and responsibility in decision-making Increases communication and coordination within the organization

vii. Identifies and eliminates wastage and obsolete operations Question 22. Can you define flexible budget?

118

Frequently Asked Questions in Management

Answer:

A flexible budget is a budget, which by recognizing different cost behavior patterns, is designed to change as volume of output changes.

Question 23. Why is it important to calculate variance for a company? Answer: Variances represent deviations of actual performance from standard performance. There can be cost variances, profit variances and sales value variances. Variances can be favorable or unfavorable depending upon their impact on the profits of the organization.

Question 24. In what way Activity Based Costing differs with the Traditional Methods of Costing? Answer: Activity Based Costing (ABC) captures costs and affects accounting practices which traditional costing fail to do so as explained below. Unlike traditional costing, ABC reveals the linkages amongst activities in different departments The underlying principle of ABC is to trace the product cost whereas traditional by the need to value stocks.

In ABC, costs are accumulated for each activity as a separate cost object while in traditional costing costs is allocated based on various departments and functions. Question 25. Why is activity-based costing, so important? Answer: Cost attribution to cost units on the basis of benefits received from indirect activities, i.e. ordering, setting-up, assuring quality, etc. Activity-based costing (also called activity accounting) emphasizes links between performance of particular activities and the demands that those activities make on the organizations resources.

119

Finance

BANKING
Question 1. Answer: What are the differences between current account and saving account? Current Account: In current account, account holder can write cheques to pay bills without having to go to the financial institution or post office to buy a money order or cashier's check. He can also set up some amounts for recurring bills (such as rent or mortgage or a car payment) to be automatically withdrawn (debited) from your account, so that you don't even have to write cheques for those. Savings Account: A savings account is an interest-bearing deposit account with no stated maturity, as opposed to time deposit. You can withdraw or deposit funds into your savings account any time. It pays daily interest on your balance and is normally credited to your account every month. You can choose to receive an account statement or a passbook to keep track of all your transactions and interest earned. Question 2. Answer: Why we cross a cheque and what do you understand by it? When the cheque is crossed on a particular banker would not pay directly to the bearer of the cheque but would transfer the amount in the account of the bearer. What is the difference between balance sheet of a bank and that of a normal manufacturing company? Similar to the balance sheet of any other firm, the banks balance sheet also has assets that represent uses of funds to generate revenue for the bank and liabilities & net worth form the sources of the banks funds. However, within this framework, there are significant differences in the basic composition of the assets and the liabilities and how they contribute towards the revenue and expenses of the bank. What is the difference between cash flow and fund flow statements? The following are the main points of difference between the Cash Flow and Funds Flow Analysis: Cash Flow Statement takes into consideration only the changes in cash position whereas funds flow statement is concerned with changes in working capital position between the two balance sheet dates. Funds from operations are calculated by making Adjusted Profit and Loss Account taking into consideration all adjustments, which are made at the end of the period. Cash flow statement deals with only actual receipt and payment of income and expenses. In order to calculate cash from operations further adjustments are to be made for increases and decreases in current assets and current liabilities. Cash flow statement is more useful to the management as a tool of financial analysis in short periods as compared to funds flow statement. While preparing funds flow statement increase in current liabilities or decrease in current assets will mean decrease in working capital but in the preparation of cash flow statement increase in current liabilities or decrease in current assets will increase the cash and vice versa

Question 3. Answer:

Question 4. Answer:

120

Frequently Asked Questions in Management

Question 5. Answer:

What is Ratio analysis? Ratio analysis is a powerful tool of financial analysis. A ratio is defined as "the indicated quotient of two mathematical expressions" and as "the relationship between two or more things". In financial analysis, a ratio is used as an index or yardstick for evaluating the financial position and performance of a firm. The absolute accounting figure reported in financial statement does not provide a meaningful understanding of the performance and the financial position of the firm and the accounting figure conveys meaning when it is related to some relevant information. The relationship between two accounting figures expressed mathematically is known as "Financial Ratio". Ratios help to summarize the large quantities of financial data and to make qualitative judgment about the firms financial performance. What is the difference between debit card and credit card? A credit card can be viewed as a payment mechanism, which enables holder of the card to purchase goods (or services) without parting with immediate cash and make a onetime payment at the end of a specified period with a provision for spreading this payment over several easy installments. In this way, the cardholder manages to postpone the expenditure by usage of card availing credit from the issuer of the card. Debit card unlike the credit card is a pay now product where customer account with the issuer is immediately debited to the extent of the value of transaction.

Question 6. Answer:

Question 7. Answer:

How does RBI regulate NBFCs? The RBI controls the activities of the commercial banks, non-banking finance companies and financial institutions by virtue of powers vested under the Reserve Bank of India. The following are most significant aspects of regulatory authority of the RBI: Maintenance of the cash reserve by the scheduled commercial banks with the RBI under Section 42 of RBI Act. Issues relating to the collection and furnishing of credit information from the commercial bank. Registration and Net Owned Fund requirement Assets, Reserve funds, Disclosure of information, inspection Powers of auditors

Question 8. Answer:

What do you mean by CRR and SLR? Cash Reserve Ratio (CRR): The cash, which the banks have to maintain with RBI as a certain percentage of their demand and time liabilities. This is to ensure the safety and liquidity of the deposits with the bank. Statutory Liquidity Ratio (SLR): The amount of funds to be invested in specified assets that include GOI securities. The ratio is expressed as a percentage of the net demand and time liabilities of a bank on a reporting Friday.

Question 9.

What is Bank rate, Repo rate and dollar rate?

121

Finance

Answer:

Bank rate is the rate at which the central bank gives loans to other banks. Repo rate is the financing rate for government securities sold against repurchase agreements. Dollar Rate is the price at which dollar is brought and sold in terms of other currency. For example current rate of dollar in terms of rupees represented as $/Rs. is 40.14. The price at which dollar can purchased and sold in terms of rupees.

Question 10. What is PLR? Answer: Prime Lending Rate (PLR) is the interest rate a bank charges on loans to its most creditworthy commercial customers. It is less frequently cited as a standard for general interest-rate levels in the economy today than in the past. It is a lending rate offered to a bank's better retail customers.

Question 11. What is real interest rate and bank rate? Answer: Real interest rate is nominal interest rate less price inflation. Bank rate is the minimum lending rate of the Bank. The bank rate is an important tool because it is seen as the trendsetter for other short-term interest rates. Question 12. Explain the importance of Credit-Deposit ratio. Answer: It is the proportion of loan-assets created by banks from the deposits received. The higher the ratio, the higher the utilization of deposits for creating revenue generating loans. Credit-Deposit ratio indicates the extent of loans (credit) extended by the bank out of the deposits it receives from the public. Credit will be extended only after maintaining statutory reserves (like Cash Reserve Ratio -CRR, Statutory Liquidity RatioSLR) by the banks. Question 13. What is the relation between home loans and taxation? Answer: If the taxation policy is such that more rebates are given on interest and capital payments, the demand for home loans will surge.

Question 14. What is DSCR? Answer: DSCR is the Debt Service Recovery Ratio. It is a ratio often used by bank loan officers when making loans to perspective income property loans. It indicates the extent of income earned by an asset (loan) financed by a bank. Ideally expressed as DSCR = Net Operating Income/Total Debt Service

Question 15. How the computation of capital adequacy ratio (CAR) of banks is done?

122

Frequently Asked Questions in Management

Answer:

For computation of CAR, we need to calculate: Tier I capital Tier II capital Risk Weighted Assets (RWA)

The first Step is to compute Tier I capital: Tier I capital is the most permanent and readily available support against unexpected losses. It consists of 1. 2. 3. 4. Less: 1. 2. 3. Equity investments in subsidiaries Intangible assets Current and Accumulated Losses, if any Paid up equity capital Statutory reserves Capital reserves Other disclosed free reserves

The next step is to compute Risk Weighted Assets and to compute Tier II capital, which includes: 1. Undisclosed reserves and cumulative perpetual preference shares- Cumulative preference shares should be fully paid and should not contain clauses which permit redemption from shareholders. Revaluation Reserves (RR) - 45% of RR is only taken in calculation of tier II capital. General Provisions and Loss Reserves (GPLR) - Actual GPLR or 1.25% of Risk Weighted Assets, whichever is lower, is taken. Hybrid Debt Capital Instruments - These combine characteristics of both equity and debt. As they are more or less similar to equity, they are included in the Tier II capital. Subordinated Debts - These must be fully paid up, unsecured, subordinated to the claims of other creditors, also there should be no such clause, which permits redemption. The amount of subordinate debts to be taken as Tier II capital depends upon the maturity of debt. Subordinate Debt Instruments will be limited to 50% of Tier I capital.

2. 3. 4. 5.

It may be noted that Tier II capital cannot be more than Tier I capital. Capital Adequacy Ratio: Capital Adequacy Ratio = (Tier I capital + Tier II capital) / RWA According to the present norm, the Capital Adequacy Ratio of bank as defined earlier should be at least 10%. Question 16. How do you compare CAR of 20% and 25% of two banks?

123

Finance

Answer:

The bank having a CAR of 25% would offer a greater cushion to the customers in terms of safety as that bank is in a much advantageous position to absorb possible losses. The capital will provide a margin of safety that preferably would allow that preferably will allow intermediary to continue operations without loss of momentum.

Question 17. What is the main problem faced by nationalized banks? What may be the solution for it? Answer: Main problem faced by nationalized banks is excessive non-performing assets (NPA) and securitization of these NPA would help in solving this problem. Securitization Act has already been notified to facilitate this process.

Question 18. What are the problems of bill of discounting in India? Answer: 1. There is no secondary market. 2. They are not widely accepted.

Question 19. Fear of NPAs, enquiries on bad loans and capital adequacy constraints make banks credit averse. Explain. Answer: Over the past few years, a combination of circumstances has made banks averse to taking on additional credit exposure. This is especially so in case of new projects where the promoter does not have a track record. The introduction of globally accepted norms of Asset Classification resulted in high level of Non-Performing Assets showing up in the books of banks, which had to be written off affecting profitability and eroding capital. The erosion of capital meant that banks were in danger of not meeting the newly introduced norm of 8% capital adequacy. Not achieving the 8% norm would affect credibility of the bank and possibility of accessing the capital markets for equity or debt. In a number of public sector banks that faced massive erosion in capital, enquiries were instituted against officials who sanctioned the bad loans thereby making bankers more cautious of taking any kind of credit decisions. In such circumstances banks increasingly resorted to investing in risk-free government debt and avoided taking on industrial credit. The problem has been more acute in the small-scale sector. The sector had witnessed a high level of directed lending in the past decade most of which turned bad thereby putting off the banks from further credit to the sector. Even otherwise, the adverse impact of liberalization has been felt more in the SME sector than among larger industries. Question 20. What is asset backed pass through Process? Answer: The Asset Backed through process is floating the Asset Backed Security (ABS). An ABS is constructed by packaging together a group of securities and then issuing a new security whose purchaser has a claim against the cash flows generated by the original package. Eg: Securitizing the future rental income of a say a shopping complex. The owner of the complex will be financed by a banker against the future rentals. The future rentals will be divided into assets and financed by the banker at the prevailing interest rates. Question 21. How Asset backed pass through can help to increase the value of the company?

124

Frequently Asked Questions in Management

Answer:

As the ABSs are off balance sheet items, these cannot increase the value of the firm

Question 22. What do you understand by NPAs? Answer: Non-performing asset (NPA) is any asset that is not effectively producing income. All the loans given by a bank are its assets. When such assets do not yield any interest income and result in capital erosion, it is called a non-performing asset. If the loan installments and interest are not forth coming, then the loan is classified as NPA after a specified period of time. As per the present guidelines if the interest has remained due for past two quarters, the loan or credit facility is classified as NPA. Question 23. What is meant by Securitization? Answer: It is a method of imparting liquidity into the system by repackaging the future cash flows of highly illiquid assets into negotiable securities and issued to the investors.

Question 24. Explain the term SPV? Answer: Transfer of assets in securitization by the originator to a person (company or a trust) specially created for the purpose is called Special Purpose Vehicle (SPV). Special Purpose Vehicle (SPV) is a separate entity formed exclusively for charting a particular deal and providing funds to originator (person holding assets). SPV maybe formed as a company under the Companies Act or a trust formed under the Indian Trust Act.

Question 25. What do you understand by Doubtful Assets? Answer: Non-performing assets, which have remained as sub-standard assets for more than 18 months, will be classified as doubtful assets. Provisioning has to be made for such assets.

Question 26. What is the utility of Magnetic Ink Character Recognition (MICR) Clearing in modern banking? Answer: A clearing system wherein the instruments like Cheques and drafts are sorted mechanically by coders by scanning the instruments on which the identification numbers are written in magnetic ink.

Question 27. What is the process involved in EFT (Electronic Funds Transfer) Answer: It is a payment transaction carried out between two parties without the use of cash or paper to affect the transaction. The payment is affected by communication of electronically transmitted message to the bank by the customer of the bank or between banks.

Question 28. What is meant by Electronic Credit Transfers? Answer: This facilitates a person to make payments through a bank to different payees of the same bank or those of other banks electronically. The credit transfers take place with the substitution of obligations without there being any actual movement of funds.

Question 29. What is the meant by Guarantee in the context of banking?

125

Finance

Answer:

It is a contract to perform the promise or discharge the liability of a third person in case of a default. It can be oral or written.

Question 30. What do you understand by Financial Guarantee? Answer: It involves an undertaking by a bank to pay certain amount of money in the event of a failure on the part of the banks client to pay the same.

Question 31. How does a bank act as Performance Guarantee? Answer: Type of contract, where the bank undertakes to compensate for any loss suffered consequent to poor performance/non-performance of the principal debtor.

Question 32. What do you understand by Hypothecation? Answer: It is a way or creating charge against the security of movable assets. Hypothecation is not defined by Law.

Question 33. What is the role of Pledge in securing a debt? Answer: It is a contract where under deposit of goods is made a security for a debt and the right to property vests in the pawnee so far as it is necessary to secure the debt.

Question 34. When is a Lien required? Answer: In the absence of contract to the contrary, bankers, factors, wharfingers, attorneys and policy brokers may retain as a security for a general balance of account, any goods bailed to them. Such right to retain is known as lien.

Question 35. What do you mean by Subrogation in a contract? Answer: Doctrine of subrogation refers to the rights of a guarantor to acquire the rights of a creditor once the guarantor fulfills the obligation under a contract.

Question 36. What a Prepayment Risk represents? Answer: It is the risk that arises due to the prepayment of loans.

Question 37. What is Prime Rate for a bank? Answer: It is the benchmark rate that the bank charges from its borrowers based on the risk level. Examples of the benchmark rates are the LIBOR prevailing in the UK market and the Prime Rate of the US market.

Question 38. What is the difference between fixed and floating rates? Answer: The future cash flows in terms of interest rate payment is fixed at the time of the contract is called fixed rates. For example, the interest rate for bond is 12% p.a. Every year the bondholder will get a interest rate of Rs. 12 calculated on the face value of a bond i.e. Rs. 100 When the future expected cash flows in terms of interest rate payments is dependent upon some benchmark rate plus certain rate, the bench mark rate being fluctuating

126

Frequently Asked Questions in Management

depending upon the conditions of the economy, it is called floating rates. For example the interest rate of bond is LIBOR+3/4% p.a. Now on the date of payment of interest rate the amount of interest would depend on the rate of LIBOR on that date. Question 39. What is the benchmark for floating rate? Answer: Benchmark for floating rate is decided by the company issuing the bond. However, the most common benchmark for floating rates notes is LIBOR London Inter-bank Offered Rate.

Question 40. How does LIBOR (London Inter Bank Offered Rate) affect banks? Answer: It is the base rate at which major banks in London lend to each other, and the standard for pricing in international financial contracts.

Question 41. How PLR (Prime Lending Rate) affects economy? Answer: It is the benchmark rate that the Indian banks/FIs set to extend credit to the most creditworthy customers. The rate is arrived at based on the interest rate structure prevailing in the market.

Question 42. What relation Prime-plus Pricing rate has got with PLR? Answer: A lending rate that is obtained by adding a mark-up percentage to the PLR based on the credit quality of the customer.

Question 43. What do you understand by Rate Adjusted Gap Model? Answer: It is an interest rate risk management model that is based on the gap between the rate sensitive assets and rate sensitive liabilities after adjusting them by assigning weights based on the estimated change in the rate for the different assets/liabilities for a given change in the interest rates.

Question 44. What does a Rate Sensitive Gap represents? Answer: The difference between Rate Sensitive Assets (RSAs) and Rate Sensitive Liabilities (RSLs) over a particular planning horizon and is measured as (RSAs RSLs).

Question 45. Can you distinguish transaction accounts from other accounts? Answer: A transaction accounts facilitate the depositor to make payments or transfer funds to a third party by means of a negotiable instrument/telephone order. Current accounts and savings bank accounts are transaction accounts.

Question 46. How do you differentiate Float Funds from other funds? Answer: They represent funds, which are generally in transit and comprise of funds received on account of demand drafts, bankers cheques, etc.

Question 47. What is an Overdraft?

127

Finance

Answer:

It is an operating account from which the borrower can withdraw funds as and when required to a predetermined credit limit.

Question 48. What is Working Capital Demand Loan (WCDL)? Answer: It is a short-term loan, which has a minimum repayment period of 3 months and is generally used to fund the working capital requirements of firms.

Question 49. What do you know about consortium finance? Answer: Consortium Finance means multiple lending; obligatory consortium arrangement exists only beyond the threshold limit of Rs.50 crore. The rules for such consortia were liberalised by the RBI in October 1996. The ground rules of consortium arrangement, namely, number of participating banks, minimum share of each bank, entry/exit of a bank from the consortium, grant of any facility to the borrower by a non-member bank, etc., could be framed by the participating banks.

Question 50. What do you mean by Lead Bank? Answer: The main bank involved in loan syndication, when a loan is extended by a syndicate of banks.

Question 51. What is a loro account? Answer: A banks account with a foreign correspondent bank, from a third partys point of view.

Question 52. What is a nostro account? Answer: A banks account with a correspondent bank located in a foreign country.

Question 53. What do you mean by Non-Banking Financial Institutions? Answer: Non-banking Financial Institutions carry out financing activities but their resources are not directly obtained from the savers as debt. Instead, these Institutions mobilize the public savings for rendering other financial services including investment. All such Institutions are financial intermediaries and when they lend, they are known as NonBanking Financial Institutions (NBFIs) or Investment Institutions. Apart from these NBFIs, another part of Indian financial system consists of a large number of privately owned, decentralized, and relatively small-sized financial intermediaries, which provide financial services of various types. Such entities are known as "Non Banking Financial Companies (NBFCs)". The RBI from time to time keeping in view the emerging situation reviews the regulations governing the NBFCs. Question 54. What do you understand by BCR and NBCR? Answer: The benefit cost ratio (BCR) is defined as follows: BCR= PV/I Where BCR= Benefit-cost ratio

128

Frequently Asked Questions in Management

PV= Present Value of future cash flows I = initial investment A variant of benefit-cost ratio is net benefit cost ratio (NBCR), which is defined as: NBCR = NPV-I = PV-I/I = BCR-1 BCR and NBCR are used to estimate the viability of a particular project. The decision rule based on BCR and NBCR would be as follows: BCR>1 BCR<1 NBCR>0 NBCR<0 Accept the project Reject the project

Question 55. While processing a home loan, how does one carry out credit appraisal? Answer: When processing a home loan, the following factors have to be kept in mind: Type and extent of collaterals Statutory limits on home loan Credit files: this file contains the detail regarding the borrower and the tenor of the loan. This is essential especially since there may always be a probability of default or a change in the risk-return profile of the customer. Monitoring Mechanism Loan-Deposit Ratio Consumer Laws and Regulations Question 56. Can you briefly tell us how you carry out a credit risk analysis of your customer? Answer: When there is a counter-party risk failure in performing the repayment on due date, it give rise to low quality assets which in turn lead to Credit Risk. Effective management of credit risk involves the following key principles: Evaluation Pricing Monitoring Credit risk management policy should cautiously evaluate and sanction the proposal by appropriately pricing it. Scrutinizing all credit information relating to the customer should be carefully estimated and interpreted. The evaluation and pricing decision should be followed with periodic review of the account and the credit rating of the borrower.

129

Finance

FINANCIAL MARKET
Question 1. Answer: Why government formulates Monetary Policy? It is a policy of monetary authority that influences money supply and/or interest rates in an attempt to achieve government objectives. What do you mean by Fiscal Policy? It is a policy of government that influences the level of economic activity through government expenditure and resource mobilization. Explain in brief the role of Money Market in an economy? It is a wholesale debt market dealing in short-term instruments. Funds are also available in this market for periods ranging from overnight to one year. What is Call Money? Call money is money in the form of loans and debt. It is adjusted daily for interest rate charges, and can be called at any time. What can be the reasons for fluctuations in call rates? The call money rates fluctuate due to short-term liquidity mismatches. If demand for short term funds goes up may be due to advance tax payments, for meeting CRR requirements or for making funding arrangements for forthcoming issue of government securities, call rates go up and vice versa How are the call rates determined? Why are sometimes high on Fridays? Who are the participants in call money market? The rate of interest paid on call money is called as call rate. Call rates are determined on the supply and demand of the call money in the market. Banks are the major participants in call money market. According to the RBI norms, banks should show the specified Cash Reserve Ratio (CRR) and Statutory Liquid Ratio (SLR) for every fortnight. As the end day of a fortnight is Friday, there will be heavy demand for the call money. As the demand increases call rates soar on Fridays. Other participants in the call money market are scheduled commercial banks, non-scheduled commercial banks, foreign banks, state, district and urban, cooperative banks, Discount Finance House of India (DFHI) and Securities Trading Corporation of India (STCI). What is fungibility? Fungibility is the interchangeability of listed options, futures contracts, and other instruments dependent upon identical terms.

Question 2. Answer:

Question 3. Answer:

Question 4. Answer:

Question 5. Answer:

Question 6. Answer:

Question 7. Answer:

130

Frequently Asked Questions in Management

Question 8. Answer:

What do you understand by Financial Intermediation? Financial Intermediation is the process of transferring the funds from savers to users, which is facilitated by institutions like banks and NBFCs. What is a Financial Intermediary (FI)? Financial intermediaries assist in the transfer of savings from economic units/ individuals with excess money to those that need capital for investments. Financial intermediary includes Banks, Investment Companies, Insurance Companies, Develop-ment Financial Institutions, Non-Banking Finance Companies, Mutual Funds, etc.

Question 9. Answer:

Question 10. What is the role of Financial Intermediaries? Answer: These are the entities that perform the role of financial intermediation and include banks, financial institutions, NBFCs, insurance companies, investment companies, mutual funds, pension funds, etc.

Question 11. What do you mean by Financial Disintermediation? Answer: It is a phase in the economy where the role of a financial intermediary in facilitating the funds transfer from savers to the users is reduced. The business profile of a financial intermediary will see a shift from fund-based activities to fee-based activities.

Question 12. Explain in brief NBFC (Non-Banking Finance Company)? Answer: It is a financial intermediary which is not a commercial bank accepting public deposits and extending both fee-based and fund-based financial services.

Question 13. What Net Interest Income (NII) mean to you? Answer: It represents spread and is the excess of the interest income over interest expense of a financial intermediary.

Question 14. What Net Interest Margin (NIM) represents to a financial intermediary? Answer: It is the ratio of net interest income to the total assets of the financial intermediary and is a measure of its profitability.

Question 15. What do you understand by net-interest expenses? Answer: The operating expenses of a financial intermediary that include salaries, wages, rent and other administration expenses.

Question 16. How is non-interest income different from other incomes? Answer: Income earned by a financial intermediary in the form of fee, commission, service charges, etc. but not in the form of interest.

131

Finance

Question 17. How does Interest Rate Risk affect market? Answer: It is the risk that affects the income and/or the market value of financial assets due to the interest rate fluctuations.

Question 18. What are the sources of income to a credit card issuing bank? Answer: Annual charges from the cardholder, interest on unpaid amount at the end of the month and commission charged from the member establishments.

Question 19. What is the role of Merchant Establishments (MEs)? Answer: MEs are establishments enlisted by the plastic money issuer who accepts valid credit cards towards payment for the goods sold or services rendered by them in lieu of cash. MEs include retail outlets, departmental stores, restaurants, hotels, hospitals, petrol bunks, etc.

132

Frequently Asked Questions in Management

CAPITAL MARKET
Question 1. Answer: What is a Secondary Market? It is a place where securities already issued are traded. It is different from the primary market wherein the issuer sells securities directly to the investor. How is a Primary Market different from Secondary Market? A market for new issues of shares, debentures and bonds, where investors apply directly to the issuer for allotment and pay application money to the issuers account. It is different from the secondary market where investors trade in shares on the stock exchange through brokers. What is a Bourse in stock market? A French stock exchange (from the French word bourse, meaning purse). The Bourse originally referred to the stock exchange in Paris, but later all stock exchanges came to be known by this name. Members of the Paris Bourse are known as agents de change. What do you know about merchant banking? Corporates raise capital by issuing securities in the market. Merchant bankers act as intermediaries between the issuers of capital and the ultimate investors who purchase these securities. Merchant banking can be broadly defined as financial intermediation that matches the entities that need capital and those have capital. Merchant banking facilitates the process of flow of capital in the market. What is the role of a Broker in stock market? A stock exchange member licensed to buy or sell shares on his own or on his clients behalf. Commission brokers just execute buy or sell orders against a commission, whereas full service brokers offer facilities such as offering investment advice, safe keeping of securities, managing portfolios, etc. In how many ways one can trade stock? There are two ways through which investors can trade stocks, one is through a broker and other is through online trading. Online trading is a new concept in Indian financial markets. In Online trading investor can purchase and sell a stock through the Internet. Many Internet sites are providing facility to trade through the Net. The commissions charged by these sites are quite low as compared to the commission to the brokers. Who are Jobbers? The specialized members of a stock exchange who stand ready to buy and sell shares in which they specialize are called jobbers. A jobber quotes his bid price (the price at which he is willing to buy) and ask price (the price at which he is willing to sell). The difference between the two is known as the jobber spread.

Question 2. Answer:

Question 3. Answer:

Question 4. Answer:

Question 5. Answer:

Question 6. Answer:

Question 7. Answer:

133

Finance

Question 8. Answer:

What changes have taken place in the equity markets last one year? Some of the major changes that have taken place in the equity market from last one year are: -T+2 Rolling settlement from April 2003 The Ministry of Finance has decided to scrap the policy on mandatory listing by companies in the regional stock exchange where the registered office of the company is located. This has reduced the importance of regional exchanges at a time when turnover was already decreasing.

Question 9. Answer:

What is a Bear situation in a stock market? A dealer on a stock exchange, currency market, or commodity market who expects prices to fall. A bear market is one in which prices are falling or expected to fall, i.e. a market in which a dealer is more likely to sell securities, currency, or goods than to buy them. A bear may even sell securities, currency or goods without having them. This is known as selling short or establishing a bear position. The bear hopes to close (or cover) a short position by buying at a lower price the securities, currency or goods previously sold. The difference between the purchase price and the original sale price represents the successful bears profit. A concerted attempt to force prices down by one or more bears by sustained selling is called a bear raid. A successful bear raid will produce a sharply falling market, known as bear slide. In a bear squeeze, sellers force prices up against someone known to have a bear position to cover.

Question 10. What is a Bull situation? Answer: A person who holds an optimistic view of the market and expects the prices to rise. A bull with a long position hopes to sell his purchases at a higher price after the market has risen.

Question 11. What are the benefits of valuation? Answer: When the market is bullish, the premium rate is high. If the valuations are steep (High Interest), the investor can short sell the index and cover the same at lower levels. When the market is bearish, the investor can go long on the index.

Question 12. Why you need a Warrant? Answer: A warrant is a tradable instrument giving its holder the right to purchase specific security/securities from the issuer, subject to certain specific conditions.

Question 13. What is a Bought-Out Deal (BOD)? Answer: BOD is a process whereby an investor or a group buys-out a significant portion of an unlisted company with a view to take it public within an agreed time frame. At the appropriate time the shares are offered to the public through a public ISSUE or OTCEI.

Question 14. What does an Odd Lot signify? Answer: An odd lot is a small number of shares one may get as a result of a bonus or rights issue or on the conversion of debentures. They are not in marketable lots (there is a

134

Frequently Asked Questions in Management

minimum quantum of shares that is usually traded which is known as marketable lot 50 or 100 shares). They are usually difficult to sell. On Saturdays in the major stock markets odd lots are traded. The price is, however, lower than the normal price of the share (about 10%). The small investors more often than not buy fewer than 100 shares of a given stock i.e. an odd lot and such buyers and sellers are called odd lotters. Question 15. What do you mean by Private Placement in capital Market? Answer: A method of raising capital in which companies directly sell their securities to a limited number of investors. Normally, the minimum size of an investment in a private placement is higher.

Question 16. What do you understand by Underwriting contract? Answer: Underwriting is a contract wherein the underwriter (usually a merchant bank, broker or a financial institution) agrees to purchase a certain number of shares in the event of under subscription of the issue. The consideration paid to the underwriter is called underwriting commission.

Question 17. What is a Contango in Security market? Answer: The consideration or interest charge paid by the buyer to carry over a transaction from one settlement period to the next.

Question 18. What is Share Splitting? Answer: Share Splitting is division of the share capital of a company into smaller units. The effect of a share split is the same as a scrip issue although the technicalities differ. Share splits are usually carried out when the existing shares reach such a high price that trading in them becomes difficult.

Question 19. When a Split of shares occur? Answer: This occurs when the shares are divided into shares of smaller denomination. Rs.100 shares may be split into ten Rs.10 shares. It usually happens when the price becomes unwieldy.

Question 20. What do you understand by Short Position? Answer: Shares which a person has sold short, by delivering borrowed certificates, but which he has not yet covered by actually buying shares to repay the loan as on a particular date.

Question 21. What do you understand by Speculation? Answer: An activity, in which a person assumes high risks, not bothered about the safety of his invested principal, to achieve large capital gains. The time span in which the gain is sought to be made is usually very short.

Question 22. What are the advantages to companies for listing in stock exchanges? Answer: There are many advantages for companies to get listed on a stock exchange. The major advantages are:

135

Finance

Increased marketability for securities Wider stockholder base Stock options and stock purchase gain known value, creating an incentive for employees to remain with the company For listed company financing is easier In the event of a merger, shareholders receive greater value, since shares often trade at a premium to their book value and listed securities may more easily be used as a currency in effecting a take-over. Question 23. What is delisting? What are its implications for investors? Answer: Delisting is removing a particular scrip from the trading activities on an exchange. The delisting decision has to be taken by the regulators of the exchange. Delisting gives the negative impression of the scrip to the investor. Hence, generally investors do not show interest in delisted scrip.

Question 24. What do you understand by Dematerialization? Answer: The process of converting securities from their physical paper form to computerized book entry form. In a dematerialized environment, there is no such thing as a share certificate.

Question 25. Explain the process of dematerialization Answer: Dematerialisation aims at paperless securities. When the company has to get its shares dematerialized, it has to send the shares in physical form to a custodian who will convert them into dematerialised form. The investors will have to open their account with depository participants, where the shares will be debited / credited to their accounts.

Question 26. What is a Grey Market with reference to Stock Market? Answer: The unofficial public trading in the shares of a company prior to its listing.

Question 27. Is "badla" trading like derivatives trading? Answer: No. Badla is a mechanism to avoid the discipline of a spot market; to do trades on the spot market but not actually do settlement. The "carry forward" activities are mixed together with the spot market. A well functioning spot market has no possibility of carryforward. Derivatives trades take place distinctly from the spot market. The spot price is separately observed from the derivative price. A modern financial system consists of a spot market which is a genuine spot market, and a derivatives market which is separate from the spot market.

Question 28. Have you heard of the term Backwardation with reference to stock market? Answer: It is also known as Ulta Badla or Undha Badla. The payment to be made by a seller to a buyer for the loan of securities for which the seller wishes to defer deliveries. When a bear sells in anticipation of a fall in prices in the immediate future, but the fall does not happen within the settlement period, he has to have his sales carried over to the next accounting period on payment of Undha Badla or backwardation charges to the buyer.

136

Frequently Asked Questions in Management

Question 29. What happens in an Insider Trading? Answer: An illegal activity in which persons in a company have unpublished price sensitive information, such as expansion plans, financial results, takeover bids, etc. takes advantage of such information to make a profit on the stock exchange by buying or selling shares.

Question 30. Which transactions are called Kerb Dealings? Answer: Transactions done among members after the closing of the official trading hours, even though such trading among the members is not strictly legal. Such an unofficial trading carried out outside the stock exchange premises is known as Kerb Trading. Kerb trading accounts for a sizeable percentage of total trading in the Indian stock exchanges.

Question 31. What do you know about online trading? Answer: In online trading investor can place a buy or a sell trade on computer. The system cheques the funds and shares available in their bank & the demat account respectively and executes the trade on the exchange online. The bank account and demat account is automatically debited or credited. Online trading is carried out on the Internet as the investor can directly assess the stocks without the intermediary. Therefore the commission is very low when compared with the commission paid to the brokers. There are main online trading sites in India including icicidirect.com, 5paise.com etc.

Question 32. What do you understand by Market Capitalization? Answer: The total market value at the current prices of the total number of equity shares issued by a company.

Question 33. What is technical analysis? Answer: Technical Analysis is basically the study of Price Chart, undertaken to get an idea about future price action of any traded stock. A Price Chart plots the quotes of a stock traded on a stock market. All past\present\future news relating to a stock, together with investors' opinion about it, determines the price of the stock on the trading floor. The "Price" discounts everything. Therefore, study of anything else is unnecessary. Technical Analysis comprise of various techniques to study such price action over a period.

Question 34. What is fundamental analysis? Answer: Fundamental Analysis is a conservative and non-speculative approach based on the "Fundamentals". A fundamentalist is not swept by what is happening in Dalal Street as he looks at a three dimensional analysis. The Economy The Industry The Company The Economy Analysis. Question 35. What is the process involved in Technical Analysis?

137

Finance

Answer:

A method of forecasting share price movements based on the study of price charts on the assumption that share price trends are repetitive, that seems investor psychology follows certain pattern, what is seen to have happened before is likely to be repeated. The technical analyst is only concerned about investor and price behavior.

Question 36. What do you understand by Efficient Market Hypothesis? Answer: The hypothesis that holds that the financial market is in possession of all available information influencing the price of a share or financial security, which results in perfect competition in the financial market. Perfect competition in the stock market context implies that buyers and sellers have perfect knowledge and neither are in possession of any information unknown to others.

Question 37. What do you mean by Random Walk? Answer: This hypothesis states that in a perfectly competitive market, investors take stock of all the factors influencing share price movements and hence further changes are caused randomly and no systematic prediction can be made. The risk of random walk can be reduced by diversification of ones portfolio.

Question 38. What is Monte Carlo Simulation? Answer: When we use the word simulation, we refer to any analytical method meant to imitate a real-life system, especially when other analyses are too mathematically complex or too difficult to reproduce. Without the aid of simulation, a spreadsheet model will only reveal a single outcome, generally the most likely or average scenario. Spreadsheet risk analysis uses both a spreadsheet model and simulation to automatically analyze the effect of varying inputs on outputs of the modeled system. One type of spreadsheet simulation is Monte Carlo simulation, which randomly generates values for uncertain variables to simulate a model. Monte Carlo simulation was named for Monte Carlo, Monaco, where the primary attractions are casinos containing games of chance. Games of chance such as roulette wheels, dice, and slot machines, exhibit random behavior. The random behavior in games of chance is similar to how Monte Carlo simulation selects variable values at random to simulate a model. When you roll a die, you know that a 1, 2, 3, 4, 5, or 6 will come up, but you don't know which for any particular roll. It's the same with the variables that have a known range of values but an uncertain value for any particular time or event (e.g. interest rates, staffing needs, stock prices, inventory, phone calls per minute). Question 39. How would you appraise a company in the Pharma Industry? Answer: For a company analysis of Pharma company we have to focus on two aspects: Non-financial parameters: Expenditure on Research and Development:

138

Frequently Asked Questions in Management

Government Policy regarding pricing of the products Distribution Network Competition Product Range Quality of Management Availability of Inputs Financial Parameters: Different Financial Ratio: Profitability, liquidity ratios, Leverage and Return on Equity Question 40. Name some economic indicators and their possible impact on the stock market. Answer: In the table below are some economic indicators and their possible impact on the stock market are given in a nut shell. Economic indicators 1. GNP -Growth -Decline 2. Price Conditions - Stable - Inflation 3. 4. Economy - Boom - Recession -Favourable -Unfavourable -Favourable -Unfavourable -Favourable -Unfavourable Impact on the stock market -Favourable -Unfavourable

Housing Construction Activity - Increase in activity - Decrease in Activity Employment - Increase - Decrease

5.

-Favourable -Unfavourable - Favourable under inflation - Unfavourable under deflation -Favourable -Unfavourable - Favourable under inflation - Unfavourable under deflation -Favourable -Unfavourable

6. 7.

Accumulation of Inventories Personal Disposable Income - Increase - Decrease Personal Savings Interest Rates - low - high

8. 9.

139

Finance

10.

Balance of trade - Positive - Negative

-Favourable -Unfavourable

11.

Strength of the Rupee in Forex market - Strong -Favourable - Weak -Unfavourable Corporate Taxation (Direct & Indirect - Low -Favourable - High -Unfavourable

12.

Question 41. What factors do you think influences the market prices? Answer: The Industry Analysis Every industry has to go through a life cycle with four distinct phases i) Pioneering Stage ii) Expansion (growth) Stage iii) Stagnation (mature) Stage iv) Decline Stage These phases are dynamic for each industry. You as an investor is advised to invest in an industry that is either in a pioneering stage or in its expansion (growth) stage. Its advisable to quickly get out of industries which are in the stagnation stage prior to its lapse into the decline stage. The particular phase or stage of an industry can be determined in terms of sales, profitability and their growth rates amongst other factors. The Company Analysis There may be situations where the industry is very attractive but a few companies within it might not be doing all that well; similarly there may be one or two companies, which may be doing exceedingly well while the rest of the companies in the industry might be in doldrums. You as an investor will have to consider both the financial and non-financial aspects so as to form a qualitative impression about a company. Some of the factors are: History of the company and line of business Product portfolio's strength Market Share Top Management Intrinsic values like patents and trademarks held Foreign Collaboration, its need and availability for future Quality of competition in the market, present and future Future business plans and projects Tags - Like Blue Chips, Market Cap - low, medium and big caps Level of trading of the company's listed scripts EPS, its growth and rating vis--vis other companies in the industry. P/E ratio

140

Frequently Asked Questions in Management

Growth in sales, dividend and bottom line

Question 42. What a Moving Average indicates? Answer: An average of share prices for specified periods showing trends of price movements, rather than daily fluctuations. A monthly moving average will take a months prices till yesterday and for tomorrows average it will drop the earliest day and include today in its place.

Question 43. What is risk? Answer: Risk reflects the chance that the actual return on an investment may vary from the expected return. One way to measure risk is to calculate the variance and standard deviation of the distribution of returns.

Question 44. What is Sharpes Ratio and Treynor ratio? Answer: Sharpes Ratio - This performance measurement parameter is the most widely used. The higher the value the better the portfolios risk return profile is said to be. Treynors Ratio - This measure takes into account the systematic risk (or beta) and the average return when assessing the overall risk adjusted return of the portfolio. Question 45. What do you understand by Systematic Risk? Answer: An investment risk which applies to all securities of the same class, which cannot be avoided by diversifying ones portfolio. Economic, social or political factors will cause price fluctuations of all shares alike. Hence, the prices of shares in the market tend to rise and fall together.

Question 46. How do you hedge against risk? Answer: While risk can be reduced by diversifying, hedging means identifying two exactly correlated assets as far as returns are concerned. On can hedge the risk by buying one of the assets while simultaneously selling the others. While complete hedging may not be possible, sometimes residual risk remains. Nevertheless, hedging is a powerful tool in reducing risk.

Question 47. How do futures helps in hedging risk? Answer: While hedging through futures, if investor wants to acquire an asset in future, investor can buy a future contract so that he becomes immune to any price rise in future. On the other hand if the investor is planning to dispose of an asset, investor can sell a future contract with the sole aim of getting immunity against falling prices. In the first case it is referred to as a long-hedge and in the second case it is referred as short-hedge.

Question 48. Explain standard deviation, variance and explain the logic between the two. Answer: Standard deviation of returns measures the extent of deviation of returns from the average value of returns. It is the square root of the average of squares of deviations of the observed returns from their expected value of returns.

141

Finance

The square of standard deviation is called variance. Thus, variance of a security returns is the average value of the squares of the deviations of the observed returns from the expected value of returns. Question 49. What do you understand by portfolio risk? Answer: Most investors do not hold stocks in isolation. The investment is spread across various stocks in different industries and countries. In other words investors hold a portfolio of several stocks. Portfolio risk is the total risk associated with holding all the stocks in one basket and rather than an individual stock. Statistically speaking, the portfolio risk is measured by using the weight of each asset in the portfolio, their standard deviations and correlation and covariance between the return of the securities. For example, variance on a Two-Asset Portfolio can be calculated as follows:

Where

!12 = the correlation coefficient between the returns on stocks 1 and 2, "12 = the covariance between the returns on stocks 1 and 2, "1 = the standard deviation on stock 1, and "2 = the standard deviation on stock 2.
W1= weight of Asset 1 W2=weight of asset 2 Question 50. Tell us something about Portfolio Theory? Answer: Overall investment strategy that seeks to construct an optimal portfolio by considering the relationship between risk and return, especially as measured by alpha, beta, and Rsquared. This theory recommends that the risk of a particular stock should not be looked at on a stand-alone basis, but rather in relation to how that particular stock's price varies in relation to the variation in price of the market portfolio. The theory goes on to state that given an investor's preferred level of risk, a particular portfolio can be constructed that maximizes expected return for that level of risk, also called modern investment theory.

Question 51. Who is a Portfolio Manager and what are his roles? State in brief. Answer: A professional who manages investment portfolios with the objectives of profitability, growth and risk minimization. He is expected to manage the investors assets prudently and chooses particular investment avenues, with a view to maximize profits.

Question 52. What is involved in forecasting Nifty? Answer: Nifty is a well-diversified portfolio of companies that make up 54% of the market capitalization of India. The diversification inside Nifty serves to "cancel out" influences of individual companies or industries. Hence Nifty, as a whole, reflects the overall prospects of Indias corporate sector and Indias economy. Nifty moves with events that impact

142

Frequently Asked Questions in Management

Indias economy. These include politics, macroeconomic policy announcements, interest rates, money supply and budgets, shocks from overseas, etc. Thomas & Shah (1999) offer some time-series econometrics applied to Nifty. Question 53. Can you explain in brief the role of Depository? Answer: An institution which holds the security in electronic forms on behalf of the investor.

Question 54. How is Overseas Depository Bank different from other Banks? Answer: Unlike other banks, Overseas Depository Bank is one that is authorized by the issuing company to issue Global Depository Receipts against the issue of Foreign Currency Convertible Bonds or ordinary shares of the issuing company.

Question 55. What is the role of Mutual Funds Company? Answer: Investment management entity, which collects money from shareholders and invest in a large variety of securities like shares, debentures, bonds set up for a limited period, or with no winding-up date. The investors thus have the advantage of owning a truly diversified portfolio, which offers attractive annual dividends and a reasonable price appreciation with minimum risk involved.

Question 56. Tell us five features of debt funds. Answer: Five features of debt funds are: Provide fixed returns for those who desire safety. Price of bond funds changes with changing interest rates. These funds invest in corporate bonds or government-backed mortgage securities that have a fixed rate of return. Fixed income funds primarily look to provide current income consistent with the preservation of capital. Within the fixed-income category, funds vary greatly in their stability of principal and in their dividend yields.

Question 57. What are the functions of AMC? Answer: Three major functions of AMC are: Taking investment decisions and making investments of the funds in primary or secondary markets. Realize fund position by taking account of all the receivables and realizations. Maintaining proper accounting information for pricing the units and arriving at Net Assets Value (NAV), the information about the listed schemes and transaction of units in the secondary market. Investors in mutual funds need to assess the profile of the Asset Management Company (AMC) before entering the fund.

143

Finance

Question 58. What is AMFI? Answer: The Association of Mutual Funds in India (AMFI) is dedicated to developing the Indian Mutual Fund Industry on professional, healthy and ethical lines and to enhance & maintain standards in all areas with a view to protecting and promoting the interests of mutual funds & their unit holders.

Question 59. Can we trade mutual funds electronically? Answer: Yes, we can trade mutual funds electronically. However the scope is limited in India. There are very few sites, which provide the facility of trading mutual funds electronically; these are icicidirect.com, 5 paise.com.

Question 60. What is meant by Alpha of a stock? Answer: Alpha of a stock is its abnormal returns.

Question 61. What does beta of a stock signify? Answer: It represents the sensitivity of a stocks return to the changes in market returns.

Question 62. What is the significance of beta? Do you think that beta captures the whole risk of the security? Answer: Beta measures the volatility of a securitys return relative to the return of a broad-based market portfolio. The beta captures only non-diversifiable (systematic) risk of an investment/security/asset.

Question 63. What is CAPM (Capital Asset Pricing Model)? Answer: The Capital Asset Pricing Model (CAPM) reflects the market for different financial assets. The model suggests that asset prices will adjust to achieve the precise return, and to compensate investors for the risk of the asset, when it is held with a perfectly diversified portfolio.

Question 64. Tell us something about the implication of CAPM Model Answer: CAPM model is based on certain assumption, which make this model limited in its scope and usage. The assumptions and their implications are listed below: Investors are risk averse individuals who maximize the expected utility of their end of period, wealth. Implication: The model is a one period model. Investors have homogenous expectations (beliefs) about asset returns. Implication: all investors perceive identical opportunity sets. That is, everyone has the same information at the same time. This is not true in real world. All assets are perfectly divisible and priced in a perfectly competitive market. Implication: e.g. human capital is non-existing (it is not divisible and it cant be owned as an asset).

144

Frequently Asked Questions in Management

Asset markets are frictionless and information is costless and simultaneously available to all investors. Implication: the borrowing rate equals lending rate. Question 65. How many types of risks are present in the market? How we can measure it? Answer: Risks can be divided into systematic and unsystematic risks. The total risk of a security/portfolio/asset can be calculated by using standard deviation and coefficient of variation.

Question 66. What is the listing requirement in Nasdaq? Answer: NASDAQ is an electronic stock market run by the National Association of Securities Dealers in the United States. It is divided into two sections, the NASDAQ National Market and the NASDAQ Small Cap market. To be listed on the NASDAQ National Market, a company requires either stockholders equity of at least US $15 million or, in the alternative, either a market capitalization of at least US $75 million, total assets of US $75 million or total revenue of US $75 million. It must also have a public float of at least 1.1 million shares, defined as the total number of shares outstanding less any shares held by officers, directors or beneficial owners of ten percent or more. To be listed on the NASDAQ Small Cap Market, a company must have either stockholders equity of US $5 million, a market capitalization of US $50 million or net income of US $750,000, as well as a public float of one million shares. NASDAQ also operates the American Stock Exchange for even smaller issuers. Listing requirements include either pre-tax income of US $750,000 and a public float of 500,000 shares (or one million, depending on the applicable standard) having an aggregate market value of at least US $3 million, or a two-year operating history and a public float having an aggregate market value of at least US $15 million. In both cases, stockholders equity must be at least US $4 million.

145

Finance

DEBT MARKET
Question 1. Answer: What is bill and bond? Bill A bill of exchange is a kind of cheque or promissory note. It is a written order by one person directing another to pay a specific sum on a specific date sometime in the future. If the bill of exchange is drawn on a bank, it is called a bank draft. If it is drawn on another party, it is called a trade draft. Bond A debt instrument issued for a period of more than one year with the purpose of raising capital by borrowing. The Federal government, states, cities, corporations, and many other types of institutions sell bonds. Generally, a bond is a promise to repay the principal along with interest (coupons) on a specified date (maturity). Question 2. Answer: What is the source for fixed income securities? The following are the sources of fixed income securities: Treasury bonds: Bonds issued by the government are termed as treasury bonds. For example, dated securities released by the government. These bonds are normally issued for longer maturity. Corporate Bonds: Like treasury bonds, these bonds are issued by the corporate sector for borrowing purposes. These bonds are traded in the secondary market and the price of the bonds depends on the market interest rates ruling at the time of trading. The price will appreciate if the coupon rate is higher than the prevailing market interest rates. As such, these bonds are quoted at a premium. Conversely, if the coupon rate on the bond is less than the prevailing market rates, then the bonds are quoted at a discount. Preferred stock or preference share: Though in the strict sense, it is to be treated as equity it is often included under fixed income securities as the dividend income is fixed for preference shares. At the time of liquidation of a company, preference shareholders have low priority compared to the bondholders, but they have higher priority compared to the equity shareholders. Dividends payable or due are cumulative and the preference shareholders have priority over ordinary shareholders in dividend payments. Floating rate bonds: Interest rates on these bonds are floated with some reference rate in the market. For example, a bond can be issued with a feature that the interest rate on this bond is 1% above the bank rate. The rates of interest are always in tune with the market rates because of this special feature of floating. Indexed bonds: In indexed bonds, the principal and coupon payment are linked to the market index like inflation and price index. Index bonds are attractive to investors as they are safer than the conventional bonds in terms of real interest rate risk and inflation expectation risk. Indexed bonds apart from providing safety to investors also provides a steady interest income of their investment while keeping the principal intact. Because both coupon and principal payments of an indexed bond are adjusted for inflation, an investor can count on the steady purchasing power provided by the coupon interest

146

Frequently Asked Questions in Management

payment during the life of the bond. Further when an indexed bond matures its principal has the same purchasing power as when it is invested. Junk bonds: Junk bonds are high yield bonds issued by companies that are considered highly speculative because of high risk of default. International bonds: They are divided into two categories namely, foreign bonds and euro bonds. Foreign bonds are issued by a borrowing company in another country and the bond is denominated in the currency of the country where it is marketed or sold. For example, an Indian company sells bonds in US in dollar denomination. Debentures: These instruments are also fixed income securities with a specified interest rate. These securities have charge over the assets of the issuer. In contrast to bonds, which are unsecured, debentures are secured. Debentures are an alternative source of borrowing based on assets similar to borrowings from banks and FIs with charge on assets. Question 3. Answer: What is credit rating? An impartial and objective opinion on the credit quality of debt or equity obligations of a company, given as a figure or letter by a credit rating agency is known as credit rating. What is the function of a dealer in debt market department of a bank? A dealer in a bank is supposed to be buying and selling government securities and other bonds/debentures in order to make profits and meet SLR requirements. In most of the banks, trading in debt market has become one of the main sources of profits. The dealer should be in touch with other market participants on a continuous basis and should know whats happening in the market. What do you understand by YTM? YTM tells us that if we hold the bond till maturity, then at the current price how much yield we are getting. It is the rate, which equates the discounted value of future cash flows to the present price of the bond. What do you understand by yield curve? Yield curve is the graph wherein the YTM is plotted on the vertical axis and the various maturities of the security on horizontal axis. It shows the changes in the term structure of interest rates as the YTM changes. Yield curve for various bonds differ. Owing to the factor of default risk premiums the yield curve for risky bonds are at a higher level than the yield curves for the comparatively safe bonds. Question 7. Answer: What do you mean by Bearer Bond? A bond which does not have the owners name on it and is redeemed at maturity to anyone who presents it.

Question 4. Answer:

Question 5. Answer:

Question 6. Answer:

147

Finance

Question 8. Answer:

What do you mean by Perpetual Bond? A bond with no pre-determined redemption date. Many have a date after which they may be redeemed by the issuer, such as UK government 2% Consols (1923 and after). Effectively, they have an open-ended issuers call. What do you understand by Zero Coupon Bond? These bonds are issued at a discount to their face value and are redeemed at par on maturity. The difference between their face value and the issue price represents the return to the investors.

Question 9. Answer:

Question 10. What is the difference between Zero Coupon Bond and Deep Discount Bond? Answer: Deep discount bond is issued with a very low coupon or no coupon that sell at a price far below. A Zero-coupon bond is a bond in which no periodic coupon is paid over the life of the contract. Instead, both the principal and the interest are paid at the maturity date. A bond that has no coupon is called a zero-coupon bond. Question 11. Who is a primary dealer? Answer: Primary dealer is a dealer in Government debt with whom Reserve Bank of India (RBI) transacts business.

Question 12. What is a Treasury Bill? Answer: A short-term government debt instrument with a maturity of one year or less. Bills are sold at a discount to face value (or par value) with the interest earned being the difference between the face value received at maturity and the price paid.

Question 13. Why is Gilt-Edged Securities considered risk free securities? Answer: Usually government securities and bonds are very safe assets to hold, as the government is responsible for the payment of interest and refund.

Question 14. Have you heard of Samurai Bonds and Shibosai Bonds? Answer: Samurai Bonds-Yen denominated bonds issued in Japan by foreign corporates, through a public offering. Shibosai Bonds -Yen denominated bonds issued in Japan by foreign companies by way of private placements. Question 15. Which bonds are referred as Bulldog Bonds? Answer: Bonds issued in United Kingdom by foreign issuers and are denominated in Pound Sterlings.

Question 16. Define Yankee Bonds?

148

Frequently Asked Questions in Management

Answer:

Bonds issued in United States of America by foreign firms and are denominated in dollars.

Question 17. What is a duration? What is the significance of duration for a zero coupon bond? Answer: A zero-coupon bond is a bond with no coupon payments. It carries zero duration. The holding period of a bond for which interest rate risk disappears is known as the duration of the bond. It is nothing but a weighted average time at which the present value is received and it is an alternative measure to maturity as a measure of the term of the bond. It measures the sensitivity of the bond price change in interest rates.

Question 18. How are bonds related to prices and why? Answer: The price of bonds is inversely related to the yield on bonds. The price of the bonds increases as yields go down and vice-versa. This is because, as yield goes down the expected value of future cash flows would be discounted with lower required rate of return yielding higher current price.

Question 19. What are spot rate and forward rates and how does one arrive at them? Answer: Spot Rate: The spot rate is discount rate of a single future cash flow such as zero. Pn = 100/(1 + Sn) n Where, Pn = Market price of an n-year zero coupon bond Sn = annualized n-year spot rate: the discount rate of an n-year zero-coupon bond Forward Rate: the rate agreed upon in a forward contract for a loan or the rate implied by the relationship between interests for different maturities, alternatively known as forward interest rate. From the spot rate arrived, one year forward rate can also be calculated. A given term structure of spot rates implies a specific term structure for forward rates. For instance, if the m-year and n-year spot rates are known, the annualized forward rates between maturities m and n i.e. fm,n can be calculated: (1 + fm,n)n-m = (1 + Sn)n /(1 + Sm)m where, fm,n = Aannualized forward rates between maturities m and n Sn = annualized n-year spot rate: the discount rate of an n-year zero-coupon bond Sm = annualized m-year spot rate: the discount rate of an m-year zero-coupon bond Question 20. How do you calculate yield on tax saving bonds? Answer: The net outflow is determined by total investment minus tax saving. This net outflow is equated to future cash inflows accruing in the future. The IRR of these inflows is called yield on tax saving bonds.

Question 21. What is the logic behind dividend tax-free?

149

Finance

Answer:

The logic behind the dividend tax-free is the avoidance of double taxation. Double taxation is unfair. Further, to double tax savings and investment goes beyond unfair; as economic and public policy. When a corporation earns a profit, it pays a tax as high as 35 percent on those earnings (and that rate can be as high as 40 percent with state taxes included). With the profits left over after taxes, many corporations in turn provide dividends to their individual shareholders. The shareholders are then taxed on those dividends. As a result, income earned by corporations is taxed twice, both at the corporate and shareholder level. The combined tax is up to 60 percent on the income from an equity investment, which is bordering on confiscation. This is a tax on corporate profitsthese shareholders are risking their money to invest in businesses that are growing the economy and creating jobs. That is a huge penalty, and it discourages savings and investment in our economy.

150

Frequently Asked Questions in Management

CAPITAL STRUCTURE
Question 1. Answer: What kind of capital structure should a newly floated company adopt? The capital structure of a newly floated company is dependent upon the management of the company. However, before deciding an appropriate capital structure, a newly floated company should bear in mind the rules and regulation regarding SEBI guidelines and norms of the financial institutions. According to the guidelines issued by SEBI, a newly established company with no previous track record can issue equity only at par unlike that of well established companies who can issue their equity capital at premium to par value. According to financial institutions normally the debt-equity norm for medium and large-scale projects is 1.5:1. Tell us something about negative capital and its worth? Upon any liquidation of partnership interest, the agreement requires partner having negative working capital that the partner whose accumulated losses and drawing are more than his capital contribution is required to pay to the partnership in cash the amount of any negative balance in her capital account. What do you understand by cost of capital? The minimum rate of return a firm must earn on its investment in order to satisfy the expectations of investors who provide the funds to the firm. It is often measured as the weighted arithmetic average of the cost of various sources of finance tapped by the firm. What is Weighted Average Cost of Capital (WACC)? In the long run, the company wants a balanced capital structure (the right mix of debt and equity) and for financing any investment project, it tries to maintain its capital structure intact. Thus the correct measure in evaluating a project is the overall cost of capital. This is the weighted average cost of capital from various sources. The weight is the proportion of each source of funds in the capital structure. The average cost of capital will change if the capital structure changes. What are agency costs? Creditors insist on certain restrictions in the form of restrictive covenants in the contract and they entail certain legal and enforcement costs. These costs impair the operating efficiency of the firm. Such costs are called agency costs. What are bankruptcy costs? In the real world, assets of a bankrupt firm cannot be sold at their economic value. But they can be sold at a discount and the firm has to incur certain legal and administrative costs. This entails significant costs in the form of sharply impaired operational efficiency. These are called Bankruptcy costs. As the Debt equity ratio increases, the bankruptcy costs also increase.

Question 2. Answer:

Question 3. Answer:

Question 4. Answer:

Question 5. Answer:

Question 6. Answer:

151

Finance

Question 7. Answer:

What is switching cost? Which industry has the lowest switching cost? A barrier to entry is created by the presence of switching costs, that is, one-time costs facing the buyer of switching from one suppliers product to anothers. Switching cost may include employee retraining costs, cost of new ancillary equipments, cost and time in testing and quantifying a new source. If these switching costs are high then new entrant must offer a major improvement in cost or performance in order for the buyer to switch from an incumbent. If IRR were less than the cost of capital of capital what would you do? While evaluating the feasibility of a project we compare the internal rate of return and cost of capital. If IRR is less than the cost of capital, then the project is not viable and cannot be accepted as the cost is less than the return, leading to loss in implementing the project. What is project finance? The amount required to meet the cost of the project is called project finance. The means of project finance are: Share Capital, Term Loans, Debenture Capital. Deferred Credit: Facility under which suppliers of plant and machinery offer to make the payment over a period of time. Incentive Sources: The aid given by government and its agencies like seed capital assistance, capital subsidy and tax deferment or exemption. Other Sources: These include public deposits, leasing and hire purchase.

Question 8. Answer:

Question 9. Answer:

152

Frequently Asked Questions in Management

CORPORATE FINANCE
Question 1. Answer: What are the important ratios in BS Analysis, Liquidity and profitability? In liquidity ratios there are three major ratios: Current Ratio: The current ratio is a test of a company's financial strength. It calculates how many rupees in assets are likely to be converted to cash within one year in order to pay debts that come due during the same year. You can find the current ratio by dividing the total current assets by the total current liabilities. For example, if a company has Rs.10 million in current assets and Rs.5 million in current liabilities, the current ratio would be 2:1 (10/5 = 2). Quick ratio: The quick ratio is more stringent measure of liquidity because inventories, which is least liquid of current assets is excluded from the ratio. It is calculated as: = (Current Assets- Inventories)/Current Liabilities Inventory turnover: The liquidity of the firms inventory maybe calculated by dividing the cost of goods sold by the firms inventory. Higher the ratio greater the efficiency of inventory management. Profitability Ratio Gross Profit margin: It is calculated as: = Gross profit/ Net sales Net sales= Sales- Excise Duty This ratio is used as an indicator of the efficiency of the production operation and the relation between production cost and selling price. Net Profit Margin: It is calculated as: = Net profit/ Net sales Net sales= Sales- Excise Duty It shows the earnings left for the shareholders as a percentage of net sales. It measures the overall efficiency of production, administration, selling, financing, pricing and tax management. Return on Equity: It measures the profitability of equity funds invested in the firm. ROE reflects the productivity of capital employed in the firm. It is calculated as: = Net Income/Average Equity Earning Power: It is a measure of the operating business performance, which is not effected by interest charges and tax payments. It is given by: = EBIT/ Average Total Assets Asset Turnover: It highlights the amount of assets that the firm used to produce its assets sales. It is given by: = Sales/ Average Total Assets. Question 2. Answer: What are the different ratios used to evaluate a manufacturing company and financial company. The following are the ratios used to evaluate a manufacturing company: Current Ratio: The current ratio is a test of a company's financial strength. It calculates how many rupees in assets are likely to be converted to cash within one year in order to pay debts that come due during the same year. You can find the current ratio by dividing

153

Finance

the total current assets by the total current liabilities. For example, if a company has Rs.10 million in current assets and Rs.5 million in current liabilities, the current ratio would be 2:1 (10/5 = 2). Quick ratio: The quick ratio is more stringent measure of liquidity because inventories, which is least liquid of current assets is excluded from the ratio. It is calculated as: = (Current Assets- Inventories)/Current Liabilities Inventory turnover: The liquidity of the firms inventory maybe calculated by dividing the cost of goods sold by the firms inventory. Higher the ratio greater the efficiency of inventory management. Profitability Ratio: Gross Profit margin: It is calculated as: = Gross profit/ Net sales Net sales= Sales- Excise Duty This ratio is used as an indicator of the efficiency of the production operation and the relation between production cost and selling price. Net Profit Margin: It is calculated as: = Net profit/ Net sales Net sales= Sales- Excise Duty It shows the earnings left for the shareholders as a percentage of net sales. It measures the overall efficiency of production, administration, selling, financing, pricing and tax management. Return on Equity: It measures the profitability of equity funds invested in the firm. ROE reflects the productivity of capital employed in the firm. It is calculated as: = Net Income/Average Equity Earning Power: It is a measure of the operating business performance, which is not effected by interest charges and tax payments. It is given by: = EBIT/ Average Total Assets Asset Turnover: It highlights the amount of assets that the firm used to produce its assets sales. It is given by: = Sales/ Average Total Assets. The ratios used by the financial company are as follows: Interest Expense Ratio= Interest Expense/Total Income Non- Interest Expense Ratio= Non-Interest Expense/Total Income Interest income to Total Assets= Interest Income/Total Assets Interest Earned on Credit = Interest Earned/Credit Interest on Deposits to Deposits= Interest on Deposits/Deposits. Question 3. Answer: What is ROI? The term return on investment is widely used in connection with the performance of a company or project. It is calculated as: ROI= Earning Before Interest and Taxes/ Total Assets.

154

Frequently Asked Questions in Management

Question 4. Answer:

What do you understand by the terms ROE and ROCE? ROE- Return on equity examines the profitability from the perspective of the equity investor, by relating profits to the equity investor to the book value of equity investment. ROE= Net Income/Book Value of Equity ROCE- The Return on Capital Employed ratio (ROCE) determines the profit earned by shareholders on their investment in the company. It is calculated as: = Net Income (NI)/Capital Employed

Question 5. Answer: Question 6. Answer:

What is debt equity ratio for a cement company? The ideal debt equity ratio for cement industry is 1:2. What are the effects of return on equity if investments are delayed by three months? The return of equity measures the profitability of equity funds invested in the firm. It is regarded as a very important measure because it reflects the productivity of the ownership capital employed in the firm. If investments are delayed by three months the return on equity will come down as there wont be any return on investments if they are kept idle. What is balanced scorecard? The balanced scorecard is a management system (not only a measurement system) that enables organizations to clarify their vision and strategy and translate them into action. It provides feedback around both the internal business processes and external outcomes in order to continuously improve strategic performance and results. When fully deployed, the balanced scorecard transforms strategic planning from an academic exercise into the nerve center of an enterprise. What is meant by gearing ratio? Gearing ratio is not a specific ratio. It is a general term given to leverage ratios that express the capital of a firm. Can you explain, what is P/E ratio? The P/E ratio takes the stock price and divides it by the last four quarters' worth of earnings. P/E= Market Price of the share/Earning Per Share (EPS) When a stock's P-E ratio is high, the majority of investors consider it overvalued. Stocks with low P/E is typically considered a good measurement of value. For instance, a Company that currently earns Re 1 per share and expects its earnings to grow at 20% p.a will sell at some multiple of its future earnings assuming that earnings will be Rs 2.50 (i.e Re 1 compounded at 20% p.a for 5 years) and that the normal P/E ratio is 15. Then the stock selling at a normal P/E ratio of 15 times of the expected earnings of Rs 2.50 could sell for Rs 37.50 (i.e. Rs. 2.5*15) or 37.5 times of this years earnings.

Question 7. Answer:

Question 8. Answer:

Question 9. Answer:

155

Finance

Question 10. What is the importance of P/E Ratio in stock valuation? Answer: Price-Earning Ratio is the ratio of market price per share to earnings per share. The higher the PE ratio the better the value of the stock.

Question 11. What do you understand by the term IRR? Answer: The Internal Rate of Return (IRR) is that rate at which the net present value of the expected cash flows is equal to zero. In other words, it is the rate, which equates the present value of cash inflows to the present value of cash outflows.

Question 12. Is cash a source or use of funds? Answer: It depends upon the source of the cash generated.

Question 13. What is "leverage"? Answer: Suppose a user of a forward market adopts a position worth Rs.100. As mentioned above, no money changes hands at the time the deal is signed. In practice, a good-faith deposit would be needed. Suppose the user puts up Rs.5 of collateral, using Rs.5 of capital, a position of Rs.100 is taken. In this case, we say there is "leverage of 20 times". This example involves a forward market. More generally, all derivatives involve leverage. Leverage makes derivatives useful; leverage is also the source of a host of disasters, payments crises, and systemic risk on financial markets. Understanding and controlling leverage is equivalent to understanding and controlling derivatives.

Question 14. What is financial and operating leverage? Answer: Financial leverage measures the effect of change in EBIT on the EPS of the company, and operating leverage examines the effect of change in the quantity produced on the EBIT of the company.

Question 15. Does higher leverage always increase the risk of a firm? Answer: No. Higher leverage always does not increase the risk of a firm. The proportion of financial and operating leverages makes the difference in the risk of a firm.

Question 16. What is factoring? Answer: Out right sale of the firms accounts receivable to another party called factor with or without recourse, who bears the risk of collection.

Question 17. What is debt-serving capacity? Answer: Company requires cash to service its debt obligations in the form of coupon payments and loan repayments in the future years. Investor protection can be impaired by the inability of borrower to meet these obligations due the insufficiency of cash. So, before investing determination of issuers capacity to cover total requirements including both interest payment and principal amount should be apprised. The formulae used for debtservicing ratio is: PAT+ Depreciation + Other non-cash charged + interest on term loan/ interest on term loan+ repayment of term loan.

156

Frequently Asked Questions in Management

Question 18. What is meant by Net working capital? What are the sources of working capital finance? Answer: Net working capital can be defined in the following two-ways: Net-working capital is the difference between current assets and current liabilities Net-working capital is the proportion of a firms current assets which is financed with long-term funds

Question 19. What is Working Capital Management? How is it done? Answer: The total funds of a firm are deployed in two ways - a certain portion for short-term requirements and the remaining balance, usually a major amount in long-term investments in fixed assets such as land, building and equipments. Funds deployed for short term are mainly for working capital or operational purposes. The total requirement of funds for operational purposes, if the credit the firm has obtained from others is deducted, the difference would be the amount of money the firm has against working capital requirements. Question 20. What are the recent changes in working capital norms? Answer: The following are the recent changes that took place with regard to capital norms: Though norms for computation of Working Capital have been liberalized, PSU banks continue to adopt the formula - based norms to assess working capital. This is on account of the affinity that the Public Sector Banks have towards norm-based lending which protects them from any accusation of bias in lending. In short, it could be said that in most cases, industries get some credit but not customized to their requirement! Question 21. How does a company get Key Cash Credit? Answer: Cash credit that is obtained by pledging inventory to the bank.

Question 22. What do you understand by Mortgage? Answer: Mortgage is the transfer to interest in specific immovable property for the purpose of securing an existing or a future debt or the performance of an engagement, which may give rise to pecuniary liability.

Question 23. What is Share buyback? What are its advantages to companies and investors? Answer: Share buyback is a companys plan to buy back its own shares from the marketplace, reducing the number of outstanding shares, and typically an indication that the companys management thinks the shares are undervalued. Generally firms buyback the shares to increase the market value of the shares. It increases the share price, which profits both the investors as well the companies.

Question 24. What is sweat equity? Answer: These are shares issued by the company to its directors and employees. The shares are issued in accordance with regulation issued by SEBI in case of listed companies.

157

Finance

Question 25. When does a company undertake a Bonus Issue? Answer: A company may choose to capitalize its reserves by issuing bonus shares to existing shareholders in proportion to their holdings. Bonus shares are issued free of cost, but since the number of shareholders remains the same and their proportionate holdings do not change, bonus shares do not increase the shareholders ownership of the company.

Question 26. What are the types of avenues for raising fund from abroad? What is the advantage/ procedure for ADRs? Answer: International capital can be mobilized by issuing variety of financial instruments. Prominent instruments are syndicated (traditional) bank loans or euro loans or euro credits, euro deposits or eurocurrency or euro money, foreign bonds, eurobonds, fixed rate notes (FRNs), note issuing facility (NIF), euro commercial paper (ECP), euro certificates of deposits (ECD), euro equities and equity related instruments like American Depositary Receipts (ADRs), Global Depositary Receipts (GDRs) and International Depositary Receipts (IDRs). The main advantage of ADR over GDR is that ADR can be converted into shares and back into ADRs while GDRs once converted into shares cannot be converted back into GDRs.

Question 27. What do you understand by Global Depository Receipts? Answer: Any instrument in the form of a depository receipt or certificate created by the Overseas Depository Bank outside India and issued to non-resident investors against the issue of ordinary shares or Foreign Currency Convertible Bonds of issuing company.

Question 28. What is an ADR? Answer: ADR (American Depository Receipts) is a certificate issued by a US bank containing a statement that a specific number of shares of a foreign company have been deposited with it. It is traded as a security in US exchanges and quoted in US dollars.

Question 29. What is the advantage of Convertible Debenture over other debentures? Answer: Fixed interest secured loan certificates, which carry a provision of conversion into a certain number of shares at par or at a premium or a certain date. When only a part of the loan is converted the certificate is called a partly convertible debenture, and when the entire amount is converted it is called a fully convertible debenture.

Question 30. What is known as bridge financing? Answer: Bridge financing is the finance made by a venture capital fund for a short period of 6 months to 1 year before the company goes public.

Question 31. What is a Venture Capital and what is its other name? Answer: Funds made available for start-up firms and small businesses with exceptional growth potential. Managerial and technical expertise is often also provided. Also called risk capital.

Question 32. What is mezzanine Finance?

158

Frequently Asked Questions in Management

Answer:

Mezzanine Financing is a type of equity financing used in takeovers, which uses preferred shares and convertible securities to make a target firm larger.

Question 33. Can all companies issue Commercial Paper? Answer: Short duration usage promissory notes with fixed maturity issued mostly by the leading creditworthy and highly credit rated companies. These are unsecured instruments negotiable by endorsement and delivery.

Question 34. What is EVA? Answer: Economic Value Added (EVA) is the difference between the firms after-tax return on capital and its cost of capital. EVA=(ROC WACC)*(Economic Book Value of Assets in place) = EBIT (1-Tax rate) WACC*(Economic Book Value of Assets in place) Question 35. What is meant by horizontal integration? Answer: An integration (merger or acquisition) of one company by another company in the same industry.

Question 36. What is Synergy in mergers? Answer: Synergy in mergers results from complementary activities. For instance, one firm may have substantial financial assets and another firm may have profitable investment opportunities. Hence after merger, the combined value of the firms is likely to be greater than the sum of the individual entities.

Question 37. Discuss the leverage buyout as a financial tool India Answer: One of the drawbacks for leverage buyouts is the lack of clear guidelines. Therefore, most of the public sector companies shy away from leveraged buyout financing. However, leveraged buyout financing is likely to emerge in India against the backdrop of the governments divestment program. Now the banks are coming forward to fund the leveraged buyout transaction. However, unlike foreign practice of funding based on the acquiree balance sheet. In India, banks fund acquisition relying on the acquirers balance sheet and the cash flows that the company can receive after the acquisition. Quite a few companies have raised financing through leveraged buyout over the past few years. The major ones are Tata takeover of Tetly, than Tata takeover of VSNL and finally Warburg. Newbridge bid for Punjab Tractor was financed by leveraged buyout transaction.

159

Finance

PROJECT MANAGEMENT
Question 1. Answer: What is a Project? A project is synthesizing predetermined amounts of the resources of an organization to generate something that will assist an organization in designing an executing its strategies. Give various steps in project management? The steps in Project Management can be segregated on various grounds. One of the practical approaches is splitting the steps in the following order: Plan: Question 3. Answer: Define Work Build Work Plan Work Plan Issues Scope Communication Risk Documents Quality Metrics

Question 2. Answer:

Manage:

What are the points to be considered before setting up a project in a new area, from the point of view of financial viability? The following points have to be taken into account before analyzing the financial viability of the project: Cost of project Means of financing Estimation of sales and production Cost of production Working capital requirements and its financing Estimates of working results Break-even point Projected cash flows statements Projected balance sheets

160

Frequently Asked Questions in Management

Question 4. Answer:

What is a risk in the context of Project Management? In the context of Project Management, a risk is any factor (or threat) that may adversely affect the successful completion of the project in terms of achievement of its outcomes, delivery of its outputs or adverse effects upon resourcing, time, cost and quality. How to identify Project risks? Risk identification in the Project Management is often done by using brainstorming sessions to identify and clarify the main risks that may work against the project achieving its stated outcomes. It is important to clearly define the scope of the project so that the identification of risks can remain focused upon what potentially threatens the achievement of outcomes, delivery of outputs, level of resourcing, time, cost and quality. What are the benefits of risk management in the context of project management? The benefit of risk management is to increase the likelihood that the project will be completed successfully by ensuring levels of risk and uncertainty in the project is properly managed. It enables those involved to identify possible risks, the manner in which they can be contained and the likely cost of mitigation strategies. Is it still worthwhile to do risk management when a project is half over? Yes, there are still risks associated with a project even if it is half over. This also forms a baseline for the remainder of the project. When is a project risk no longer a risk? A risk is no longer a risk when it no longer is a factor (or threat) that may adversely affect the successful completion of the project. This is usually as a result of mitigation strategies taken, if the threat has been realized, or if there has been a change in the environment that makes the risk no longer relevant. What is the difference between a risk and an issue in the context of Project Management? An issue is a problem or concern that may impede the progress of the project if not addressed. A risk is any factor (or threat) that may adversely affect the successful completion of the project in terms of achievement of its outcomes, delivery of its outputs or adverse effects upon resourcing, time, cost and quality.

Question 5. Answer:

Question 6. Answer:

Question 7. Answer:

Question 8. Answer:

Question 9. Answer:

Question 10. What is a Gantt Chart? Answer: A Gantt Chart is a way of illustrating multiple, time-based activities on a horizontal time-line.

Question 11. What is Critical Path? Give various steps of Critical Path Method (CPM)? Answer: Critical Path is the longest path from the beginning event to the end event. The minimum time required for completing the project is the duration on the critical path. Steps to compute Critical Path Method (CPM): 1 Inputs to Compute CPM

161

Finance

a b c 2

Activities Durations Immediate Predecessors

Methodology 2.1 Schedule all the paths with no unscheduled predecessors 2.1.1 Determine ES (Early Starts) Start ASAP EF (Early Finishes) ES + Duration 2.2 Schedule in reverse (schedule all activities with no unscheduled followers) 2.2.1 Determine LF (Late Finish) LS (Late Start) LF-Duration 2.3 Find Slack Slack = LS-ES

Determine Critical Path

162

Frequently Asked Questions in Management

HIRE PURCHASE / LEASING


Question 1. Answer: What is Hire Purchase Agreement? An agreement under which goods are let on hire and the hirer has an option to purchase them in accordance with the terms of the agreement. What do you understand by Cancelable Lease? A lease agreement that provides an option to the lessee for canceling the lease at short notice during the lease period. What do you understand by Non-cancelable Lease? A lease that is cancelable only (a) upon the occurrence of some remote contingency; or (b) with the permission of the lessor; or (c) if the lessee enters into a new lease for the same or an equivalent asset with the lessor; or (d) upon payment by the lessee of an additional amount such that at inception, continuation of the lease is reasonably certain. What do you understand by Net Lease? A finance lease where the executory costs incurred in relation to the leased asset like insurance, repairs, maintenance, etc. are paid by the lessee. What do you understand by Dry Lease? A variant of the operating lease where the lessee pays the executory costs connected with the lease asset like insurance, maintenance, repairs, etc. (contrast with Wet Lease) What is operating lease? It is a short-term, cancelable lease agreement where the lessor is responsible for maintenance, insurance and taxes. The lease period is usually less than the useful life of the equipment and the lease is not fully amortized. (Also called wet lease, service lease). An operating lease has the following characteristics: The lease term is significantly less than the economic life of the equipment. The lessee enjoys the right to terminate the lease at short notice without any significant penalty.

Question 2. Answer:

Question 3. Answer:

Question 4. Answer:

Question 5. Answer:

Question 6. Answer:

Question 7. Answer:

What do you understand by Open-ended Lease? A lease where the title to the asset passes to the lessee upon exercising a purchase option or payment of a guaranteed residual. Unlike a closed-end lease, ownership possibilities are open to the lessee under this lease. What is closed-end lease? A lease, which does not provide a purchase option to the lessee at the end of the lease period (contrast with Open-end Lease). In a closed-end lease, the asset reverts back to the lessor, on the expiry of the lease term.

Question 8. Answer:

163

Finance

Question 9. Answer:

What do you understand by Financial Lease? It is an intermediate-term to long-term non-cancelable lease agreement, where the lessee is responsible for maintenance, insurance and taxes. The lease is fully amortized during primary lease period, which is generally for 3 years. The lessee usually has the option of renewing the lease for further periods. (Also called capital lease).

Question 10. What is a Captive Leasing Company? Answer: A leasing company which is formed as a subsidiary of a manufacturing company for leasing the products of the parent company.

Question 11. How do you calculate Estimated Economic Life of an asset? Answer: The estimated period for which the asset is expected to be economically usable, by one or more users, with normal repairs and maintenance, for the purpose of which it was intended at the inception of the lease.

Question 12. What do you understand by Full Pay-out Lease? Answer: A lease where the aggregate lease rentals alone (without taking the residual value into consideration) equates to the investment in the leased asset plus an acceptable return on the investment.

Question 13. What do you understand by Indexed Lease? Answer: A lease with an indexed escalation clause. The indexed escalation clause requires the lease payment to be adjusted upward periodically by a percentage amount equal to the rate of increase in an external measure or index of inflation like the Consumer Price Index, Wholesale Price Index or the GNP Deflator. A real estate lease is usually structured as an indexed lease or as a percentage lease for compensating the lessor of the real estate for the loss in the purchasing power.

Question 14. What do you understand by Lease Line? Answer: A lease line of credit is similar to a bank line of credit under which the lessee is allowed to add on to the leased assets on the same basis terms and conditions without negotiating a new lease agreement every time a new equipment is leased.

Question 15. What do you understand by Skipped-payment Lease? Answer: A lease designed to skip rentals during the part of a year when the leased asset is expected to remain idle on account of uncontrollable factors like the seasonality of the operations, adverse weather conditions, etc.

Question 16. What do you understand by Swap Lease? Answer: A lease which allows the lessee to exchange equipment in need of major repair with properly working replacement equipment thereby enabling the lessee to avoid costly maintenance and repair delays.

164

Frequently Asked Questions in Management

Question 17. What is Bargain purchase option? Answer: A provision allowing the lessee at his option, to purchase the leased asset during the currency of the lease or at the end of lease period at a price lower than the expected fair market value at the date the option becomes exercisable. In India, an equipment lease cannot be structured with this option.

Question 18. What is the difference between Leasing and Hire Purchase? Explain especially from the accounting standards? Answer: Lease: Accounting Treatment of the Lease To the lessee, the end result of the lease transaction is the same as that of a finance transaction or a sale/purchase transaction. Yet the intent of both parties in a lease transaction is different than lending money or selling an asset. The purpose of a lease transaction is to provide to the lessee the right of profitable use and enjoyment of an asset which belongs to the lessor and thus legally the leased asset is the property of the lessor. Forming the basis of this opinion is the fact that since the introduction of the lease the accounting treatment of the lease has been of a letting and hiring agreement. (1) Lessor's Accounting Treatment: The lease rental collected by the lessor is recognized as revenue and listed on the lessor's income statement under sales. The leased asset is listed at the original purchase price on the lessor's balance sheet as a fixed asset. The depreciation of the leased asset is to coincide with the lease rentals received using the straight line (fixed amount) depreciation method over the length of the lease term, and reported in the income statement as cost of goods sold. Future lease rentals for active lease transactions are listed as accounts receivable on the footnotes to the balance sheet. (2) Lessee's Accounting Treatment: For the lessee, the lease rental can be treated as a production cost, sales cost, or fixed operating cost as appropriate depending on the use of the asset. The accounting for the lease is thus reported on the income statement and is not listed on the lessee's balance sheet. However, it is believed that to better disclose the economic realities of the lessee such long term obligations (future lease rentals) ought to be mentioned in the footnotes to the balance sheet. Hire-Purchase: The accounting mechanisms from the hirers angle is as follows: The cash purchase price of the asset is capitalized and the capital content of the hire purchase installments. Depreciation is charged on the cash purchase price of the asset in line with the depreciation policy pursued by the hirer with regard to other owned assets. The total charge for credit or the unmatured finance charge at the inception of the hire purchase transaction is allocated over the hire period using one of the following methods: Effective rate of interest method Sum-of-the-Years (SOYD) Method Straight Line Method

165

Finance

In the books of finance company, (owner) the accounting treatment is as follows: At the inception of the transaction, the finance company records the hire purchase installments receivable as a current assets and the finance income component of these installments as a current liability under the head of Unmatured Finance Charged. At the end of each accounting period, the finance company recognizes an appropriate part of the Unmatured Fixed Income as current income of the period. The methods that are followed for allocating the unmatured finance income over the relevant accounting periods. At the end of each accounting period, the hire purchase price less the installments received is shown as a receivable and the finance income component of these installments is shown as a current liability. The direct costs associated with setting up the transaction are either expensed immediately or allocated against the finance income over the hire period. Question 19. What is the significance of Lease Term in a Leasing agreement? Answer: Lease Term signify the non-cancelable period for which the lessee has contracted to lease the asset, together with any further terms for which the lessee has the option to continue to lease the asset, with or without payment, which option, at the inception of the lease, is reasonably certain that the lessee will exercise.

Question 20. What are the various requirements of leasing document? Answer: There are a number of legal issues to be considered before drafting a lease agreement. Some of these issues are: - Legal relationship between equipment supplier, the lessor and the lessee - Insurance -Usage and maintenance -Sub-lease -Set-off provisions -Defaults and remedies.

166

Frequently Asked Questions in Management

INTERNATIONAL FINANCE
Question 1. Answer: What are fixed exchange rate system and floating exchange rate systems? Fixed exchange rate system is a monetary system under which the exchange rate system between currencies is maintained at particular levels, which do not change frequently. Floating exchange rate system is a monetary system under which exchange rates change frequently in accordance with the market forces. What is Forward Rate Agreement (FRA)? FRA is an agreement under which the seller assures the buyer certain interest rate on a notional sum for a pre-determined term, which is with reference to a pre-selected market rate, at the end of a specified period. The difference between the agreed rate and actual market rate prevailing at the end of the specified period is paid by the seller to the buyer if the agreed rate is higher than the market rate, vice versa. What is Letter of Credit? Letter of credit is an arrangement by means of which an issuing bank acting at the request of an applicant, undertakes to pay to a beneficiary a predetermined amount by a given date according to agreed stipulations and against presentation of stipulated documents. What is Irrevocable Letter of Credit? It is a Letter of Credit (LC) where the cancellation or any amendment cannot be made without the prior acceptance of all the parties to the said Letter of Credit (LC). What is revolving letter of credit? A letter of credit whereby the credit available to the beneficiary gets reinstated to the original amount once a drawing is made, is called revolving credit. What is CIF? CIF is cost, insurance and freight. It is a trade term requiring the seller to arrange for the carriage of goods by sea to a port of destination, and provide the buyer with the documents necessary to obtain the goods from the carrier. What is Interest rate parity? It is a condition under which the premium on a foreign currency is equal to interest rate differential between the two countries.

Question 2. Answer:

Question 3. Answer:

Question 4. Answer:

Question 5. Answer:

Question 6. Answer:

Question 6: Answer:

167

Finance

DERIVATIVES
Question 1. Answer: What are "derivatives"? A derivative is a financial instrument, which derives its value from some other financial price. This "other financial price" is called the underlying. A wheat farmer may wish to contract to sell his harvest at a future date to eliminate the risk of a change in prices by that date. The price for such a contract would obviously depend upon the current spot price of wheat. Such a transaction could take place on a wheat forward market. Here, the wheat forward is the "derivative" and wheat on the spot market is "the underlying". The terms "derivative contract", "derivative product", or "derivative" are used interchangeably. The most important derivatives are futures and options. What are "OTC derivatives"? A derivative contract that is privately negotiated is called an OTC derivative. OTC trades have no anonymity, and they generally do not go through a clearing corporation. Every derivative product can either trade OTC (i.e., through private negotiation), or on an exchange. In one specific case, the jargon demarcates this clearly: OTC futures contracts are called "forwards" (or, exchange-traded forwards are called "futures"). In other cases, there is no such distinguishing notation. There are "exchange-traded options" as opposed to "OTC options"; but they are both called options. What is Value Engineering? It is an approach that examines each element of a product or system to determine if there is a better or cheaper way of achieving the same function. What is the process involved in Financial Engineering? Financial Engineering involves design, development and implementation of innovative financial instruments and processes and the formulation of creative solutions to the problems in corporate finance. How firms can manage their risks by using derivatives? By using derivatives, firms can follow hedging technique to minimize the risk. In the hedging process, firms identify two exactly correlated assets as far as returns are concerned and buy one asset while selling other asset. In what way an investor can participate in the derivatives market? An Investor can participate in the derivatives market by using futures contract for Hedging Speculation Arbitrage Spread trading

Question 2. Answer:

Question 3. Answer:

Question 4. Answer:

Question 5. Answer:

Question 6. Answer:

168

Frequently Asked Questions in Management

Question 7. Answer:

What determines the fair price of a derivative? The market forces determine the fair price of derivative. The fair price of a derivative is the price at which profitable arbitrage is infeasible. In this sense, arbitrage (and arbitrage alone) determines the fair price of a derivative: this is the price at which there are no profitable arbitrage opportunities. How can the risk be transferred in the derivative market? Derivatives are used to transfer risk by hedging. A Hedge helps to lock in existing profits of portfolio. Hedging is used to protect portfolio volatility due to market fluctuation during budget, elections and other political or corporate turmoil. Thus hedging helps to reduce risk by locking returns but does not maximize them. The basic rule in hedging is that the risk of loss in portfolio is offset by the gains in the futures. Suppose we have a portfolio of Rs.2 cr and it is expected that the market to fall by 400-500 points in the short term. Then, to safeguard the interest, we can go short in the derivatives market. In a situation where the expected fall takes place, the loss in the portfolio is compensated by gains in the derivatives market. In the absence of any hedging, loss would have been substantial. Selling the portfolio would involve high cost. Hence hedging is beneficial. Which derivatives are traded in Indian exchanges? The derivatives traded in Indian exchanges are: 1. Stock Options 2. Index Options. 3. Index Futures 6. 4. Stock Futures 5. Currency Options. Interest Rate Futures

Question 8. Answer:

Question 9. Answer:

Question 10. Are Indian companies permitted to hedge their exposures in the international derivatives markets? What about banks? Answer: Yes, the Indian firms are allowed to hedge their exposures in international derivatives markets. The same is true with the Indian banks also but subject to mentioned norms.

Question 11. Who needs hedging using index futures? Answer: The general principle is: you need hedging using index futures when your exposure to movements of Nifty is not what you would like it to be. If your index exposure is lower than what you like, you should buy index futures. If your index exposure is higher than what you like, you should sell index futures.

Question 12. What is margin money? Answer: Margin money means the sum that has to be paid by both buyer and seller for entering into a future contract. The aim of margin money is to minimize the risk of default by either of the parties of the transaction. Margin money is like a security deposit. There are different types of margins like Initial margin, Variation margin, Maintenance margin and Additional margin.

169

Finance

Question 13. What are the index futures contracts available in India? Answer: The National Stock Exchange commenced trading in index futures on June 12, 2000. The NIFTY futures contracts are based on the popular market benchmark S&P CNX NIFTY Index. S&P CNX Nifty futures contracts have a maximum of 3-month trading cycle - the near month (one), the next month (two) and the far month (three). A new contract will be introduced on the trading day following the expiry of the near month contract. S&P CNX Nifty futures contracts expire on the last Thursday of the expiry month. If the last Thursday is a trading holiday, the contracts shall expire on the previous trading day. The permitted lot size of S&P CNX NIFTY contracts is 200 and multiple. The Bombay Stock Exchange also commenced trading in index futures in June 2000. This index future is based on the BSE Sensitive Index of 30 shares. The permitted lot size is 50 and multiples.

Question 14. How are index futures valued? Answer: Future value = Spot price + carry cost - carry returns Where, Spot price = current index Carry cost = Holding cost of the future index Carry return = Dividends accrued during the period of carry. Question 15. What is marked-to-market? Answer: This is procedure whereby the profits or losses on the positions of traders are determined every day and settled. The traders who incurred losses have to pay variation margins.

Question 16. What is an "option"? Answer: An option is the right, but not the obligation, to buy or sell something at a stated date at a stated price. A "call option" gives one the right to buy, a "put option" gives one the right to sell. Consider a typical transaction. On 1 July 2000, S sells a call option to L for a price of Rs.3.25. Now L has the right to come to S on 31 Dec 2000 and buy 1 share of Reliance at Rs.500. Here, Rs.3.25 is the "option price", Rs.500 is the "exercise price" and 31 Dec 2000 is the "expiration date". L does not have to buy 1 share of Reliance on 31 Dec 2000 at Rs.500 from S (unlike a forward/futures contract which is binding on both sides). It is only if Reliance is above Rs.500, on 31 Dec 2000, that L will find it useful to exercise his right. If L chooses to exercise the option, S is obliged to live up to his end of the deal: i.e. S stands ready to sell a share of Reliance to L at Rs.500 on 31 Dec 2000. Hence, at option expiration, there are two outcomes that are possible: an option could be profitably exercised, or it could be allowed to die unused. If the option lapses unused, then L has lost the original option price (Rs.3.25) and S has gained it. When L and S enter into a futures contract, there is no payment (other than initial margin). In contrast, the option has a positive price, which is paid in full on the date that the option is purchased. Options come in two varieties - European and American. In a European option, the holder of the option can only exercise his right (if he should so desire) on the expiration date. In an American option, he can exercise this right anytime between purchase date

170

Frequently Asked Questions in Management

and the expiration date. The price of an option is determined on the secondary market. An option always has a non-negative value: i.e., the value of an option is never negative. Question 17. What is the role of Option? Answer: A legally binding agreement that confers the right, but not the obligation, to the holder to buy (in the case of a call option) or sell (in the case of a put option) an underlying asset (which may be a financial instrument, commodity, or futures contract) at a price agreed now (the exercise or strike price) by any time before a specified expiry date in the future (if it is an American-style option) or on a specified date in the future (if it is a European-style option). The writer or seller of the option has the obligation to fulfill the contract if the holder wishes to exercise the option (take up the rights). This option is obtained in exchange for payment of a premium. There are exchange-traded options available as well as over-the-counter varieties.

Question 18. What is a put option? Answer: An option (but not obligation) to sell the underlying asset at a decided price and time in future.

Question 19. What is a call option? Answer: An option (but not obligation) to buy the underlying asset at a decided price and time in future.

Question 20. What is a Currency Option? Answer: The option to buy or sell a specified amount of a given currency at a stated rate at or by a specified date in the future.

Question 21. What do you mean by an American Option? Answer: An option that can be exercised at any time prior to or on expiry.

Question 22. What is a European Option? Answer: An option that can be exercised only on the expiry date.

Question 23. What is the difference between American Option and European Option? Answer: European Options give the holder the right, but not the obligation, to buy or sell the underlying instrument only on the expiry date. This means that the Option cannot be exercised early. Settlement is based on a particular strike price at expiration. On the other hand, American Options give the holder the right, but not the obligation, to buy or sell the underlying instrument on or before the expiry date. This means that this Option can be exercised early.

Question 24. What do you mean by Naked Option? Answer: A Call Option wherein the seller does not own shares but hopes to buy from the market, expecting the price to fall. If it does, he makes a profit on the difference; if it doesnt

171

Finance

and rises, the seller is caught in a naked position and must sustain a loss by buying at the higher price. Question 25. What are Covered & Naked Calls? Answer: A call option position that is covered by an opposite position in the underlying instrument (for example shares, commodities etc), is called a covered call. Writing covered calls involves writing call options when the shares that might have to be delivered (if option holder exercises his right to buy), are already owned. Covered calls are far less risky than naked calls (where there is no opposite position in the underlying), since the worst that can happen is that the investor is required to sell shares already owned at below their market value. When a physical delivery uncovered/ naked call is assigned an exercise, the writer will have to purchase the underlying asset to meet his call obligation and his loss will be the excess of the purchase price over the exercise price of the call reduced by the premium received for writing the call.

Question 26. What are call / put options on bonds? Answer: A call option on a bond is option to the issuer of the bond to call back the bonds after a stipulated period of time. The issuer will call back the bond in the scenario of falling interest rates. A put option is an option to the bond investor to ask for redemption of a bond after the stipulated period of time. An investor would exercise the option in the scenario of rising interest rates.

Question 27. What is a straddle strategy in options? Answer: A straddle strategy is a combined option position adopted by buying or selling a call option and put option simultaneously.

Question 28. What are the factors influencing option prices? Answer: The spot price or current price of the underlying asset The exercise price or strike price of the option The time-to-maturity Volatility of the underlying asset The risk-free rate of interest Dividend expected during the life of the option is a dividend paying stock.

Question 29. What is debt-serving capacity? Answer: Company requires cash to service its debt obligations in the form of coupon payments and loan repayments in the future years. Investor protection can be impaired by the inability of borrower to meet these obligations due the insufficiency of cash. So, before investing determination of issuers capacity to cover total requirements including both interest payment and principal amount should be apprised. The formulae used for debtservicing ratio is: PAT+ Depreciation + Other non-cash charged + interest on term loan/ interest on term loan+ repayment of term loan. Question 30. What is Option Pricing?

172

Frequently Asked Questions in Management

Answer:

Giving a quote to create an obligation under the option sold by an option writer or seller. Option pricing is influenced by elements such as Price of Underlying assets Time to Expiry Exercise Price Time to Maturity Volatility of the Underlying assets.

Question 31. Name the popular option pricing model? Answer: There are various models developed to price the option. Two very important models are: The Black & Scholes Model The Black & Scholes model was published in 1973 by Fisher Black and Myron Scholes. It is one of the most popular options pricing models. It is noted for its relative simplicity and its fast mode of calculation: unlike the binomial model, it does not rely on calculation by iteration. The Black-Scholes model is used to calculate a theoretical call price (ignoring dividends paid during the life of the option) using the five key determinants of an option's price: stock price, strike price, volatility, time to expiration, and short-term (risk free) interest rate. The original formula for calculating the theoretical option price (OP) is as follows: Where:

The variables are: S = stock price; X = strike price t = time remaining until expiration, expressed as a percent of a year r = current continuously compounded risk-free interest rate v = annual volatility of stock price (the standard deviation of the short-term returns per one year). ln = natural logarithm N(x) = standard normal cumulative distribution function e = the exponential function The Binomial Pricing Model The binomial model is an options pricing model, which was developed by William Sharpe in 1978. Today, one finds a large variety of pricing models, which differ according to their hypotheses or the underlying instruments upon which they are based (stock options, currency options, options on interest rates). Question 32. What is arbitraging in the index futures market?

173

Finance

Answer:

The fair price of a future is the cost of the underlying assets plus the cost of carrying it till the maturity date. The fair pricing of index futures results from actions of traders known as arbitrageurs. The arbitrageur can buy the portfolio of stocks on which the index is based (or a good approximation) and hold it until the maturity date of the future contract. This will involve the interest cost on money borrowed to buy the portfolio. If the actual futures price on the day exceeds the cost of holding the index portfolio, the arbitrageur sells the index futures contract and buys the index portfolio in the cash market. On the maturity date, the proceeds from the sale position of the futures would exceed the sum of money to be repaid. The excess would be arbitrage profit. Such sales by the arbitrageur force the futures price to go down. Similarly arbitrageurs can buy in future markets and sell in cash markets to correct the opposite type of difference in price of futures and cash market.

Question 33. What is currency swap? Answer: An interest rate swap under which the underlying notional principal and the interest rates streams is denominated in two different currencies.

Question 34. What is interest rate swap? Answer: An agreement under which two parities contract to pay each other a stream of interest payments on a notional principal up to a termination date.

Question 35. When might I find that my index exposure is not what it should be? Answer: A few situations are: You are a speculator about an individual stock or an industry. You have an equity portfolio and become uncomfortable about equity market risk for the near future. You expect to obtain funds at a known future date, but you would like to lock in on equity investments right now at present prices. You have underwritten an IPO and are vulnerable to losses if the market crashes and the IPO devolves on you. You are uncomfortable with the vulnerabilities of your business, where cash flows swing dramatically with movements of Nifty.

Question 36. How would a seller "deliver" a market index? Answer: On futures markets, open positions as of the expiration date are normally supposed to turn into delivery by the seller and payment by the buyer. It is not feasible to deliver the market index. Hence open positions are squared off in cash on the expiration date, with respect to the spot Nifty. Specifically, on the expiration date, the last mark to market margin is calculated with respect to the spot Nifty instead of the futures price.

FOREX
Question 1. What do you understand by transaction exposure?

174

Frequently Asked Questions in Management

Answer:

It measures the foreign exchange rate between the time the transaction is executed and the time it is settled. What is translation exposure with reference foreign transaction? It refers to the risk arising on account of changes in exchange rates at the time of finalizing/consolidating the financial statements which has assets/liabilities denominated in foreign currencies. What are "exchange-traded derivatives"? Derivatives which trade on an exchange are called "exchange-traded derivatives". Trades on an exchange generally take place with anonymity. Trades at an exchange generally go through the clearing corporation. What is a "forward" transaction? In a forward contract, two parties irrevocably agree to settle a trade at a future date, for a stated price and quantity. No money changes hands at the time the trade is agreed upon. Suppose a buyer L and a seller S agree to do a trade in 100 grams of gold on 31 Dec 2001 at Rs.5,000/tola. Here, Rs.5,000/tola is the "forward price of 31 Dec 2001 Gold". The buyer L is said to be long and the seller S is said to be short. Once the contract has been entered into, L is obligated to pay S Rs.500,000 on 31 Dec 2001, and take delivery of 100 tolas of gold. Similarly, S is obligated to be ready to accept Rs.500,000 on 31 Dec 2001, and give 100 tolas of gold in exchange. Why is forward contracting useful? Forward contracting is valuable in hedging and speculation. The classic hedging application is that of a wheat farmer forward-selling his harvest, at the time of sowing, in order to eliminate price risk. Conversely, a bread factory could buy wheat forward in order to assist production planning without the risk of price fluctuations. If a speculator has information or analysis which forecasts an upturn in a price, then he can adopt a buy position (go long) on the forward market instead of the cash market. The speculator would wait for the price to rise, and then close out the position on the forward market (by selling off the forward contracts). This is a good alternative to speculation using the spot market, which involves buying wheat, storing it for a while, and then selling it off. A speculator prefers transactions involving a forward market, because (a) the costs of taking or making delivery of wheat is avoided, and (b) funds are not blocked for the purpose of speculation. What is SWIFT? And what does it do? Society for Worldwide Interbank Financial Telecommunications - It is a worldwide computer network that supports international funds transfers between banks. What is a vostro account? A nostro account from the correspondent banks point of view

Question 2. Answer:

Question 3. Answer:

Question 4. Answer:

Question 5. Answer:

Question 6. Answer:

Question 7. Answer:

175

Finance

Question 8. Answer: Question 9. Answer:

What is euribor? The German Interbank offer rate for loans denominated in euro. What is a wharfinger? The owner or incharge of a structure built near a harbor for facilitating the loading and unloading of goods into ships. He gets rent for the goods placed in that area. He has lien on the goods till the rent is paid.

176

Frequently Asked Questions in Management

FUTURES
Question 1: Answer: What is a futures contract? A futures contract is an exchange tradable contract between two parties to buy or sell at the certain time in the future for a certain price. How do futures trade? In a cash market, the basic dynamic is that the issuer puts out paper, and people trade this paper. In contrast, with futures (as with all derivatives), there is no issuer, and hence, there is no fixed issue size. The net supply of all derivatives contracts is 0. For each buyer, there is an equal and opposite seller. A contract is born when a buyer and a seller meet on the market. The total number of contracts that exist at a point is called open interest. What is cash settlement? The forward or futures contracts discussed so far involved physical settlement. On 31 Dec 2001, the seller was supposed to come up with 100 tolas of gold and the buyer was supposed to pay for it. In practice, settlement involves high transactions costs. This is particularly the case for products such as the equity index, or an inter-bank deposit, where effecting settlement is extremely difficult or impossible. In these cases, futures markets use "cash settlement". Here, the terminal value of the product is deemed to be equal to the price seen on the spot market. This is used to determine cash transfers from the counter parties of the futures contract. The cash transfer is treated as settlement. Example. Suppose L has purchased 30 units of Nifty from S at a price of 1500 on 31 Dec 2000. Suppose we come to the expiration date, i.e. 31 Dec 2000, and the Nifty spot is actually at 1600. In this case, L has made a profit of Rs.100 per Nifty and S has made a loss of Rs.100 per Nifty. A profit/loss of Rs.100 per nifty applied to a transaction of 30 nifties translates into a profit/loss of Rs.3,000. Hence, the clearing corporation organizes a payment of Rs.3,000 from S and a payment of Rs.3,000 to L. This is called cash settlement. Cash settlement was an important advance, which extended the reach of derivatives into many products where physical settlement was unviable. What is the market lot? The market lot is 200 nifties. A user will be able to buy 200 or 400 nifties, but not 300 nifties. If Nifty is at 1500, the smallest transaction will have a notional value of Rs.300,000. What is "basis risk"? Basis risk is the risk that users of the futures market suffer, owing to unwanted fluctuations of the basis. In the ideal futures market, the basis should reflect interest rates, and interest rates alone. In reality, the basis fluctuates within a band. These fluctuations reduce the usefulness of the futures market for hedgers and speculators. How does the futures market solve the problems of forward markets? Futures markets feature a series of innovations to solve the problems of forward markets:

Question 2: Answer:

Question 3: Answer:

Question 4: Answer:

Question 5: Answer:

Question 6: Answer:

177

Finance

Futures contracts trade at an exchange with price-time priority. All buyers and sellers come to one exchange. This reduces search costs and improves liquidity. This harnesses the gains that are commonly obtained in going from a non-transparent club market (based on telephones) to an anonymous, electronic exchange, which is open to participation. The anonymity of the exchange environment largely eliminates cartel formation. Futures contracts are standardized - all buyers or sellers are constrained to only choose from a small list of tradable contracts defined by the exchange. This avoids the illiquidity that goes along with the unlimited customization of forward contracts. A new credit enhancement institution, the clearing corporation, eliminates counter party risk on futures markets. The clearing corporation interposes itself into every transaction, buying from the seller and selling to the buyer. This is called Novation. This insulates each from the credit risk of the other. In futures markets, unlike in forward markets, increasing the time to expiration does not increase the counter party risk. Novation at the clearing corporation makes it possible to have safe trading between strangers. This is what enables large-scale participation into the futures market - in contrast with small clubs, which trade by telephone - and makes futures markets liquid.

Question 7. Answer:

What determines the fair price of an index futures product? The pricing of index futures depends upon the spot index, the cost of carry, and expected dividends. For simplicity, suppose no dividends are expected, suppose the spot Nifty is at 1000 and suppose the one-month interest rate is 1.5%, then the fair price of an index futures contract that expires in a month is 1015. Who are the users of index futures? As with all derivatives, there are (a) speculators, (b) hedgers and (c) arbitrageurs. Speculators would make forecasts about movements in Nifty or movements in futures prices. Hedgers would take buy or sell positions on Nifty futures in offsetting equity exposure that they have, which they consider undesirable. Arbitrageurs lend or borrow money from the market, depending on whether rates of return are attractive. How does one speculate using index futures? There are several kinds of speculations that are possible - forecasting movements of Nifty, forecasting movements in Nifty futures prices, and forecasting interest rates.

Question 8. Answer:

Question 9. Answer:

Question 10. How do I lend money into the futures market? Answer: Buy a million rupees of Nifty on the spot market. Pay for them, and take delivery. When you make the payment, you are "giving a loan". Simultaneously, sell off a million rupees of Nifty futures. Hold these positions till the futures expiration date. On the futures expiration date, sell off the Nifty shares on the spot market. When you get paid for these, you are "getting your loan repaid".

178

Frequently Asked Questions in Management

PART - 5

INSURANCE

179

Insurance

180

Frequently Asked Questions in Management

Question 1. Answer:

Which Act governs insurance? In India, the business of insurance is governed by IRDA Act, 1999 Insurance Regulatory and Development Authority Act, 1999. What is IRDA? IRDA stands for Insurance Regulatory and Development Authority, which came into being in 1999. It is the regulatory authority, which governs all business pertaining to all forms of insurance in India. What is the difference between insurance and assurance? Insurance is basically a risk-transfer tool, which provides for economic compensation of a loss incurred by the insured. Assurance applies mostly to life insurance contracts, in which there is a certainty of repayment on the happening of the event that is either on maturity or on the death of the assured. In non-life insurance, however, the insured event may occur or not during the contracted period and as such there is no assurance of payment. Differentiate between life and non-life insurance. Most life insurance contracts are contracts of assurance whereas all the non-life insurance contracts are contracts of insurance. Life insurance contracts are essentially long-term in nature whereas non-life insurance contracts are short-term in nature (mostly for one year, with a few exceptions). Non-life insurance contracts are contracts of indemnity whereby actual loss incurred by the insured is indemnified. Life insurance contracts are known as benefit contracts because there cannot be indemnification for the loss of human life. Do you have any knowledge about insurance marketing? Insurance marketing is a part of the services marketing and in fact, the most complex among them. In India, especially the concept of insurance is not very well understood and people buy insurance for reasons other than risk-coverage. It is popularly said that insurance is sold and seldom bought thereby indicating that insurance is bought on needs of compulsion or other mandatory requirements. What are the various steps involved in issuing a policy to a client? What are the various departments involved in this process and what role do they play? There has got to be a proposal by the party interested in purchasing insurance, giving the details of the life or property to be insured. The underwriting department of the insurer, after analyzing the contents and placing the risk properly, then insures the proponent or the proposed property. The process has to be done through the distribution channel, which in India, is mostly the insurance agent. Depending on the type of insurance precisely, the various departments involved are Marketing, New Business, and Accounts, although the nomenclature might differ from company to company. What do you understand by underwriting?

Question 2. Answer:

Question 3. Answer:

Question 4. Answer:

Question 5. Answer:

Question 6. Answer:

Question 7.

181

Insurance

Answer:

Underwriting is the process of deciding whether or not a proposal for insurance is to be accepted; and if it is to be accepted, at what terms and rates. Thus there is an element of selection and classification in underwriting. How do you handle claims in insurance? The process differs depending on the class of insurance, i.e. whether it is life, motor, health or others. Within life insurance, it would matter again whether the claim is a death claim or a maturity claim. In any case, the policyholder or the claimant has to submit the details of the policy and the claim in the required format and the insurance company takes up the payment of the claim after deciding whether the claim is as per the terms of the policy conditions or not. What is the outlook for insurance industry? Although the insurance concept is not very well understood in India, for different reasons, insurance companies have been performing reasonably well, especially the life insurance giant, LIC of India is synonymous with insurance itself. With the opening up of the insurance sector, it is expected that insurance business is going to show tremendous growth and attain world-class standards.

Question 8. Answer:

Question 9. Answer:

Question 10. What is human life value? Answer: Life insurance contracts are one of benefit and not of indemnity. As such, the amount of insurance up to which a person can go, is limitless. Logically, however, it depends upon the persons earnings, premium paying capacity, the needs of the dependents (should anything happen to the breadwinner) etc. Human Life Value (HLV), which was propounded by S.S.Huebner, the father of life insurance, is the limit up to which a person can go for life insurance and is arrived at by taking care of all the components enunciated above.

Question 11. Name some general insurance products. Answer: Some of the general insurance products are: Mediclaim policy, Motor Vehicle insurance (Liability and Own damage sections), Home owners policy, Overseas Mediclaim policy, Standard Fire and Special Perils policy (SFSP), etc.

Question 12. Amongst individual financial products portfolio, where do you place an insurance product? Answer: Insurance occupies a very special place in any investment portfolio. It is the only instrument, which takes care of the element of risk. For example: If a persons life comes to an end unfortunately after paying a paltry sum of money towards the first premium, his family would stand to get the total insured sum, which is not possible in any other form of saving. In this regard, insurance occupies a very unique place among the several forms of saving.

Question 13. What do you understand by risk management?

182

Frequently Asked Questions in Management

Answer:

Risk management is the process of taking stock of the various risks that an entity or an individual is exposed to and then decides how to manage these risks. Risk management can be said to be either by conscious risk avoidance, risk absorption or risk transfer. Insurance is a risk transfer tool.

Question 14. How does insurance mitigate risk? Answer: While insurance cannot obviate the happening of the event and thereby avoid any economic loss to the entity, it mitigates the loss by providing economic indemnification of the losses incurred by the entities, provided the risk is a covered risk and not an excluded one.

Question 15. What are the factors you will consider while evaluating risk in manufacturing factory? Answer: While evaluating the risk in a manufacturing concern, the various factors that should be considered are: the type of business or the product being manufactured; the environs in which the business/manufacture is being conducted; additional susceptibility of the process involved to special hazards, if any; involvement of different types of machinery; number and skill of the personnel involved; working hours etc.

Question 16. Apart from traditional channels which channels can we use in marketing insurance? Answer: The traditional channel for insurance has been the agent. However, in different parts of the world, several new channels have been very actively performing. Some of them are brokers, bancassurance, internet etc. In India, while these channels have not made any headway all these years, they may play a very crucial role in the coming years in view of the opening up of the insurance industry and introduction of these channels.

Question 17. How would you use your marketing knowledge in insurance? Answer: The bane of insurance industry in India is the lack of awareness about the insurance concept among the common populace and the lack of thrust on the part of the marketing personnel to do anything to improve upon this apathy. The real marketing challenge would be to promote the understanding of the benefits of insurance among the people and ensure that the age-old adage of insurance being sold and not bought is put to an end.

Question 18. What do you understand by fidelity guarantee? Answer: Fidelity guarantee is a policy by which indemnity to the employer for financial loss sustained as a result of dishonesty, default, negligence, misappropriation, forgery, embezzlement, larceny or fraudulent conversion of money or goods of the employer committed by salaried employees in the course of performance of their duties is provided.

Question 19. Can you tell us what a bancassurance is? Answer: Bancassurance is an integration of the services of banking and insurance and is a popular mode of distribution in the developed insurance markets especially in Europe. It aims at

183

Insurance

cross-selling of the banking and insurance products by banks and insurance companies. It is yet to gain much momentum in India. Question 20. What knowledge do you have about health insurance and pension insurance? Answer: Health insurance provides for financial compensation in the treatment of covered diseases during the period of coverage. Normally the policy covers all diseases except preexisting diseases, that is, those diseases which the proponent has been suffering from at the time of proposal or has earlier suffered. In India, Mediclaim policy is the most popular product of health insurance. Pension products pay for the superannuation benefits of a retired employee and are issued by insurance companies, mainly life insurance companies, in the form of annuities.

Question 21. How do you propose to manage your workforce to meet your targets? Answer: First and foremost, the corporate goals must be kept in mind and in tune with these goals, there must be zone-wise, division-wise and branch-wise targets for achieving the goals in various areas like new business, retention of existing business, timely claim settlement etc. Within the goals fixed for a branch, each marketing person would be assigned targets to be achieved during the period under consideration, depending on his/her existing quantum of business, potential for fresh business in the geographical region and other considerations.

Question 22. How would you handle the conflicting interests between sales and underwriting? Answer: While the interests of the sales personnel and the underwriters are not totally mutually conflicting, a situation could arise wherein the strict interpretation of the guidelines by the underwriting personnel would jeopardize the enthusiasm of the marketing force. While it is true that a very liberal underwriting stance could put the corporate interests in trouble, an over-exacting and tough regimentation in the area of underwriting could put the companys business interests in jeopardy, especially in the competitive landscape. Hence, there must be a judicious trade-off between these two functions.

184

Frequently Asked Questions in Management

PART - 6

INFORMATION TECHNOLOGY

185

Information Technology

186

Frequently Asked Questions in Management

IT - GENERAL
Question 1: Answer: What do you understand by Client-server Computing? Storage of commonly used data on server computer which client computer accessing it when needed and processing it to present in the way needed by the client/user. Client/server is a computational architecture that involves client process requesting service from server processes. Question 2: Answer: What are the characteristics of client/server architecture? The basic characteristics of client/server architectures are: Combination of a client or front-end portion that interacts with the user, and a server or back-end portion that interacts with the shared resource. The client process contains solution-specific logic and provides the interface between the user and the rest of the application system. The server process acts as a software engine that manages shared resources such as databases, printers, modems, or high-powered processors. Question 3: Answer: What is a thin client? A thin client is defined as one that does minimum amount of processing. The bulk of the application runs on the server. Eg: A personal computer node without any local storage like Hard disk or Floppy. What are the different types of servers? The common types of servers are a) File server - With a file server, the client passes requests for files or file records over a network to the file server b) Database server - In database servers, clients passes SQL (Structured Query Language) requests as messages to the server and the results of the query are returned over the network. c) Transaction server - In transaction servers, clients invoke remote procedures that reside on servers, which also contains an SQL database engine. d) Application server - Application servers are used to serve user needs, such as downloading capabilities, regulating electronic mail process etc. Question 5: Answer: Question 6: Answer: What is a Two-Tier Architecture? A two-tier architecture is where a client talks directly to a server, with no intervening server. It is typically used in small environments (less than 50 users). What do you mean by Three-Tier Architecture? The three-tier architecture has three-layers the Application-server, the client and the data server. Application-server-tier: In this tier the business-objects that implement the business rules are stored, and are available to the client-tier. This level forms the central key to solving 2-tier problems. This tier protects the data from direct access by the clients.

Question 4: Answer:

187

Information Technology

Client-tier: The client tier is responsible for the presentation of data, receiving user events and controlling the user interface as the actual business logic has been moved to an application-server. Data-server-tier: This tier is responsible for data storage. Besides the widespread relational database systems, existing legacy systems databases are often reused here. Question 7: Answer: Give an example of middleware? Client server sub-systems are implemented by components that must be capable of interacting with one another within a single machine (client/server) or across the network. An object request broken is a middle ware that enables an object that resides on a client to send a method that is encapsulated by an object that resides over the server. CORBA, COM, Java are widely used standards. What do you understand by Server Process? A server process (program) fulfills the client request by performing the task requested. Server programs generally receive requests from client programs, execute database retrieval and updates, and manage data integrity and dispatch responses to client requests. Also execute common or complex business logic. What is a Client Process? The client process is a program that sends a message to a server process (another program), requesting that the server perform a task (service). Client programs usually manage the user-interface portion of the application, validate data entered by the user, dispatch requests to server programs, and sometimes execute business logic. The clientbased process is the front- end of the application that the user sees and interacts with.

Question 8: Answer:

Question 9: Answer:

Question 10: What are the different models used in distributed computing? (CORBA, COM/DCOM) Answer: Distributed computing is any computing that involves multiple computers remote from each other. Each has a role in solving computation problems or information processing. CORBA, COM/D-COM Both are standards for distributed computing CORBA is promoted by Sun Microsystems where as COM/D-COM is promoted by Microsoft. Question 11: What is CORBA? Answer: CORBA stands for the Common Object Request Broker Architecture. It is a set of standard mechanisms for naming, locating, and defining objects in a distributed computing environment. It is a standard set by consortium led by Sun Microsystems

Question 12: What do you understand by the term Intranet? How is it different from the Extranet? Answer: Intranet is an organizations own internal network that uses the same services, when the organizations Intranet is made available to selected users outside its own network, it is known as Extranet.

Question 13: What is Distributed Processing? Answer: Distributed processing implies that processing will occur on more than one processor in order for a transaction to be completed. In other words, processing is distributed across two or more machines and the processes are most likely not running at the same time,

188

Frequently Asked Questions in Management

i.e. each process performs part of an application in a sequence. Often the data used in a distributed processing environment is also distributed across platforms. This will ensure faster thought put by better utilization of processing power. Question 14: What do you understand by embedded software? Answer: Software that is embedded in micro controllers which is involved automatically when the machine containing micro controller is started.

Question 15: What are the functions of the storage manager? Answer: The storage manager components provide the interface between the low level data stored in the database and the application programs and queries submitted to the system. The storage manager components include: i. ii. Authorization and integrity manager, which tests for the satisfaction of integrity constraints and checks the authority of users to access data. Transaction manager which ensures that the database remains in a consistent state despite system failures and those concurrent transaction executions proceed without conflicting. File manager which manages the allocation of space on disk storage and the data structures used to represent information stored on the disk.

iii.

Buffer manager, which is responsible for fetching data from disk storage into main memory, and deciding what data to cache in the memory. Question 16: What are Distributed Object Management Systems (DOMS)? Answer: DOMS provide a way of pulling diverse components together and managing their communications. DOMS functions as a single interface to manage the complexities of a heterogeneous environment, a uniform framework, based on standards and extensibility, to build, integrate, and deploy open distributed-computing applications and a method for creating location independence for client applications.

Question 17: What do you mean by Cooperative Processing? Answer: Cooperative processing is computing which requires two or more distinct processors to complete a single transaction. It is related to both distributed and client/server processing. Usually, these programs interact and execute concurrently on different processors.

Question 18: Differentiate LAN and WAN? Answer: A LAN (Local Area Network) is a network of computers that operate in a contained area. This could be a single floor or a small building. WAN (Wide Area Network) is a computer network that uses telecommunication links to join computers over a vast geographic area.

Question 19: What do you understand by TCP/IP, encoding, bandwidth? Answer TCP/IP is a protocol suit comprising of two different protocols Transmission Control Protocol and Internet Protocols, that could hook up dissimilar computers and transfer

189

Information Technology

information between them. IP handles the rating of packets and TCP provides reliable end-to-end service. Bandwidth: Frequency spectrum is a correction of component frequencies. Bandwidth is difference between highest and lowest range of component frequencies. On a line if frequency is too low it cannot overcome capacitance of line, if it is too high can be impeded by inductance. Every line has an upper limit and lower limit on the frequencies it can carry. This limit or range is called bandwidth. Encoding: Date to be carried from one place to another place has to be first stand in 0s and 1s. Then these 0s and 1s are to be connected into digital signal. This is called digitalto-digital conversation or encoding. Some times we need to convert an oral signal (voice in telephone conversation) into digital signal called A to D. Question 20: Explain the difference between Routers and Brouters. Answer: Routers: A Router accepts packets on at least two network interfaces, and forwards packets from one interface to another. Also needed for interconnection between multiple LANs running similar or multiple protocols. Brouters: Protocol dependent devices. When a Brouter receives a frame to forward to the remote segment, it checks to see if it recognizes the Network layer protocol. If the Brouter does, it acts like a router and finds the shortest path. If it doesnt, it acts like a bridge and forwards the frame to the next segment. It can act as both a bridge and a router. Question 21: What is Business Process Reengineering? Answer: Business Process Reengineering is a organizational process to critically analyze the business process with a view to simplify and modify then for better organizational effectiveness.

Question 22: How is CRM different from selling? Answer: CRM (Customer Relationship Management) encompasses all process of an organization to give a comprehensive profile of the customers across the organization. They include pre-sales, sales and service.

Question 23: What are the various ways of data security? Explain them briefly (like fire walls, encryption, PKI) Answer: Encryption: The initial message prepared by the sender is written as plaintext, which the sender converts into cipher text before the message is transmitted. The process of converting plaintext into cipher text is called encryption. The encryption process requires an encryption algorithm and a key. The process of recovering plaintext from cipher text is called decryption. There are two types of encryption: Symmetric and Asymmetric. The symmetric encryption requires a single key to be used by both the

190

Frequently Asked Questions in Management

sender and receiver to encrypt and decrypt the message. In asymmetric encryption the sender and receiver each one uses a set of two keys: a public key and a private key. Both the keys are mathematically related. The private key is kept secrete while the public key is exchanged or shared. If one key is used to encrypt the message then the other key can only be used to decrypt the message. This method is also called public key encryption. PKI: A PKI (Public Key Infrastructure) consists of using a combination of public key and private key for better security of the data. Data is encrypted with public key and decrypted using private key. Firewall: A firewall is a set of related programs, located at a network gateway server that protects the resources of a private network from users from other networks. Basically, a firewall, working closely with a router program, examines each network packet to determine whether to forward it toward its destination. Question 24: What are Neural Networks? Answer: Artificial Neural Networks are an attempt to replicate functioning of human brain. The functions of axioms, neurons and synapses of human brain is divided through Artificial Neural Networks. It is an attempt to develop machines that can think and learn like human beings.

Question 25. How do you sell software packages? Answer: By explaining the features of the product and also demonstrating how this can be effective for their business. Also telling them the savings they are likely to achieve and also understanding user requirements and explaining benefit at use of software.

Question 26: Tell us the points that should be considered during implementation of the product at the customers site? Answer: Implementation of product at customer site. - Site preparation - Hardware & Software preparation - Installation - Demonstration - Training the users - Making available documentation/help Question 27: How will you design a classroom ergonomically? Answer Proper seating arrangements Proper ventilation Proper lighting arrangements Proper placement for equipment. Question 28: Distinguish between ergonomics and usability?

191

Information Technology

Answer:

Ergonomics deals with environment. Usability refers to software.

Question 29: What is .net framework? Answer: Multi-language development platform developed by Microsoft.

192

Frequently Asked Questions in Management

INTERNET & WEB TECHNOLOGIES


Question 1: Answer: What is a URL? URL stands for Uniform Resource Locator. Each web page has a unique address called uniform resource locator. It consists of the computer and path to the page on the computer where that page is available. URL is an identifier to access a specific website on Internet otherwise called address of a website. What is a browser? Name two of the popular browsers? A browser is software tool using which one can view web pages and move from one web page to another web page in World Wide Web. Internet Explorer and Netscape are two of the most popular browsers. Question 3: Answer: What do you understand by web server? A web server is a network server with a pool of information that can be accessed by the users of the network. It is used to publish information on the Internet. What is a newsgroup? A newsgroup is a bulletin board on which readers interested in that newsgroups particular topic can read and respond to messages posted by other users. What do you understand by CGI and PERL? CGI is a short form for Common Gateway Interface. This is a program that runs on the server side (web site). It provides interactivity with the web sites, such as guest book etc. PERL is a scripting language, which is easy to use and mostly used for creating CGI scripts. It looks similar to C and Unix Shell scripting language. What is the difference between a website and a portal? A web site is a web page or a group of web pages that provide information about a person, an institution etc. It primarily focuses on content and information about the subject. A portal is also a web site, but has a lot of interactivity and links (to news, weather, and other sites that have relevant information) and enables processing of transactions on the Internet. All E-business sites are portals. What is the advantage of using web forms over using windows forms? Web forms enable internet-based applications whereas windows form does not have this capability. What do you understand by Web Services and what are the benefits? Web services are services that are made available from a business's Web server for Web users or other Web-connected programs. Providers of Web services are generally known

Question 2: Answer:

Question 4: Answer:

Question 5: Answer:

Question 6: Answer:

Question 7: Answer:

Question 8: Answer:

193

Information Technology

as Application Service Providers. Web services range from such major services as storage management and Customer Relationship Management (CRM) down to much more limited services such as furnishing of a stock quote and checking of bids for an auction item. Question 9: Answer: What are payment gateways? What is their importance in e-Business applications? A payment gateway is a software application that processes payment instructions in an ecommerce site. It is a service that provides connectivity among merchants, customers, customers and financial networks to process authorization and payments.

194

Frequently Asked Questions in Management

SYSTEM ANALYSIS AND DESIGN & SOFTWARE ENGINEERING


Question 1: Answer: What do you understand by CASE tool? Give some examples. CASE tools stand for Computer Aided Software Engineering tools. These tools use computers assisted methods to organize and control the development of software. Example 1. ORACLE Designer, 2. Rational Rose. Please explain various activities involved in SDLC. The different stages of SDLC are: Initial investigation and feasibility study System Analysis and Requirement Determination/Definition System Design System Development and Testing System Documentation User Training and System Implementation Maintenances and System Review Question 3: Answer: What are the stages in system development life cycle? (a) Preliminary investigation (b) Determination of requirements (c) Prototype development (d) System Design (e) System Development (f) System Testing (g) Implementation Question 4: Answer: How do you gather system requirements from end-users? System requirement analysis begins with customer communication. Leading questions should be asked that will underline the objectives and scope clearly. Attempt should be made to understand the current approach and the problems therein. Each session with the customer must be documented and the written document must be reviewed along with the customer. After due corrections and sign off, this document forms the basis for the subsequent design and testing of the software. How would you incorporate usability in software? Usability refers to users friendliness. Can be measured by four characteristics. Effort is required to learn the product, operate, prepare input and interpret output of a program. This can be achieved by placing user in control, reducing users memory load, and make interface consistent. Usability starts from system requirement stage and must be kept in mind during all phases of software development lifecycle.

Question 2: Answer:

Question 5: Answer:

195

Information Technology

Question 6: Answer:

How do you conduct the feasibility study for an IT Project? A feasibility study is carried out for the proposed system before it is started. There are four primary areas of attention when feasibility is to be ascertained. Economic Feasibility An evaluation of development cost weighted against the ultimate income benefit derived from the developed system on product. Technical Feasibility A study of function, performance, and constraints that may affect the ability to achieve an acceptable system. Legal Feasibility A determination of any infringement, violation, or liability that could result from development of the system. Feasibility study is conducted to evaluate the economic and technical viability of the project. Economic feasibility is assessment of benefits (returns) over the investment made. Technical viability is the assessment of implementability of the project from technical and functional prospective.

Question 7: Answer:

What are the types and techniques of software testing? There are two prominently techniques in software testing Black box testing and White box testing. White box testing focuses on the program control structure. Test cases are designed to ensure that all statements in a program have been executed at least once during the testing and all the logical conditions have been exercised. Black box tests are designed to uncover errors in functional requirements, without regard to the internal workings of a program. They focus on the domain of the software. There are four prominent types of software testing. Each focuses on the different stages of a software project, namely, system engineering, requirements, design and coding. There needs to be a test to check each of these stages. Unit testing focuses on the objective of the module developed. Taking the module description as a guide, important control paths are tested to uncover errors within the boundary of the module. Unit tests are normally white box oriented. This is code level testing. Integration test: Once all the modules of a software system are developed, an integration test needs to be performed. The focus is on the design and construction of the software architecture. Validation test: Requirements established as part of software requirements analysis (SRA) are validated against the software that has been designed and developed. System test: All the tests described above cover the software and ensure that the software is error free and is according to the users requirements. Issues such as hardware, security, performance etc., are some of the issues that also need to be addressed and the system test covers these aspects. System test is actually a group of tests, which are prominently, recovery test, security test, stress test (pushing the software to the limits) and performance test.

Question 8:

Explain Black box and White box testing.

196

Frequently Asked Questions in Management

Answer:

Block box testing applies to tests that are conducted at the software interface. Used to demonstrate that software functions are operational, input is properly accepted and output is correctly produced, and integrity of external information is maintained. Examines some fundamental aspect of the system and not internal logical structure. White box testing is for close examination of procedural detail. Logical patterns are tested using test cases. Status of program examined at various points. Also test internal data structures.

Question 9: Answer:

What is Alpha and Beta testing? When software is built for use by a number of customers, Alpha and Beta testing is used. Alpha testing is done at developers site, by a customer. Testing is done in natural setting, controlled environment and developer notes the errors. Beta testing is conducted at one or more customer sites. Developer is not present. Beta testing is live. Customer records all the problems and informs developer. Developer makes changes and then releases software.

Question 10: How do you estimate efforts involved in the entire project? Answer: For estimation of efforts in development and testing, techniques like Quantitative Methods (Eg. FP analysis) are used. For other stages like requirement identification and implementation, efforts are estimated on the basis of time and manpower needed.

Question 11: Can you explain the SEIs Capability Maturity Model? What is the significance of the various levels? Answer: SEIs CMM defines the level of maturity of software process is an organization. The Capability Maturity Model of SEI has five levels: Initial, Repeatable, Defined, Managed and Optimizing. Organizations in the first level are those that have rudimentary predictability of schedules and costs. This level is called Initial. In the second level the organization achieves a stable process with a repeatable level of statistical control by initiating a rigorous project management. In the third level the organization would have Defined the process as a basis for consistent implementation and better understanding. At level four (Managed) the organization has initiated comprehensive process measurements and analysis. This is when the significant quality improvements begin. The optimizing level is reached by an organization when it has a foundation for continuing improvement and optimization of the process. Question 12. Why is CMM used in IT specific companies? Answer: Because CMM defines key activities required at different levels of process maturity.

Question 13: What are the differences between ISO and CMM? Answer: ISO Checks for adherence of processes already defined (Yes / No).

197

Information Technology

CMM Assesses the level of maturity of the organization (Quantitative). ISO The primary focus of ISO is the customer supplier relationship, to reduce the risk in choosing a supplier. ISO is written for a wide range of industries, other than software. Documents are more abstract and identify only the minimal requirements for a quality system. It has a broad scope that encompasses hardware, software, processed materials, and services. The ISO 9000s concept is to follow a set of standards to make success repeatable. It means that some basic practices are in place and the challenge is only to maintain certification. Question 14: How do you conduct a System Audit? Answer: Every stage of the software development life cycle needs to be subject to audit, mainly to ensure that projects are adhering to quality standards and are not deviating due to project pressures. The following will ensure that all the stages of a project are duly covered by the audit process. Software Requirements Review: This is to ensure the adequacy of requirements. Preliminary Design Review: To evaluate the adequacy of the preliminary review. Critical Design Review: To determine that the SDS (Software Design Specifications) in fact satisfies the SRS. Software Verification and Validation Review: To evaluate the adequacy and completeness of the software verification and validation plan. Functional Audit: An audit prior to software delivery to verify that the requirements were met. Physical Audit: An audit prior to software delivery to determine that the software and documentation are consistent with the design and ready for delivery. In-Process audits: A statistical sample of the development process is audited to verify the consistency of the code versus the design and interface specifications, the design versus the requirements and test plans versus requirements. Managerial Reviews: Independent reviews are conducted on the execution of the quality assurance plan. CMM In CMM, the focus is on the supplier to improve the internal software process. CMM is written specifically for the software industry. The document is very detailed (about 500 pages) and elaborate and describes the software process in detail. It is specific to the software development.

The CMM emphasizes on achieving "maturity" and improving its process continuously. It emphasizes on continuous improvement, even at the last level.

198

Frequently Asked Questions in Management

Question 15: How do you do risk analysis for software projects? How do you mitigate the risks? Answer: Risk analysis evaluates the factors/issues, which affect completion of the project as per the committed deliverables line and cost. Each factor is analyzed and pro-actively steps are identified to prevent the failures in advance. The risk components for a software project can be classified as the following: Performance risk - the degree of uncertainty that the product will meet its requirements and useful to the end user. Cost risk - the degree of uncertainty that the project budget will be maintained. Support risk - the degree of uncertainty that the software will be easy to correct, adapt and enhance. Schedule risk - the degree of uncertainty that the product will be delivered on time.

199

Information Technology

OPERATING SYSTEM
Question 1: Answer: What is an operating system? An operating system (OS") is a program that is initially loaded into the computer by a boot program. The OS then manages all the other programs in a computer. The other programs, called applications programs make use of the operating system by making requests for services through a defined application program interface (API). In addition, users can interact directly with the operating system through an user interface such as a command language or a graphical user interface (GUI) What do you mean by a Network Operating System? A network operating system (NOS) is an operating system that is designed primarily to support computers (which may include workstations, PCs, and in some instances, other devices like printers etc.,) that are connected on a local area network (LAN). A network operating system provides printer sharing, common file system and database sharing, application sharing, and the ability to manage a network name directory, security, and other housekeeping aspects of a network. Some examples include Artisoft's LANtastic, Novell's NetWare, and Microsoft's Windows NT. Question 3: Answer: What is a deadlock in OS? Deadlock is a situation where two or more processes get into a state whereby each is holding a resource and the other is requesting. Since a request operation blocks the caller until the resource becomes allocated, no process will ever have the desired resource allocated to it and both will remain in the blocked state forever. What do you mean by process? A process is nothing but a running application on the operating system. What do you understand by thread? It is possible to design an application in such a way that it can simultaneously handle multiple functions within itself, to speed up the execution of the application. When such functionality is required, threads need to be created. Each thread handles a function and all threads run simultaneously. What is difference between UNIX and LINUX? UNIX is a proprietary and close stand (licensed) system. LINUX is an open source system. What do the following UNIX commands do? ls, pwd, ps, cat ls - Lists files in a directory (similar to dir of DOS); pwd - displays current working directory; ps - shows the list of processes running on the machine; cat - concatenates files.

Question 2: Answer:

Question 4: Answer: Question 5: Answer:

Question 6. Answer:

Question 7: Answer:

200

Frequently Asked Questions in Management

DBMS
Question 1: Answer: Differentiate between Network, Hierarchical and Relational database model. Network model Data in the network model are represented by collection of records and relationships among data are represented by links, which can be viewed as pointers. The records in the database are organized as collections of arbitrary graphs. Hierarchical model The hierarchical model is similar to the network model in the sense that data and relationships among data are represented by records and links, respectively. It differs from the network model in the only aspect that the records here are organized as collections of trees rather than arbitrary graphs. Relational model The relational model uses a collection of tables to represent both data and the relationships among those data. Each table has multiple columns and each column has a unique name. Question 2: Answer: What is an ER Diagram? An ER diagram is a representation of the relationships of tables within a database. This diagram helps in understanding the database design. What is an Entity, Entity sets and Relationship set? An entity is a thing or an object in the real world that is distinguishable from all other objects. For example, each person in an enterprise is an entity. An Entity set is a set of entities of the same type that share the same properties or attributes. The set of all persons who are customers of a bank can be defined as the entity set customer. A relationship is an association among several entities. Question 4 Answer: What is data dictionary? An internal database structure that captures a description of the objects that are stored within the database. This is the key to having database like ORACLE, find information that you store within the database. The description about the data in a database is called Metadata. In data dictionary, Metadata are stored. Question 5: Answer: Explain database schema. A database schema defines a database containing different relations and each relation may have different set of attributes. A database schema is, in a sense a database or a group of relations for a specific application. We can have more than one schema in a system residing simultaneously but with different applications.

Question 3: Answer:

201

Information Technology

Database schema definition consists of three major components i. ii. iii. Question 6: Answer: Schema name Authorization or name of the owner Schema elements

Differentiate among the three levels of abstraction (Physical or internal level, Logical or conceptual level, View or external level). Physical level: The lowest level of abstraction describes how the data are actually stored. Logical level: The next higher level of abstraction describes what data are stored in the database, and what relationships exist among these data. Used by database administrators, who must decide what information is to be kept in the database. Programmers use a programming language to work at this level of abstraction. View level: The highest level of abstraction describes only part of the entire database. The system may provide many for the same database. For example tellers in a bank see only that part of a database that has information on customer accounts; they cannot access information concerning salaries of employees.

Question 7: Answer:

What is data independence? The ability to modify a schema definition in one level without affecting a schema definition in the next higher level is called data independence. What do you mean by Physical data independence and Logical data independence? Physical data independence is the ability to modify the physical schema without causing application programs to be rewritten. Modifications at the physical level are occasionally needed to improve performance. Logical data independence is the ability to modify the logical schema without causing application programs to be rewritten. Modifications at the logical schema are necessary whenever the logical structure of the database is altered.

Question 8: Answer:

Question 9: Answer:

Define Normalization and the different forms of Normalization Normalization is a formal process for deciding which attributes should be grouped together in a relation. It is normally a tool to validate and improve a logical design, so that it satisfies certain constraints that avoid unnecessary duplication of data. It is the process of decomposing relations with anomalies to produce smaller, well-structured relations. i. ii. iii. A relation R is in First Normal Form (1NF) if and only if all underlying domains contain atomic values only. A relation R is in Second Normal form (2NF) if and only if it is in 1NF and every non-key attribute is fully dependent on the primary key. A relation R is in Third Normal Form (3NF) if and only if it is in 2NF and every nonkey attribute is nontransitively dependent on the primary key.

202

Frequently Asked Questions in Management

iv. v.

A relation R is in Boyce/Codd Normal Form (BCNF) if and only if every determinant is a candidate key. A relation R is in Fourth Normal Form (4NF) if and only if whenever there exists an MVD (Multivalued dependency) in R, then all attributes of R are also functionally dependent on A A relation R is in Fifth Normal Form (5NF) also called as Projection-Join normal form (PJ/NF) if and only if every join dependency in R is implied by the candidate keys of R.

vi.

Question 10: Tell us some thing about SQL. Answer Structured Query Language (SQL) is an industry standard language for defining and manipulating data contained in a relational database. SQL, like non-procedural language provides commands for a variety or tasks including:1. 2. 3. 4. DDL Data Definition language e.g. create, alter (database, table, view etc.) DML Data Manipulation Language e.g. (insert, update, delete) DCL Data Control Language e.g. (grant, revoke, commit, rollback) DRL Data Retrieval Language e.g. (select) Question 11: What is table in SQL? Answer: Its the organizing principle in a database, comprises of a row/column arrangement of data values.

Question 12: What are the commands under DDL, DML and TCL? Answer: DDL: Commands for defining relation schemas, deleting relations, creating indices and modifying relation schemas. DML: Insert tuples, delete tuples, and modify tuples. TCL: Commit a transaction, rolling back a transaction, and creating a Savepoint. Question 13: What is functional dependency? Answer: Functional dependency is a constraint between two attributes or two sets of attributes. Given a relation R, attribute Y of R is functionally dependent on attribute X of R if and only if each x value in R has associated with it precisely one Y value in R.

Question 14: What are multivalued dependencies? Answer: The type of dependency that exists when there are at least three attributes in a relation (A, B, C) and for each value of A there is a well defined set of values of B and a well

203

Information Technology

defined set of values of C. However the set of values of B is independent of set C and vice versa. Question 15: Define super key, candidate key and primary key. Answer: A super key is a set of one or more attributes taken collectively, that allows us to identify uniquely an entity in the entity set. The super keys for which no proper key is a subset is a super key, such minimal super keys are called candidate keys. The term primary key is used to denote a candidate key that is chosen by the database designer as the principle means of identifying entities with in an entity set.

Question 16: What is the difference between composite attributes and multivalued attributes? Answer: Composite attributes can be divided into sub parts (that is other attributes). For example customer-name could be structured as a composite attribute consisting of first-name, middle-initial, and last-name. Using composite attributes in a design schema is a good choice if a user wishes to refer to an entire attribute on some occasions and to only a component of the attribute on some occasions. Multi-valued attributes are attributes, which have a set of values for a specific entity. For example the dependent name in the entity set employee. Question 17: What is specialization, generalization and aggregation? In what situations do you use them? Answer: An entity set may include sub groupings of entities that are distinct in some way from other entities in the set. For instance, a subset of entities within an entity set may have attributes that are not shared by all the entities in the entity set. The mean of representing these groupings is called specialization. The refinement from an initial entity set into successive levels of entity sub groupings represent a top-down design process in which distinctions are made explicit. The design process may also proceed in a bottom-up manner in which multiple entity sets are synthesized into a higher-level entity set based on common features. This commonality can be expressed by generalization, which is a containment relationship that exists between a higher-level entity set and one or more low level entity sets. Aggregation is an abstraction through which relationships are treated as higher level entities. Question 18: What is the difference between the Primary key constraint and the unique constraint? Answer: When column is specified as Primary Key that column is also assumed to be NOT NULL even if NOT NULL is not explicitly stated. With Unique, NOT NULL has to be specified. Unique constraint can contain NULL values. Question 19: What do you mean by integrity constraints? Answer: This is a mechanism used by the database to prevent invalid data entry into the table. It is nothing but enforcing rules for the columns in a table.

204

Frequently Asked Questions in Management

Question 20: What do you mean by referential integrity? Answer: Referential integrity is an integrity constraint of a database, which ensures that the contents of common attributes in two or more relations are consistent. In order to implement this constraint Foreign key is used. Foreign key is a set of attributes of one relation whose values are required to match values of some key attributes of another relation.

Question 21: What are dominant and subordinate entities? Answer: If the existence of entity x depends on the existence of entity y, then x is said to be existence dependent on y. if y is deleted so is x. Entity y is said to be a dominant entity and x is said to be a subordinate entity. Example: loan-payment relationship.

Question 22: Which are the two types of locks? Answer: Shared locks Shared locks also called as S locks or read locks allow other transactions to read (but not update) record or other resource. A transaction should place a shared lock on a record or a data resource when it will only read but not update that record. Placing a shared lock prevents another user from placing an exclusive lock on that record. Exclusive locks Also called X locks or write locks prevent another transaction from reading (and therefore updating) a record until it is unlocked. A transaction must place an exclusive lock on a record when it is about to update that record. Placing an exclusive lock on a record prevents another user from placing any type of lock on that record. Question 23: What is deadlock? Answer: Locking solves the problem of erroneous updates but may lead to another problem called deadlock, an impasse that results when two or more transactions have locked a common resource, and each must wait for the other to unlock that resource.

Question 24: What are triggers? Answer: Database triggers are procedures that are stored in the database and are executed when contents of a table are changed. Oracle allows users to dyne procedures and executes them when an insert, delete or update is issued against a table from SQL * plus or through an application.

Question 25: What are different types of triggers? Answer: (a) Row trigger Row trigger is fixed each time the table is affected by a triggering statement e.g. if an UPDATE statement updates multiple rows, triggers are fixed for each row affected. (b) Statement level trigger Row trigger fixed once on behalf of triggering statements, independent of number of rows, affected. (c) Before Vs After Triggers

205

Information Technology

While defining a trigger, one can specify trigger timing, when the action is to be executed in relation to the triggering statement. - Before trigger execute triggering action. - Before triggering statement. After triggers: Fixed after executing triggering statement combinations: Before row trigger, before statement trigger, after statement trigger, after row trigger can be related. Question 26: What are distributed databases? Answer: A single logical database that is spread physically across computers in multiple locations that are connected by a data communications link. It is still centrally administered as a corporate resource while providing flexibility and customization. A distributed database requires multiple database management systems running at each remote site.

Question 27: What is a weak entity set? Define Identifying relationship? Answer: An entity set not processing sufficient attributes to form a primary key is termed as a weak entity set. The relationship that associates the weak entity set with an owner (strong entity set) is the identifying relationship.

206

Frequently Asked Questions in Management

ORACLE AND PL/SQL


Question 1: Answer: What do you mean by SQL*Plus and where does it come from? SQL*Plus is a command line SQL and PL/SQL language interface and reporting tool that ships with the ORACLE Database Client and Server. It can be used interactively or driven from scripts. SQL*Plus is frequently used by DBAs and Developers to interact with the ORACLE database. SQL*Pluss predecessor was called UFI (User Friendly Interface). UFI was included in the first releases of ORACLE, its interface was extremely primitive and anything but user friendly. What do you mean by PL/SQL and what is it used for? PL/SQL is Oracles Procedural Language extension to SQL. The PL/SQL language includes object oriented programming techniques such as encapsulation, function overloading, information hiding (all but inheritance). PL/SQL is commonly used to write data-centric programs to manipulate data in an ORACLE database. What is an index? How is it created? Adding indexes to the database enables SQL to use the direct access method, by creating a tree like structure for quick access of records in a database. An index is created by the SQL command create index. What do you understand by a join? join is a way retrieving data from two or more tables by matching values in columns. Suppose there are two tables - employees and salary and it is required to retrieve the salary of employee from the salary table based on say employee_id, which is common to both employee and salary tables. SELECT employee.employee_name, salary.salary FROM employee, salary WHERE employee.employee_id=salary.employee_id. This will fetch the employee name and salary of the employee from employee and salary tables respectively, based on the common attribute (column) employee_id. This feature is called join. Question 5: Answer: Question 6: Answer: What SQL command does the DBA use to provide users with permissions? GRANT What is the difference between a table and view in a database? The tables that are defined in a database schema are called base tables because they contain the basic data of the database. A view is an alternative way of looking at the data in one or more tables. It's derived from one or more real tables whose data is physically stored in the database. The definition of a view is stored in the database, but no separate copies of data are associated with this stored definition. A view looks exactly like any

Question 2: Answer:

Question 3: Answer:

Question 4: Answer:

207

Information Technology

other database table, and is operationally quite similar to any other table. There is no restriction on querying through views, but a few on modifying them. For example we may have a table of employees with employee_id, name and designation, and a salary table with employee_id and salary. We can create a view called emp_salary by picking employee_id from employee table and salary from the salary table as follows: CREATE VIEW emp_salary AS SELECT employee.employee_name, salary.salary FROM employee, salary WHERE employee.employee_id=salary.employee_id. Question 7: Answer: What is ORACLE Forms and what is it used for? ORACLE Forms is a 4GL Rapid Application Development (RAD) environment. Forms Builder is used to create applications to enter, access, change, or delete data from ORACLE (and other) databases. The Forms Runtime environment is required to execute compiled Forms modules. Forms can also be deployed across the Web using the ORACLE Internet Application Server (iAS) Forms Services. ORACLE Forms is part of the ORACLE Internet Developer Suite (iDS). It was previously called SQL*Forms. Question 8: Answer: How can a users ORACLE password be changed? Issue the following SQL command: ALTER USER <username> IDENTIFIED BY <new_password> / From Oracle8 you can just type "password" from SQL*Plus, or if you need to change another users password, type "password user_name". Question 9: Answer: Can dynamic SQL be executed from Forms? Yes, use the FORMS_DDL built-in or call the DBMS_SQL database package from Forms. Eg: FORMS_DDL(INSERT INTO X VALUES ( || col_list || )); Just note that FORMS_DDL will force an implicit COMMIT and may de-synchronize the ORACLE Forms COMMIT mechanism Question 10: Can both PL/SQL and JAVA be used to write procedures and triggers? Differentiate between the two language environments. Answer: Internally the ORACLE database supports two procedural languages, namely PL/SQL and JAVA. Many ORACLE applications are based on PL/SQL and it would be difficult for ORACLE to ever stop support to PL/SQL. Many enhancements are still being made to PL/SQL. For example, ORACLE 9iDB supports native compilation of Pl/SQL code to binaries. PL/SQL and JAVA appeal to different people in different job roles. The following table briefly describes the difference between these two language environments: PL/SQL

208

Frequently Asked Questions in Management

i. ii. iii. i.

Data centric and tightly integrated into the database Proprietary to ORACLE and difficult to port to other database systems Data manipulation is slightly faster in PL/SQL than in JAVA Open standard, not proprietary to ORACLE

JAVA ii. Incurs some data conversion overhead between the Database and JAVA type systems Question 11: What are the differences among DDL, DML and DCL commands? Answer: DDL is Data Definition Language statements. Some examples: CREATE - to create objects in the database ALTER - alters the structure of the database DROP - delete objects from the database TRUNCATE - remove all records from a table, including all spaces allocated for the records COMMENT - add comments to the data dictionary GRANT - gives users access privileges to database REVOKE - withdraw access privileges given with the GRANT command DML is Data Manipulation Language statements. Some examples: SELECT - retrieve data from a database INSERT - insert data into a table UPDATE - updates existing data within a table DELETE - deletes all records from a table, the space for the records remain CALL - call a PL/SQL or JAVA subprogram EXPLAIN PLAN - explain access path to data LOCK TABLE - control concurrency DCL is Data Control Language statements. Some examples: COMMIT - save work done SAVEPOINT - identify a point in a transaction to which you can later roll back ROLLBACK - restore database to original since the last COMMIT SET TRANSACTION - Change transaction options like what rollback segment to use Question 12: What is the procedure for generating primary key values for a table? Answer: Create your table with a NOT NULL column (say SEQNO). This column can now be populated with unique values: SQL> UPDATE table_name SET seqno = ROWNUM; or use a sequences generator: SQL> CREATE SEQUENCE sequence_name START WITH 1 INCREMENT BY 1;

209

Information Technology

SQL> UPDATE table_name SET seqno = sequence_name.NEXTVAL; Finally, create a unique index on this column. Question 13: Can one drop a column from a table? Answer: From ORACLE8i one can DROP a column from a table. Look at this sample script, demonstrating the ALTER TABLE table_name DROP COLUMN column_name; command. Other workarounds: i. SQL> update t1 set column_to_drop = NULL; SQL> rename t1 to t1_base; SQL> create view t1 as select <specific columns> from t1_base; ii. SQL> create table t2 as select <specific columns> from t1; SQL> drop table t1; SQL> rename t2 to t1; Question 14: Can one rename a column in a table? Answer: No, this is listed as Enhancement Request 163519. Some workarounds: i. Use a view with correct column names... rename t1 to t1_base; create view t1 <column list with new name> as select * from t1_base; ii. Recreate the table with correct column names... create table t2 <column list with new name> as select * from t1; drop table t1; rename t2 to t1; iii. Add a column with a new name and drop an old column... alter table t1 add ( newcolame datatype ); update t1 set newcolname=oldcolname; alter table t1 drop column oldcolname Question 15: How does one eliminate duplicate rows from a table? Answer: Choose one of the following queries to identify or remove duplicate rows from a table leaving unique records in the table: Method 1 SQL> DELETE FROM table_name A WHERE ROWID > ( SELECT min(rowid) FROM table_name B WHERE A.key_values = B.key_values); Method 2

210

Frequently Asked Questions in Management

SQL> create table table_name2 as select distinct * from table_name1; SQL> drop table_name1; SQL> rename table_name2 to table_name1; Method 3 SQL> delete from my_table where rowid not in( SQL> select max(rowid) from my_table SQL> group by my_column_name ); Method 4 SQL> delete from my_table t1 SQL> where exists (select x from my_table t2 SQL> where t2.key_value1 = t1.key_value1 SQL> and t2.key_value2 = t1.key_value2 SQL> and t2.rowid > t1.rowid); Note 1: If you create an index on the joined fields in the inner loop, you, for all intents purposes, eliminate N^2 operations (no need to loop through the entire table on each pass by a record). These will speed-up the process. If you are comparing NOT-NULL columns, use the NVL function. Remember that NULL is not equal to NULL. This should not be a problem as all key columns should be NOT NULL by definition.

Note 2:

Question 16: Can one pass an object/table as an argument to a remote procedure? Answer: The only way the same object type can be referenced between two databases is via a database link. Note that it is not enough to just use the same type definitions. For example: -- Database A: receives a PL/SQL table from database B CREATE OR REPLACE PROCEDURE pcalled(TabX DBMS_SQL.VARCHAR2S) IS BEGIN -- do something with TabX from database B null; END; / -- Database B: sends a PL/SQL table to database A CREATE OR REPLACE PROCEDURE pcalling IS TabX DBMS_SQL.VARCHAR2S@DBLINK2; BEGIN pcalled@DBLINK2(TabX); END;

211

Information Technology

/ Question 17: What is DLL? Answer: Dynamic Link Library. A Dynamic Link Library (DLL) is a collection of small programs, any of which can be called when needed by a larger program that is running in the computer. The small program that lets the larger program communicate with a specific device such as a printer or scanner is often packaged as a DLL program (usually referred to as a DLL file).

Question 18: Explain DLL in Visual Basic. Answer: Stands for Dynamic Link Library. DLLs can be called from any language. A DLL is loaded at runtime and need not be linked to any application. That is how name DLL comes in. Contents of DLL are linked to your application dynamically at runtime and not at design time.

Question 19: What are forms? Answer: The visual part of the application with which the user interacts is called the interface. Forms & Controls the building blocks used to create the interface. Visual Basic forms all objects that have properties (define appearance), Methods (define behaviour) and Events (define interaction).

Question 20: Name the file, which is created when a form is executed. Answer: FRX is created. FRX is a binary data file used to store binary information for the FRM file with the same filename.

212

Frequently Asked Questions in Management

DATA WAREHOUSING AND MINING


Question 1: Answer: What is the difference between data warehousing, data mining and Business Intelligence? Data Warehousing Repertoire of Raw & Processed data Data Mining Tools to extract needed data/information Business Intelligence Usage of tools for better business decision-making Question 2: Answer: How is data mining different from data warehousing? Data warehouse is the data (meta/ fact/ dimension / aggregation) and the process managers (load / warehouse / query) that make information available, enabling people to make informed decisions. In its simplest perception a data warehouse is no more than key pieces of information gathering. Whereas data mining deals with the discovery of hidden knowledge, unexpected patterns and new rules from large databases. Question 3: Answer: What are the benefits derived by implementing business intelligence solutions? Business Intelligence solutions connect people to their business and create an information environment that makes it easy for people to get the reports they need in the context of their day-to-day activities. They provide an accessible means of analyzing the business and getting to the bottom of what is behind trends and anomalies. The BI tools offer a reliable focused barometer of how well the business is performing so corrective actions can be taken in a timely manner. How do you go about building a data warehouse? The process of building a data warehouse is as follows: a) Identify the required types of queries or most frequently raised queries b) Identify the sources of these data elements c) Define the mode/frequency of updation/maintenance of data warehouse. Question 5: Answer: What are the steps involved in populating a data warehouse? The steps involved are:a) Extract and load the data. b) Claim and transforms the data into a form that can cope with large data volumes and provide good query performance. c) Backup and archive data. d) Manage queries and direct them to appropriate data source. Question 6: Answer: What do you mean by data cleansing? A data warehouse integrates data from its identified sources. Often those sources don't match in terms of descriptive attributes and in many cases even in terms of unique identifiers. Data cleansing is required to evaluate at all the data coming from different sources and convert them to a single format before they are passed to a data warehouse.

Question 4: Answer:

213

Information Technology

Question 7: Answer:

What is a data mart? Data mart hives off a subject of data in warehouse into a separate database, possibly at a different location. Data marts are created: a) To speedup queries by reducing volume. b) To structure data as required by user access. c) To partition data. d) To segment data into different hardware platforms.

Question 8: Answer:

How is data mart different from a data warehouse? Data warehouse is an enterprise wide solution where as Data mart is a departmental/ specific business application solution. Data mart is a smaller version of data warehouse.

214

Frequently Asked Questions in Management

C
Question 1: Answer: Question 2: Answer: What is the variable that represents the program name in a C program? argv(0) - This represents the program name. What is the iterative statement in C that uses a counter? The for statement is typically used for counter based operations. For example if a program is supposed to print the first ten natural numbers, then the ideal statement to be used is a for. In this case for(counter=0; counter<10; counter=counter+1) printf("%d", counter); Question 3: Answer: What are stdin and stdout? stdin is a pointer, which refers to terminal (screen) input. stdout is a pointer that refers to terminal (screen) output. Question 4: Answer: Question 5: Answer: How do I read a single character from the screen? getchar() reads a character from stdin, that is the terminal. How do you declare command line arguments in a C program? Consider the following example: C>copy file1 file2 The program name and the arguments to it are available to a C program during runtime by way of what are called command line arguments. These are provided to the program as arguments to the main line of the program, as given below:maint(int argc, char*argv[ ].) The first is the count of the argument list (argc), while the second is the list of argument strings (argv[ ]) In the above example, argc is 3. argv[0] is "copy", argv[1] is "file1" and argv[2] is "file2" Question 6: Answer: What are the fundamental data types used in a C program? The fundamentals data types are char, int, float, double, void and enum. The first four data types have modifiers short, long, signed, which define the amount of storage avoidable to the variable e.g. short int, long double, unsigned clear, unsigned short int etc.

215

Information Technology

Question 7:

a. Any array of characters can be called a string. a) A string is an array of characters. Which of these is correct? Why?

Answer:

a. b.

Wrong. An array of characters can be called a string only if it ends with a null \0 character. Correct

Question 8:

If I have to read data into the following variables from terminal, what should be the function used? How? eng_marks This can be achieved by the following: #include <string.h> void main() { int eng_marks = 0; printf ( \n Enter English Marks > ); scanf (%d, & eng_marks); } The function scanf() is used to do the formatted read of data from the terminal into the variables.

Answer:

Question 9: Answer: Question10: Answer:

How do you declare a function f() that returns a pointer to a variable of type double? double *f(); What is the difference between an array int x[5], pointer int *x ? In the first case, x refers to a pre-allocated memory for 5 integers. In the second case, x is just declared as a pointer to integer. In order to use this to refer to an array of integers, memory first needs to be allocated and the pointer to that memory needs to be assigned to x.

Question 11: Does the size of a pointer depend on the variable it is pointing to? Answer: No. A pointer contains just an address. It should be an unsigned, integer value.

Question 12: Is it correct to say that arguments to functions are called by value in C? Answer: Yes. When a call f(x) is made to a function f() we are actually sending the value of x to the function only. Any changes made to x in the function will only be reflected in the function locally. When the control returns to the calling function x will have the same value as that sent to the function f(x).

216

Frequently Asked Questions in Management

Question 13: What is a structure? Answer: A structure provides a way to define a collection of data having similar attributes. Consider, for example all students in a school. Each student has an enrolment no, name, and marks in all subjects. C provides a way to group such a set of data. This is called a structure. It is represented by the key word, 'struct'. A struct is represented as follows: struct students { char student_name[25]; int mat_marks; int phy_marks; int che_marks; } st; Question 14: Consider the structure below. struct students { char student_name[25]; int mat_marks; int phy_marks; int che_marks; }; struct students *pst; struct students stu; How do I refer phy_marks for a student represented in pst? How do I refer phy_marks for a student represented in st; Answer: pst->phy_marks stu.phy_marks Question 15: What is a union? Answer: A union is a group of variables that share the same address.

Question 16: Why doesnt a programs memory usage go down when I free memory? Answer: Most implementations of malloc/free do not return freed memory to the operating system.

Question 17: How can I invoke another program from within a C program? Answer: Use system()

217

Information Technology

C++
Question 1: Answer: What are the main characteristics of object oriented technology? Abstraction Encapsulation Polymorphism Inheritance Question 2: Answer: Explain concept of Component Object Modeling. COM is a specification developed by Microsoft in component-based software development. What is OOP? In Object Oriented Programming (OOP) programs are organized in a way that mirrors the way objects are organized. In OOP programs are organized into elements called classes. These classes are used to create objects. Two aspects define classes, one is attribute and the other is behaviour. Objects are created from class. They are also known as instances. Classes are connected to one another in such a way that one inherits functionality of other class. The classes are linked to one another through interfaces. What is a class? A class is a template used to create multiple objects with similar features. Every class has two components. Attributes and Behaviours. Attribute differentiates one class from other. Behaviour is the way a class of objects can do anything to themselves or others. Behaviour of a class of objects is done by methods. Methods are a group of states and accomplish a specific task. What is an abstract class? An abstract class is a class created solely for the purpose of being extended. For example, shape can be abstract class, which can be extended to create specific classes like square, triangle or a hexagon. What is a derived class? A derived class starts with an existing base class and builds upon it. For example, shape can be a base class, which can be extended to create specific classes like square, triangle or a hexagon, which are 'derived' from shape. What is the difference between C and C++? C++ is object oriented programming language using most of the primary constructs of C. What do you understand by scope resolution operator?

Question 3: Answer:

Question 4: Answer:

Question 5: Answer:

Question 6: Answer:

Question 7: Answer: Question 8:

218

Frequently Asked Questions in Management

Answer:

The scope resolution operator '::' is used to specify to which class the member (function/variable) belongs. Which feature of C++ makes it possible to use the operator '<<' for both bit wise left-shift as well as count? Operator Overloading

Question 9: Answer:

Question 10: What is the purpose of operator 'this'? Answer: The operator 'this' is used to refer to the current instance of an object (that is the object currently executing).

Question 11: What is the difference between Malloc and Calloc function in C? Answer: Malloc takes single parameter to specify the memory space to be allocated size of data type. Calloc takes two parameters, one to specify the number of blocks and the other to specify block size. Question 12: What is the function rw+ in C? Answer: While opening up the file rw+ is used to read, write and append.

Question 13: What do you mean by a "sequence point"? Answer: This is a point (at the end of a full expression, or at the ||, &&, ?:, or comma operators, or just before a function call) at which all side effects are guaranteed to be complete.

Question 14: What is a 'friend' function? Answer: When a function is declared to be a friend of a class, although this function is not a member of that class, it can access the private variables and functions of that class.

Question 15: Explain the anomaly. NULL is guaranteed to be 0, but the null pointer is not. Answer: A "null pointer" is a language concept whose particular internal value does not matter. A null pointer is requested in source code with the character "0". "NULL" is a preprocessor macro, which is always defined to be 0 by using #define syntax or ((void *) 0).

Question 16: Why does C++ have both pointers and references? Answer: C++ inherited pointers from C, and hence retains them to avoid backward compatibility problems. Even though references are useful for several things, their most important use in C++ was to support operator overloading. For example: void f1(const complex* x, const complex* y) { complex z = *x+*y; // ... } // ugly // without references

219

Information Technology

void f2(const complex& x, const complex& y) { complex z = x+y; // ... } // better

// with references

In general to use both the functionality of pointers and the functionality of references, either two different types are needed or two different sets of operations are needed on a single type. For example, with a single type, two operations are needed, one to assign to the object referred to and another to assign to the reference/pointer. This can be done using separate operators (as in Simula). For example: Ref<My_type> r :- new My_type; r := 7; r :- new My_type; // assign to object // assign to reference

The alternative is to rely on type checking (overloading). For example: Ref<My_type> r = new My_type; r = 7; r = new My_type; // assign to object // assign to reference

Question 17: How much memory does a pointer variable allocate? Answer: Only enough memory to hold the pointer itself, not any memory for the pointer to point.

Question 18: What is hashing? Answer: Hashing refers to the mapping of strings (or other data structures) to integers, for easier searching.

Question 19: Distinguish between private and protected members in a class. Answer: Private members of a class can only be accessed by methods defined as part of that class. Members declared as protected are private within a class and are available for private access only in the derived classes of the parent class in which they are declared as protected.

Question 20: What is the main difference between a structure and class? Answer: In a structure the members are public by default. They can be accessed freely, while in a class they by default are private.

Question 21: How can I declare a function that returns a pointer to a function of its own type? Answer: This can be done by using a cast, or by wrapping a struct around the pointer and returning that.

Question 22: How can a multidimensional array be allocated dynamically?

220

Frequently Asked Questions in Management

Answer:

The best method is to allocate an array of pointers, and then initialize each pointer to a dynamically allocated "row.

Question 23: What is the most important difference between arrays and pointers? Answer: Arrays automatically allocate space, which is fixed in size and location; pointers are dynamic.

Question 24: What do you mean by a null pointer? Answer: The null pointer is a pointer where the pointer variable does not refer any object (i.e. a pointer to nothing). It is usual for functions which return pointers to return NULL if they failed in some way. The null pointer is distinguishable from all other pointer values and it does not contain the address of any object or function.

Question 25: How can a linked list be sorted? Answer: Algorithms like insertion sort and merge sort can be used to sort a link list. Another way is to keep the list in order as it is built in.

Question 26: When is a constructor called? Answer: A constructor is called when an object is created.

Question 27: What is a function object? Answer: This is an object that in some way behaves like a function, of course. Typically, that would mean an object of a class that defines the application operator operator (). A function object is a more general concept than a function because a function object can have state that persist across several calls (like a static local variable) and can be initialized and examined from outside the object (unlike a static local variable). For example: class Sum { int val; public: Sum(int i) :val(i) { } operator int() const { return val; } // extract value int operator()(int i) { return val+=i; } // application }; void f(vector<int> v) { Sum s = 0; // initial value 0 s = for_each(v.begin(), v.end(), s); // gather the sum of all elements cout << "the sum is " << s << "\n"; // or even: cout << "the sum is " << for_each(v.begin(), v.end(), Sum(0)) << "\n"; } Function objects are extensively used to provide flexibility in the standard library.

221

Information Technology

JAVA
Question 1: Answer: What makes JAVA machine independent? JAVA programs create their code in a language that is interpreted by the JVM. So it is the JVM (JAVA Virtual Machine) that ensures that JAVA programs remain machine independent. In what way Java is superior to C? Java is platform independent and architecture neutral. C is designed to be compiled for a specific target. Java is object-oriented language where as C is not. Security and probability of Java are better than C. What do you understand by Byte Code? Byte Codes are highly optimized set of instructions designed to be executed by the JAVA run time system, called JVM(java virtual machine). Byte Codes are generated by compilation of JAVA source files (.java). What is an applet in JAVA? An applet is a small application that can be accessed on an Internet server. It can be transported over Internet, automatically installed, and run as part of a web document. What is the main difference between servlets and JSP? The main difference is that servlets are precompiled before execution while JSP pages are compiled at run time only. Is JavaScript the scripting language of JAVA? No, JavaScript is a scripting language with control over the browser behavior, while JAVA is a full-fledged object oriented programming language with no control over the browser behavior. What is the JAVA equivalent of the C++ code - cout<<"Hello"? system.out.println ("Hello"); What is the correct ordering for the import, class and package declarations when found in a single file? Package, import, class From which class is applet derived? Panel.

Question 2: Answer:

Question 3: Answer:

Question 4: Answer:

Question 5: Answer:

Question 6: Answer:

Question 7: Answer: Question 8: Answer: Question 9: Answer:

Question 10: What is the name of the method used to schedule a thread for execution? Answer: Start()

222

Frequently Asked Questions in Management

Question 11: What is the correct way to pass a parameter into an applet? Answer: Example: <param name=age value=33>

Question 12: What is the JAVA equivalent of the C++ macro #include ? Answer: Import is very similar to the '#include' feature of C++

Question 13: How do we ensure that a subclass does not override methods declared in the super class? Answer: Declare the methods static in the super class.

Question 14: How does one create a Thread in JAVA? Answer: By extending a class from Thread class or by implementing Runnable interface.

Question 15: How do you create animation thread in an applet? Answer: This is achieved by implementing a runnable interface to an applet.

Question 16: How are the members of a class protected in Java(like C++)? Answer: Unlike C++, protected in JAVA means that sub-classes AND classes in the same package have access. This can be achieved by the following: eclass A { protected int i; } class B { int f() { A a = new A(); a.i = 0; } } Question 17: What are the different access rules available with the different types of inheritance of the members (attributes and methods) of a class in JAVA? Answer: Java provides 5 levels of protection through different access rules in inheritance: i. ii. iii. iv. v. publicprotected default private protected private access to all only package and sub-class have access only package has access (that is any class in the package) class/subclass access only the class has access in which the members have bee defined. // in C++ this can't be done // ... in Java it can

Question 18: How can one declare a constant (like C++ const) in JAVA?

223

Information Technology

Answer:

In Java variables declared with the keyword final are non-modifiable. eg: class Test { public final static int CONST = 1; } class UseIt { public void f() { int i = Test.CONST; } }

Question 19: How can enums (like C enum) be declared in Java(tm)? Answer: This is not possible directly by using the enum keyword in C. This can be achieved by using interfaces: i. Define the interface: "public interface TOKEN { public static final int IDENT = 1; public static final int KEYWORD = 2; } ii. Implement that interface, like: public class parser implements TOKEN { } iii. The methods in parser will be able to refer to IDENT all by itself.

224

Frequently Asked Questions in Management

ERP & ORACLE FINANCIAL


Question 1: Answer: What are the benefits out of implementing ERP? How do you evaluate them? Benefits a) b) c) Question 2: Answer: Improve business cycle times (like order delivery, delivery collection, etc.) Reduce cost (like Inventory carrying cost) Better decision making

What are the problems and issues faced in implementing ERP Projects? How do you address them? Problems faced in implementing ERP Projects 1. 2. 1. 2. 3. 4. Understanding of the customer requirements and current business process clearly Standardization of processes or codification schemes implementation End user involvement Seek top management support Training Change management

How to address the problems

Question 3: Answer:

How would you implement ERP? Steps involved in total ERP implementation are as followed:a) b) c) d) e) f) g) h) i) Identification of needs for implementation of ERP. Evaluating present business situation. Deciding upon designed business situation. Finalizing the ERP consultants. Reengineering business processes. Evaluation of various ERP packages. Finalizing of ERP package. Installation of hardware and software. Implementation of ERP package.

Question 4: Answer:

What is ORACLE Financials? ORACLE Financials products provide organizations with solutions to a wide range of long- and short-term accounting system issues. ORACLE Assets: Ensures that an organizations property and equipment investment is accurate and that the correct asset tax accounting strategies are chosen.

225

Information Technology

ORACLE General Ledger: ORACLE Inventory: ORACLE Order Entry: ORACLE Payables:

Offers a complete solution to journal entry, budgeting, allocations, consolidation, and financial reporting needs. Helps an organization make better inventory decisions by minimizing stock and maximizing cash flow. Provides organizations with a sophisticated order entry system for managing customer commitments. Lets an organization process more invoices with fewer staff members and tighter controls. Helps save money through maximum discounts, bank float, and prevention of duplicate payment. Improves the management of employee- related issues by retaining and making available every form of personnel data. Improves buying power, helps negotiate bigger discounts, eliminates paper flow, increases financial controls, and increases productivity. Improves cash flow by letting an organization process more payments faster, without off-line research. Helps correctly account for cash, reduce outstanding receivables, and improve collection effectiveness.

ORACLE Personnel:

ORACLE Purchasing:

ORACLE Receivables:

ORACLE Revenue Accounting: Gives an organization timely and accurate revenue and flexible commissions reporting. ORACLE Sales Analysis: Allows for better forecasting, planning. and reporting of sales information

Question 5: Answer Question 6: Answer:

What are known as 3 Cs in ORACLE Applications? Chart of Account, Currency and Calendar What are the various modules in ORACLE application (Materials Management, production planning, sales and distribution etc)? a. b. c. d. e. f. g. h. i. j. k. Asset Management Financials Human Resources Manufacturing Marketing Order Management Procurement Projects Sales Service Supply Chain

226

Frequently Asked Questions in Management

Question 7: Answer:

What is the most important module in ORACLE Financials? The General Ledger (GL) module is the basis for all other ORACLE Financial modules. All other modules provide information to it. If you implement ORACLE Financials, you should switch your current GL system first. GL is relatively easy to implement. You should go live with it first to give your implementation team a chance to be familiar with ORACLE Financials. What is the MultiOrg and what is it used for? MultiOrg or Multiple Organizations Architecture allows multiple operating units and their relationships to be defined within a single installation of ORACLE Applications. This keeps each operating units transaction data separate and secure. Use the following query to determine if MultiOrg is installed: Select multi_org_flag from fnd_product_groups;

Question 8: Answer:

Question 9: Answer:

What are the different invoice options? What is the difference between PO match and PO default? What happens when you use PO match? Standard / Quick Match / PO Default / Expense Report / Prepayment / Debit memo / Credit memo / Mixed. Quick Match Invoice - matches the entire PO shipment line amount and quantity for all the lines of the PO to create the invoice distribution lines. The default values can be overridden at the time of entry. It is only for quick and ease of PO matched Invoice creation. In a PO default - the PO shipment or distribution line has to be selected prior to matching an invoice. After matching with a PO shipment line / distribution line, the invoice distribution line with an amount equals the quantity times the amount is automatically created in the invoice. The account is the Invoice AP accrual account, which defaults from the PO.

227

You might also like